250
2015 FINANCIAL ACCOUNTING AND REPORTING II QUESTION BANK CAF-07

CAF7-Financial Accounting and Reporting II_Questionbank

Embed Size (px)

DESCRIPTION

financial reporting question book for ca students

Citation preview

Page 1: CAF7-Financial Accounting and Reporting II_Questionbank

2015

FINANCIAL ACCOUNTINGAND REPORTING IIQUESTION BANK

CAF-07

Page 2: CAF7-Financial Accounting and Reporting II_Questionbank
Page 3: CAF7-Financial Accounting and Reporting II_Questionbank

Question Bank IC

AP

P

Financial accounting and reporting II

Page 4: CAF7-Financial Accounting and Reporting II_Questionbank

© Emile Woolf International ii The Institute of Chartered Accountants of Pakistan

Second edition published by Emile Woolf Limited Bracknell Enterprise & Innovation Hub Ocean House, 12th Floor, The Ring Bracknell, Berkshire, RG12 1AX United Kingdom Email: [email protected] www.emilewoolf.com © Emile Woolf International, February 2015 All rights reserved. No part of this publication may be reproduced, stored in a retrieval system, or transmitted, in any form or by any means, electronic, mechanical, photocopying, recording, scanning or otherwise, without the prior permission in writing of Emile Woolf Publishing Limited, or as expressly permitted by law, or under the terms agreed with the appropriate reprographics rights organisation. You must not circulate this book in any other binding or cover and you must impose the same condition on any acquirer. Notice Emile Woolf International has made every effort to ensure that at the time of writing the contents of this study text are accurate, but neither Emile Woolf International nor its directors or employees shall be under any liability whatsoever for any inaccurate or misleading information this work could contain.

Page 5: CAF7-Financial Accounting and Reporting II_Questionbank

© Emile Woolf International iii The Institute of Chartered Accountants of Pakistan

Certificate in Accounting and Finance Financial accounting and reporting II

C

Contents

Page

Question and Answers Index v

Questions

Section A Questions 1

Answers

Section B Answers 93

Page 6: CAF7-Financial Accounting and Reporting II_Questionbank

Financial accounting and reporting II

© Emile Woolf International iv The Institute of Chartered Accountants of Pakistan

Page 7: CAF7-Financial Accounting and Reporting II_Questionbank

© Emile Woolf International v The Institute of Chartered Accountants of Pakistan

Certificate in Accounting and Finance Financial accounting and reporting II

I

Index to questions and answers

Question

page Answer

page

CHAPTER 2 – IAS 1: PRESENTATION OF FINANCIAL STATEMENTS

2.1 LARRY 2 94

2.2 MINGORA IMPORTS LIMITED 3 95

2.3 BARRY 4 97

2.4 OSCAR INC 6 99

2.5 CLIFTON PHARMA LIMITED 7 101

2.6 SARHAD SUGAR LIMITED 8 103

2.7 BSZ LIMITED 10 105

2.8 YASIR INDUSTRIES LIMITED 12 108

2.9 SHAHEEN LIMITED 14 112

2.10 MOONLIGHT PAKISTAN LIMITED 15 114

2.11 FIGS PAKISTAN LIMITED 17 115

CHAPTER 3 – IAS 7: STATEMENTS OF CASH FLOWS

3.1 KLEA 19 119

3.2 STANDARD INC 21 121

3.3 FALLEN 22 124

3.4 BIN QASIM MOTORS LIMITED 24 126

3.5 ITTEHAD MANUFACTURING LTD 27 129

3.6 WASEEM INDUSTRIES LIMITED 29 131

3.7 JALIB INDUSTRIES LIMITED 31 134

Page 8: CAF7-Financial Accounting and Reporting II_Questionbank

Financial accounting and reporting II

© Emile Woolf International vi The Institute of Chartered Accountants of Pakistan

Question

page Answer

page

3.8 APOLLO INDUSTRY LIMITED 32 136

3.9 MARVEL ENGINEERING LIMITED 34 137

CHAPTER 4 – CONSOLIDATED ACCOUNTS: STATEMENTS OF FINANCIAL POSITION – BASIC APPROACH

4.1 HALL 36 139

4.2 HASSLE 37 140

4.3 HYMN 37 141

4.4 HANG 38 143

4.5 HASH 38 144

CHAPTER 5 – CONSOLIDATED ACCOUNTS: STATEMENTS OF FINANCIAL POSITION - COMPLICATIONS

5.1 HAIL 39 146

5.2 HAIRY 40 148

5.3 HARD 41 150

5.4 HALE 42 152

5.5 HELLO 42 153

5.6 HASAN LIMITED 43 155

CHAPTER 6 – CONSOLIDATED ACCOUNTS: STATEMENTS OF COMPREHENSIVE INCOME

6.1 HARRY 45 159

6.2 HORNY 46 161

6.3 HERON 47 163

6.4 HANKS 48 164

CHAPTER 7 – PROPERTY, PLANT AND EQUIPMENT

7.1 ROONEY 50 168

7.2 EHTISHAM 51 170

7.3 CARLY 51 172

7.4 ADJUSTMENTS LIMITED 52 173

7.5 FAM 53 175

7.6 IMRAN LIMITED 54 177

7.7 HUMAYUN CHEMICALS LIMITED 55 179

7.8 FARADAY PHARMACEUTICAL LIMITED 55 180

Page 9: CAF7-Financial Accounting and Reporting II_Questionbank

Index to questions and answers

© Emile Woolf International vii The Institute of Chartered Accountants of Pakistan

Question

page Answer

page

7.9 SPIN INDUSTRIES LIMITED 56 182

7.10 SCIENTIFIC PHARMA LIMITED 56 183

7.11 QURESHI STEEL LIMITED 57 184

7.12 GRANITE CORPORATION 58 185

CHAPTER 8 – IAS 38: INTANGIBLE ASSETS

8.1 FAZAL 59 186

8.2 HENRY 59 186

8.3 TOBY 60 187

8.4 BROOKLYN 60 188

8.5 ZOUQ INC 61 189

8.6 STAR-BRIGHT PHARMACEUTICAL LIMITED

62 190

8.7 RAISIN INTERNATIONAL 62 191

CHAPTER 9 – IAS 17: LEASES

9.1 DAWOOD 64 192

9.2 FINLEY 64 192

9.3 FABIAN 64 193

9.4 XYZ INC 65 194

9.5 SNOW INC 65 197

9.6 MIRACLE TEXTILE LIMITED 66 199

9.7 SHOAIB LEASING LIMITED 66 200

9.8 NEPTUNE LIMITED 67 202

9.9 QUARTZ AUTO LIMITED 67 204

9.10 LODHI TEXTILE MILLS LIMITED 68 205

9.11 NOMAN ENGINEERING LIMITED 68 206

CHAPTER 10 – IAS 37: PROVISIONS CONTINGENT LIABILITIES AND CONTINGENT ASSETS AND IAS 10: EVENTS OCCURRING AFTER THE REPORTING DATE

10.1 BADAR 69 207

10.2 GEORGINA 69 207

10.3 EARLEY INC 70 209

10.4 ACCOUNTING TREATMENT 70 210

Page 10: CAF7-Financial Accounting and Reporting II_Questionbank

Financial accounting and reporting II

© Emile Woolf International viii The Institute of Chartered Accountants of Pakistan

Question

page Answer

page

10.5 J-MART LIMITED 71 211

10.6 AKBER CHEMICALS LIMITED 72 212

10.7 QALLAT INDUSTRIES LIMITED 72 213

10.8 SKYLINE LIMITED 73 213

10.9 WALNUT LIMITED 74 214

10.10 ATTOCK TECHNOLOGIES LIMITED 75 215

CHAPTER 11 – IAS 8: ACCOUNTING POLICIES, CHANGES IN ACCOUNTING ESTIMATES AND ERRORS

11.1 WONDER LIMITED 76 216

11.2 DUNCAN 77 217

11.3 MOHANI MANUFACTURING LIMITED 78 218

CHAPTER 12 – IAS 12: INCOME TAXES

12.1 FRANCESCA 79 219

12.2 SHEP (I) 79 220

12.3 SHEP (II) 80 221

12.4 SHEP (III) 81 222

12.5 SHEP (IV) 82 224

12.6 WAQAR LIMITED 82 225

12.7 SHAKIR INDUSTRIES 83 227

12.8 MARS LIMITED 84 228

12.9 BILAL ENGINEERING LIMITED 85 230

12.10 GALAXY INTERNATIONAL 85 231

12.11 APRICOT LIMITED 86 232

CHAPTER 13 – RATIO ANALYSIS

13.1 WASIM 87 233

13.2 AMIR AND MO 88 233

CHAPTER 14 – ETHICAL ISSUES IN FINANCIAL REPORTING

14.1 ETHICAL ISSUES 90 235

14.2 SINDH INDUSTRIES LTD 90 236

Page 11: CAF7-Financial Accounting and Reporting II_Questionbank

© Emile Woolf International 1 The Institute of Chartered Accountants of Pakistan

Certificate in Accounting and Finance Financial accounting and reporting II

S E

C T

I O

N

A

Questions

Page 12: CAF7-Financial Accounting and Reporting II_Questionbank

Financial accounting and reporting II

© Emile Woolf International 2 The Institute of Chartered Accountants of Pakistan

CHAPTER 1 – LEGAL BACKGROUND TO THE PREPARATION OF FINANCIAL

STATEMENTS

There are no questions specific to chapter one. This is because the learning outcomes in this area concern the preparation of financial statements and these questions have given in relation to chapter 2 in this question bank.

CHAPTER 2 – IAS 1: PRESENTATION OF FINANCIAL STATEMENTS

2.1 LARRY

The trial balance of Larry at 31 December 2015 is as follows.

Rupees in million Dr Cr

Administration charges 342

Bank account 89 Cash 2 Payables’ ledger 86 Accumulated amortisation on patents at 31 December 2015 5 Accumulated depreciation at 31 December 2015 918 Receivables’ ledger 189 Distribution expenses 175 Property, plant and equipment at cost 2,830 Interest received 20 Issued share capital 400 Loan 18 Patents at cost 26 Accumulated profits 1,562 Purchases 2,542 Sales 3,304 Inventories at 31 December 2014 118 ––––– ––––– 6,313 6,313 ════ ════

The following information is also relevant.

(1) Inventories on 31 December 2015 amounted to Rs. 127 million.

(2) Current tax of Rs. 75 million is to be provided.

(3) The loan is repayable by equal annual instalments over three years.

Required

Prepare an statement of profit or loss (analysing expenses by function) for the year ended 31 December 2015 and a statement of financial position as at that date.

Page 13: CAF7-Financial Accounting and Reporting II_Questionbank

Questions

© Emile Woolf International 3 The Institute of Chartered Accountants of Pakistan

2.2 MINGORA IMPORTS LIMITED

The trial balance of Mingora Imports Limited at 31 December 2015 is as follows.

Rupees in million Dr Cr

Patent rights 60

Work-in-progress, 1 January `2015 125 Leasehold buildings at cost 300 Ordinary share capital 600 Sales 1,740 Staff costs 260 Accumulated depreciation on buildings, 1 January 2015 60 Inventories of finished games, 1 January 2015 155 Consultancy fees 44 Directors’ salaries 360 Computers at cost 50 Accumulated depreciation on computers, 1 January 2015 20 Dividends paid 125 Cash 440 Receivables 420 Trade payables 92 Sundry expenses 294 Accumulated profits, 1 January 2015 121 –––––– –––––– 2,633 2,633 ════ ════

The following information is also relevant.

(1) Closing inventories of finished games are valued at Rs. 180 million. Work in progress has increased to Rs. 140 million.

(2) The patent rights relate to a computer program with a three year lifespan.

(3) On 1 January 2015 buildings were revalued to Rs. 360 million. This has not yet been reflected in the accounts. Computers are depreciated over five years. Buildings are now to be depreciated over 30 years.

(4) An allowance for bad debts (irrecoverable debts) of 5% is to be created.

(5) There is an estimated bill for current tax of Rs. 120 million which has not yet been recognised.

Required

Prepare an statement of profit or loss (analysing expenses by nature for the year ended 31 December 2015 and a statement of financial position as at that date.

Page 14: CAF7-Financial Accounting and Reporting II_Questionbank

Financial accounting and reporting II

© Emile Woolf International 4 The Institute of Chartered Accountants of Pakistan

2.3 BARRY

Barry has prepared the following draft financial statements for your review Barry: Statement of profit or loss for year to 31st August 2015 Rs. in

‘000

Sales revenue 30,000 Raw materials consumed (9,500)

Manufacturing overheads (5,000)

Increase in inventories of work in progress and finished goods 1,400

Staff costs (4,700)

Distribution costs (900)

Depreciation (4,250)

Interest payable (350) –––––– 6,700 ════

Statement of financial position as at 31st August 2015

Rs. in ‘000

Rs. in ‘000

Assets

Non-current

Freehold land and buildings 20,000

Plant and machinery 14,000

Fixtures and fittings 5,600 ––––––– 39,600

Current assets

Prepayments 200

Trade receivables 7,400

Cash at bank 700

Inventories 4,600 –––––––

12,900 ––––––– Total assets 52,500 ––––––– Equity and liabilities

Equity shares of Rs. 1 each 21,000

Accumulated profit 14,000

Share premium 2,000 ––––––– Total equity 37,000

Revaluation surplus 5,000

Current liabilities 5,300

Non-current liabilities

8% Debentures 2019 5,200 ––––––– Total equity and liabilities 52,500 ═════

Page 15: CAF7-Financial Accounting and Reporting II_Questionbank

Questions

© Emile Woolf International 5 The Institute of Chartered Accountants of Pakistan

Additional information

1 Income tax of Rs. 2.1 million has yet to be provided for on profits for the current year. An unpaid under-provision for the previous year’s liability of Rs. 400,000 has been identified on 5th September 2015 and has not been reflected in the draft accounts.

2 There have been no additions to, or disposals of, non-current assets in the

year but the assets under construction have been completed in the year at an additional cost of Rs. 50,000. These related to plant and machinery.

The cost and accumulated depreciation of non-current assets as at 1st September 2014 were as follows:

Cost Depreciation

Rs. in ‘000 Rs. in ‘000

Freehold land and buildings 19,000 3,000 (land element Rs. 10 million) Plant and machinery 20,100 4,000 Fixtures and fittings 10,000 3,700 Assets under construction 400 -

3 There was a revaluation of land and buildings during the year, creating the revaluation surplus of Rs. 5 million (land element Rs. 1 million). The effect on depreciation has been to increase the buildings charge by Rs. 300,000. Barry adopts a policy of transferring the revaluation surplus included in equity to retained earnings as it is realised.

4 Staff costs comprise 70% factory staff, 20% general office staff and 10% goods delivery staff

5 An analysis of depreciation charge shows the following:

Rs. in ‘000

Buildings (50% production, 50% administration) 1,000 Plant and machinery 2,550 Fixtures and fittings (30% production, 70% administration) 700

Required

Prepare the following information in a form suitable for publication for Barry’s financial statements for the year ended 31st August 2015.

Statement of profit or loss

Statement of financial position

Reconciliation of opening and closing property, plant and equipment (25)

Page 16: CAF7-Financial Accounting and Reporting II_Questionbank

Financial accounting and reporting II

© Emile Woolf International 6 The Institute of Chartered Accountants of Pakistan

2.4 OSCAR INC

The following trial balance has been extracted from the books of accounts of Oscar Inc as at 31 March 2015.

Rs. in ‘000 Dr Cr

Administrative expenses 210 Share capital 600 Receivables 470 Bank overdraft 80 Income tax (overprovision in 2014) 25 Provision 180 Distribution costs 420 Non-current investments 560 Investment income 75 Plant and machinery At cost 750 Accumulated depreciation (at 31 March 2015) 220 Retained earnings (at 1 April 2014) 180 Purchases 960 Inventory (at 1 April 2014) 140 Trade payables 260 Sales revenue 2,010 Interim dividend paid 120

3,630 3,630

Additional information

(1) Inventory at 31 March 2015 was valued at Rs. 150,000.

(2) The income tax charge based on the profits on ordinary activities is estimated to be Rs. 74,000.

(3) The provision is to be increased by Rs. 16,000.

(4) There were no purchases or disposals of fixed assets during the year.

Required

Prepare the company’s statement of profit or loss for the year to 31 March 2015 and a statement of financial position as at that date in accordance with IAS 1. (18)

Page 17: CAF7-Financial Accounting and Reporting II_Questionbank

Questions

© Emile Woolf International 7 The Institute of Chartered Accountants of Pakistan

2.5 CLIFTON PHARMA LIMITED

The following trial balance relates to Clifton Pharma Limited, a public listed company, at 30 September 2015.

Rs. in ‘000 Dr Cr

Cost of sales 134,000

Operating expenses 35,000

Loan interest paid (see note (1)) 1,500

Rental of vehicles (see note (2)) 8,600

Revenue 338,300

Investment income 2,000

Leasehold property at cost (see note (4)) 250,000

Plant and equipment at cost 197,000

Accumulated depreciation at 1 October 2014:

- leasehold property 40,000

- plant and equipment 47,000

Investments 92,400

Share capital 280,000

Share premium 20,000

Retained earnings at 1 October 2014 19,300

Loan notes (see note (1)) 50,000

Deferred tax balance at 1 October 2014 (see note (5)) 20,000

Inventory at 30 September 2015 23,700

Trade receivables 76,400

Trade payables 14,100

Bank 12,100

830,700 830,700

The following notes are relevant

(1) The effective interest rate on the loan notes is 6% per year.

(2) There are two separate contracts for rental of vehicles. A recent review by the finance department of these contracts has reached the conclusion that Rs. 7 million of the total rental cost of vehicles relates to a finance lease rather than an operating lease or rental arrangement.

The finance lease was entered into on 1 October 2014 which was when the Rs. 7 million was paid: the lease agreement is for a four-year period in total, and there will be three more annual payments in advance of Rs. 7 million, payable on 1 October in each year. The vehicles in the finance lease agreement had a fair value of Rs. 24 million at 1 October 2014 and they should be depreciated using the straight line method to a nil residual value. The interest rate implicit in the lease is 10% per year. The other contract for vehicle rental is an operating lease and the rental payment should be charged to operating expenses. (Note: You are not required to calculate the present value of the minimum lease payments for the finance lease.)

(3) Other plant and equipment is depreciated at 20% per year by the reducing balance method.

All depreciation of property, plant and equipment should be charged to cost of sales.

Page 18: CAF7-Financial Accounting and Reporting II_Questionbank

Financial accounting and reporting II

© Emile Woolf International 8 The Institute of Chartered Accountants of Pakistan

(4) The leasehold property has a 25-year life and is amortised at a straight-line rate. On 30 September 2015 the leasehold property was re-valued to Rs. 220 million and the directors wish to incorporate this re-valuation in the financial statements.

(5) The provision for income tax for the year ended 30 September 2015 has been estimated at Rs. 18 million. At 30 September 2015 there are taxable temporary differences of Rs. 92 million. The rate of income tax on profits is 25%.

Required

(a) Prepare an statement of profit or loss for Clifton Pharma Limited for the year to 30 September 2015 (8)

(b) Prepare a statement of financial position (balance sheet) for Clifton Pharma Limited as at 30 September 2015 (17)

(25)

2.6 SARHAD SUGAR LIMITED

The following trial balance relates to Sarhad Sugar Limited at 30 September 2015:

Rs. in ‘000 Dr Cr

Leasehold property – at valuation 1 October 2014 (note (i)) 50,000

Plant and equipment – at cost (note (i)) 76,600

Plant and equipment – accumulated depreciation at 1 October 2014

24,600

Capitalised development expenditure – at 1 October 2014 (note (ii)) 20,000

Development expenditure – accumulated amortisation at 1 October 2014

6,000

Closing inventory at 30 September 2015 20,000

Trade receivables 43,100 Bank 1,300 Trade payables and provisions (note (iii)) 23,800

Revenue (note (i)) 300,000

Cost of sales 204,000

Distribution costs 14,500

Administrative expenses (note (iii)) 22,200

Interest on bank borrowings 1,000

Equity dividend paid 6,000

Research and development costs (note (ii)) 8,600

Share capital 70,000

Retained earnings at 1 October 2014 24,500

Deferred tax (note (v)) 5,800

Revaluation surplus (Leasehold property) 10,000

466,000 466,000

The following notes are relevant:

(i) Non-current assets – tangible:

The leasehold property had a remaining life of 20 years at 1 October 2014. The company’s policy is to revalue its property at each year end and at 30 September 2015 it was valued at Rs. 43 million.

Page 19: CAF7-Financial Accounting and Reporting II_Questionbank

Questions

© Emile Woolf International 9 The Institute of Chartered Accountants of Pakistan

On 1 October 2014 an item of plant was disposed of for Rs. 2·5 million cash. The proceeds have been treated as sales revenue by Sarhad Sugar Limited. The plant is still included in the above trial balance figures at its cost of Rs. 8 million and accumulated depreciation of Rs. 4 million (to the date of disposal).

All plant is depreciated at 20% per annum using the reducing balance method.

Depreciation and amortisation of all non-current assets is charged to cost of sales.

(ii) Non-current assets – intangible:

In addition to the capitalised development expenditure (of Rs. 20 million), further research and development costs were incurred on a new project which commenced on 1 October 2014. The research stage of the new project lasted until 31 December 2014 and incurred Rs. 1·4 million of costs. From that date the project incurred development costs of Rs. 800,000 per month. On 1 April 2015 the directors became confident that the project would be successful and yield a profit well in excess of its costs. The project is still in development at 30 September 2015.

Capitalised development expenditure is amortised at 20% per annum using the straight-line method. All expensed research and development is charged to cost of sales.

(iii) Sarhad Sugar Limited is being sued by a customer for Rs. 2 million for breach of contract over a cancelled order. Sarhad Sugar Limited has obtained legal opinion that there is a 20% chance that Sarhad Sugar Limited will lose the case. Accordingly Sarhad Sugar Limited has provided Rs. 400,000 (Rs. 2 million x 20%) included in administrative expenses in respect of the claim. The unrecoverable legal costs of defending the action are estimated at Rs. 100,000. These have not been provided for as the legal action will not go to court until next year.

(iv) The directors have estimated the provision for income tax for the year ended 30 September 2015 at Rs. 11·4 million. The required deferred tax provision at 30 September 2015 is Rs. 6 million.

Required

(a) Prepare the statement of profit or loss for the year ended 30 September 2015. (10)

(b) Prepare the statement of financial position as at 30 September 2015. (10)

Note: notes to the financial statements are not required. (20)

Page 20: CAF7-Financial Accounting and Reporting II_Questionbank

Financial accounting and reporting II

© Emile Woolf International 10 The Institute of Chartered Accountants of Pakistan

2.7 BSZ LIMITED

The post-closing trial balance of BSZ Limited, a listed company, as at June 30, 2015 is given below: Debit Credit

Rs. in million

Cash at banks – current accounts 7

Cash at banks – in saving accounts 22

Stocks in trade – closing 90

Accounts receivable 60

Provision for bad debts 3

Advances to suppliers 16

Advances to staff 6

Short term deposits 11

Prepayments 4

Sales tax receivable 12

Freehold land – at revalued amount 375

Furniture and fixtures - cost 27

Accumulated depreciation – Furniture and fixtures 8

Machines - cost 85

Accumulated depreciation – Machines 27

Building on freehold land – cost 150

Accumulated depreciation – Building 26

Computer software – cost 10

Accumulated amortization – Computer software 2

Deferred taxation 40

Short term loan 85

Accounts payable 75

Accrued liabilities 7

Provision for taxation 17

Issued, subscribed and paid up capital (Rs. 10 each) 400

Surplus on revaluation of fixed assets 120

Accumulated profits 65

875 875

Additional Information

(i) The first revaluation of freehold land was carried out in 2011 and resulted in a surplus of Rs. 120 million. The valuation was carried out under market value basis by an independent valuer, Mr. Dee, Chartered Civil Engineer of M/s SSS Consultants (Pvt.) Ltd., Islamabad.

(ii) The details relating to additions, disposal and depreciation/amortization of fixed assets, during the year 2015 are given below:

The company uses the straight line method for charging depreciation and amortization. The building is depreciated at a rate of 5% whereas 10% is charged on machines, furniture and fixtures and computer software.

Construction on third floor of the building commenced on March 1, 2015 and is expected to be completed on September 30, 2015. The cost incurred during the year i.e. Rs. 20 million was capitalised on June 30, 2015.

Page 21: CAF7-Financial Accounting and Reporting II_Questionbank

Questions

© Emile Woolf International 11 The Institute of Chartered Accountants of Pakistan

Furniture and fixtures worth Rs. 8 million were purchased on April 1, 2015.

A machine was sold on February 28, 2015 to NJ Enterprise at a price of Rs. 13 million. At the time of disposal, the cost and written down value of the machine was Rs. 15 million and Rs. 10 million respectively.

(iii) 50% of the accounts receivable were secured and considered good. 10% of the unsecured accounts receivable were considered doubtful. Bad debts expenses for the year amounted to Rs. 1.0 million. An amount of Rs. 1.4 million was written off during the year.

(iv) All advances given to suppliers are considered good and include an amount of Rs. 4.0 million paid for goods which will be supplied on December 31, 2016.

(v) Cash at banks in saving accounts carry interest / mark-up ranging from 3% to 7% per annum.

(vi) The authorised share capital of the company is Rs. 500 million.

Required

Prepare the statement of financial position as at June 30, 2015 along with the relevant notes showing all possible disclosures as required under the International Accounting Standards and the Companies Ordinance, 1984.

(Comparative figures and the note on accounting policies are not required.)(22)

Page 22: CAF7-Financial Accounting and Reporting II_Questionbank

Financial accounting and reporting II

© Emile Woolf International 12 The Institute of Chartered Accountants of Pakistan

2.8 YASIR INDUSTRIES LIMITED

The following trial balance related to Yasir Industries Limited (YIL) for the year ended June 30, 2015:

Dr Cr

Rs. in million

Ordinary share capital (Rs. 10 each) - 120.00

Retained earnings - 10.20

Sales - 472.40

Purchases 175.70 -

Production labour 61.00

Manufacturing overheads 39.00

Inventories (July 1, 2014) 38.90

Administrative expenses 40.00 -

Distribution expenses 19.80 -

Financial charges 0.30 -

Cash and bank - 13.25

Trade creditors - 30.40

Accrued expenses - 16.20

10% redeemable preference shares - 40.00

Debentures - 80.00

Deferred tax (July 1, 2014) - 6.00

Suspense account 30.00 -

Leasehold property - at cost 230.00 -

Machines – at cost 168.60 -

Software – at cost 20.00 -

Acc. depreciation – Leasehold property (June 30, 2015) - 40.25

Acc. depreciation – Machines (June 30, 2015) - 48.60

Acc. amortization – Software (June 30, 2015) - 12.00

Trade receivables 66.00 -

889.30 889.30

Additional Information

(i) Sales include an amount of Rs. 27 million, made to a customer under sale or return agreement. The sale has been made at cost plus 20% and the expiry date for the return of these goods is July 31, 2015.

(ii) The value of inventories at June 30, 2015 was Rs. 42 million.

(iii) A fraud of Rs. 30 million was discovered in October 2014. A senior employee of the company who left in June 2014, had embezzled the funds from YIL’s bank account. The chances of recovery are remote. The amount is presently appearing in the suspense account.

(iv) On January 1, 2015 YIL issued debenture certificates which are repayable in 2020. Interest is paid on these at 12% per annum.

(v) Financial charges comprise bank charges and bank commission.

(vi) The provision for current taxation for the year ended June 30, 2015 after making all the above adjustments is estimated at Rs. 16.5 million.

Page 23: CAF7-Financial Accounting and Reporting II_Questionbank

Questions

© Emile Woolf International 13 The Institute of Chartered Accountants of Pakistan

(vii) The carrying value of YIL’s net assets as on June 30, 2015 exceeds their tax base by Rs. 30 million. The income tax rate applicable to the company is 30%.

(viii) On July 1, 2014, the leasehold property having a useful life of 40 years was revalued at Rs. 238 million. No adjustment in this regard has been made in the books.

(ix) Depreciation of leasehold property is charged using the straight line method. 50% of depreciation is allocated to manufacturing, 30% to administration and 20% to selling and distribution.

Required

In accordance with the requirements of the Companies Ordinance, 1984 and International Accounting Standards, prepare the:

(a) statement of financial position as of June 30, 2015.

(b) statement of profit or loss for the year ended June 30, 2015. (20)

(Comparative figures and notes to the financial statements are not required.)

Page 24: CAF7-Financial Accounting and Reporting II_Questionbank

Financial accounting and reporting II

© Emile Woolf International 14 The Institute of Chartered Accountants of Pakistan

2.9 SHAHEEN LIMITED

Following is the trial balance of Shaheen Limited (SL) as at June 30, 2015:

Rs. in ‘000 Dr Cr

Sales revenue 200,000 Manufacturing costs 100,000 Selling and distribution costs 35,000 Administrative costs 30,000 Opening inventories 23,000 Interest on borrowings 5,000 Provision for income tax 2,000 Advance income tax paid 6,000 Property, plant and equipment 86,000 Accumulated depreciation on property, plant and equipment 12,000 Export licence 6,000 Trade receivables 37,800 Cash and bank balances 4,725 Other receivable and prepayments 14,000 Trade payables 12,000 Provisions for litigation 5,000 Long term borrowings 31,525 Deferred tax 5,000 Share capital (Rs. 10 each and fully paid) 60,000 Retained earnings 20,000

347,525 347,525

Additional information

(i) Sales last year (year ended 30 June 2014) included goods invoiced at Rs 10 million which were sent to a customer on June 25, 2014 under a sale or return agreement, at cost plus 20%. The goods were returned on August 25, 2014. No correction has been made for the return.

(ii) The export licence has been obtained for exporting a new product and is effective for five years up to December 31, 2019. However, the exports commenced from July 1, 2015.

(iii) Closing inventories are valued at Rs. 30 million.

(iv) Details of property, plant and equipment are as follows:

Land Buildings Plant and equipment

Rs in ‘000 Cost as at June 30, 2014

20,000 36,000 30,000

Fully depreciated amounts included in cost 3,000

Estimated useful life at the date of purchase 20 years 10 years

The company uses straight line method for charging depreciation. Depreciation is allocated to manufacturing, distribution and administrative costs at 75%, 15% and 10% respectively.

(v) Rs. 6 million of the long term borrowings is of current maturity (i.e. will be repaid within 12 months).

Page 25: CAF7-Financial Accounting and Reporting II_Questionbank

Questions

© Emile Woolf International 15 The Institute of Chartered Accountants of Pakistan

(vi) During the year Rs. 5 million was paid in full and final settlement of income tax liability against which a provision of Rs. 7.0 million had been made in the previous year. Current year’s taxable income exceeds accounting income by Rs. 5 million of which 0.8 million are permanent differences. Applicable tax rate for the company is 35%.

(vii) On July 30, 2015 the board of directors proposed a final dividend at 15% for the year ended June 30, 2015 (2014: at 20%)

Required

In accordance with the requirements of the Companies Ordinance, 1984 and International Financial Reporting Standards, prepare:

(a) The statement of financial position as of June 30, 2015

(b) The statement of profit or loss for the year ended June 30, 2015

(c) The statement of changes in equity for the year ended June 30, 2015.

(Comparative figures and notes to the financial statements are not required) (25)

2.10 MOONLIGHT PAKISTAN LIMITED

Following is the summarised trial balance of Moonlight Pakistan Limited (MPL), a listed company, for the year ended December 31, 2015:

Rs. in million

Debit Credit

Land and buildings - at cost 2,600 -

Plants – at cost 2,104 -

Trade receivables 702 -

Stock in trade at December 31, 2015 758 -

Cash and bank 354 -

Cost of sales 1,784 -

Selling expenses 220 -

Administrative expenses 250 -

Financial charges 210 -

Accumulated depreciation as on January 1, 2015 – Buildings - 400

Accumulated depreciation as on January 1, 2015 – Plants - 670

Ordinary shares of Rs. 10 each fully paid - 1,200

Retained earnings as at January 1, 2015 - 510

12% Long term loan - 1,600

Provision for gratuity - 8

Deferred tax on January 1, 2015 - 22

Trade payables - 544

Right subscription received - 420

Revenue - 3,608

8,982 8,982

Page 26: CAF7-Financial Accounting and Reporting II_Questionbank

Financial accounting and reporting II

© Emile Woolf International 16 The Institute of Chartered Accountants of Pakistan

Additional Information

(i) The land and buildings were acquired on January 1, 2011. The cost of land was Rs. 600 million. On January 1, 2015 a professional valuation firm valued the buildings at Rs. 1,840 million with no change in the value of land. The estimated life at acquisition was 20 years and the remaining life has not changed as a result of the valuation. 60% of depreciation on buildings is allocated to manufacturing, 25% to selling and 15% to administration.

(ii) Plant is depreciated at 20% per annum using the reducing balance method.

(iii) On March 31, 2015 MPL made a bonus issue of one share for every six held. The issue has not been recorded in the books of account.

(iv) Right shares were issued on September 1, 2015 at Rs. 12 per share.

(v) The interest on long term loan is payable on the first day of July and January. No accrual has been made for the interest payable on January 1, 2013.

(vi) MPL operates an unfunded gratuity scheme for all its eligible employees. The provision required as on December 31, 2015 is estimated at Rs. 23 million. Rs. 3 million were paid during the year and debited to the provision for gratuity account. Cost of gratuity is allocated to production, selling and administration expenses in the ratio of 60% : 20% : 20%.

(vii) The tax charge for the current year after making all related adjustments is estimated at Rs. 37 million. The timing differences related to taxation are estimated to increase by Rs. 80 million, over the last year. The applicable income tax rate is 35%.

Required

In accordance with the requirements of Companies Ordinance, 1984 and International Financial Reporting Standards, prepare the following:

(a) Statement of Financial Position as of December 31, 2015.

(b) Statement of profit or loss for the year ended December 31, 2015. (22)

(Comparative figures and notes to the financial statements are not required)

Page 27: CAF7-Financial Accounting and Reporting II_Questionbank

Questions

© Emile Woolf International 17 The Institute of Chartered Accountants of Pakistan

2.11 FIGS PAKISTAN LIMITED

Figs Pakistan Limited is a listed company engaged in the business of manufacturing and marketing of personal care and food products. Following is an extract from its trial balance for the year ended 31 December 2015:

Debit Credit

Rs. in million

Sales - Manufactured goods 56,528

Sales - Imported goods 1,078

Scrap sales 16

Dividend income 12

Return on savings account 2

Sales tax - Imported goods 53

Sales tax - Manufactured goods 10,201

Sales discount 2,594

Raw material stock as on 1 January 2015 1,751

Work in process as on 1 January 2015 73

Finished goods (manufactured) as on 1 January 2015 1,210

Finished goods (imported) as on 1 January 2015 44

Purchases - Raw material 22,603

Purchases - Imported goods 658

Stores and spares consumed 180

Salaries, wages and benefits 2,367

Utilities 734

Depreciation and amortization 1,287

Stationery and office expenses 230

Repairs and maintenance 315

Advertisement and sales promotion 4,040

Outward freight and handling 1,279

Legal and professional charges 71

Auditor's remuneration 13

Donations 34

Workers Profit Participation Fund 257

Worker Welfare Fund 98

Loss on disposal of property, plant and equipment 10

Financial charges on short term borrowings 133

Exchange loss 22

Financial charges on lease 11

Additional information

(i) The position of inventories as at 31 December 2015 was as follows:

Rs. m

Raw material 2,125 Work in process 125

Finished goods (manufactured) 1,153

Finished goods (imported) 66

Page 28: CAF7-Financial Accounting and Reporting II_Questionbank

Financial accounting and reporting II

© Emile Woolf International 18 The Institute of Chartered Accountants of Pakistan

(ii) The basis of allocation of various expenses among cost of sales, distribution costs and administrative expenses are as follows:

Cost of sales

Distribution costs

Administrative expenses

% % %

Salaries, wages and benefits 55 30 15

Depreciation and amortization 70 20 10

Stationery and office expenses 25 40 35

Repairs and maintenance / Utilities 85 5 10

(iii) Salaries, wages and benefits include contributions to provident fund (defined contribution plan) and gratuity fund (defined benefit plan) amounting to Rs. 54 million and Rs. 44 million respectively.

(iv) Auditor’s remuneration includes taxation services and out-of-pocket expenses amounting to Rs. 4 million and Rs. 1 million respectively.

(v) Donations include Rs. 5 million given to Dates Cancer Foundation (DCF). One of the company’s directors, Mr. Peanut is a trustee of DCF.

(vi) The tax charge for the current year after making all related adjustments is estimated at Rs. 1,440 million. Taxable temporary differences of Rs. 3,120 originated in the year million, over the last year. The applicable income tax rate is 35%.

(vii) 274 million ordinary shares were outstanding as on 31 December 2015.

(viii) There is no other comprehensive income for the year.

Required

Prepare the statement of profit or loss for the year ended 31 December 2015 along with the relevant notes showing required disclosures as per the Companies Ordinance, 1984 and International Financial Reporting Standards. Comparatives are not required. (24)

Page 29: CAF7-Financial Accounting and Reporting II_Questionbank

Questions

© Emile Woolf International 19 The Institute of Chartered Accountants of Pakistan

CHAPTER 3 – IAS 7: STATEMENTS OF CASH FLOWS

3.1 KLEA

The statement of financial position and statement of profit or loss for Klea for the year to 31st March 2015 are provided below.

Statement of financial position as at 31st March 2015

2015 2014

Rs. in ‘000

Assets

Non-current assets

Intangible assets 300 200

Property, plant and equipment 3,450 1,600

Financial assets 400 200 –––––– –––––– 4,150 2,000 –––––– –––––– Current assets

Inventory 3,200 2,000

Trade receivables 2,400 2,000

Cash and cash equivalents 32 580 –––––– –––––– 5,632 4,580 –––––– –––––– Total assets 9,782 6,580 –––––– –––––– Equity and liabilities

Equity

Issued share capital 3,000 2,000

Share premium account 838 560

Retained earnings 910 354 –––––– –––––– Total equity 4,748 2,914 –––––– –––––– Revaluation surplus 1,000 - Non-current liabilities

Interest-bearing loans and liabilities 1,600 2,000 Current liabilities

Bank overdraft 414 -

Trade payables 1,600 1,266

Taxation 420 400 –––––– –––––– 2,434 1,666 –––––– –––––– Total liabilities 4,034 3,666 –––––– –––––– Total equity and liabilities 9,782 6,580

════ ════

Page 30: CAF7-Financial Accounting and Reporting II_Questionbank

Financial accounting and reporting II

© Emile Woolf International 20 The Institute of Chartered Accountants of Pakistan

Statement of profit or loss for the year ended 31st March 2015

Rs. in ‘000

Revenue 10,000

Other income 100

Change in inventory of finished goods and WIP 1,300

Raw materials and consumables used 4,000

Employee benefits costs 3,000

Depreciation and amortisation expense 800

Other expenses 1,724 –––––– Total expenses (9,524) ––––– 1,876

Finance costs (320)

Finance income 50 ––––– Profit before tax 1,606

Income tax expense (650) ––––– Profit for the year 956 ════

Additional information

(i) Non-current assets Rs. in ‘000

2015 2014

Cost Deprec’n Cost Deprec’n

Intangible assets 700 400 400 200

Property, plant and equipment 5,000 1,550 3,000 1,400

(ii) At 1 April 2014 land was revalued from Rs. 1million to Rs. 2 million.

(iii) During the year, plant and machinery costing Rs. 600,000 and depreciated by Rs. 500,000 was sold for Rs. 150,000.

(iv) The interest bearing loans relate to debentures which were issued at their nominal value. Rs. 400,000 of these debentures were redeemed at par during the year.

(v) Ordinary shares were issued for cash during the year.

(vi) Rs. 100,000 of current asset investments held as cash equivalents were sold during the year for Rs. 94,000.

(vii) Dividends paid in the year were Rs. 200,000 relating to the 2014 proposed dividend and a Rs. 200,000 interim dividend for 2015.

Required

Prepare a statement of cash flows for Klea for the year ended 31 March 2015 in accordance with IAS 7 using the indirect method. (25)

Page 31: CAF7-Financial Accounting and Reporting II_Questionbank

Questions

© Emile Woolf International 21 The Institute of Chartered Accountants of Pakistan

3.2 STANDARD INC

The summarised statements of financial position of Standard Inc at 31 December 2014 and 2015 are as follows.

2015 2014 Rs. in ‘000 Rs. in ‘000

Issued share capital 150,000 100,000 Share premium 35,000 15,000 Retained earnings 41,000 14,000 Long-term loans 30,000 70,000 Payables 48,000 34,000 Bank overdraft – 14,000 Tax payable 33,000 21,500 Proposed dividends 15,000 7,500 Depreciation Plant and machinery 54,000 45,000 Fixtures and fittings 15,000 13,000 ———— ———— 421,000 334,000 ———— ————

Freehold property at cost 130,000 110,000 Plant and machinery at cost 151,000 120,000 Fixtures and fittings at cost 29,000 24,000 Inventories 51,000 37,000 Trade receivables 44,000 42,800 Long-term investments 4,600 – Cash at bank 11,400 200 ———— ———— 421,000 334,000 ———— ————

The following information is relevant:

(a) There had been no disposal of freehold property in the year.

(b) A machine tool which had cost Rs. 8,000,000 (in respect of which Rs. 6,000,000 depreciation had been provided) was sold for Rs. 3,000,000, and fixtures which had cost Rs. 5,000,000 (in respect of which depreciation of Rs. 2,000,000 had been provided) were sold for Rs. 1,00,0000. Profits and losses on those transactions had been dealt with through the statement of profit or loss.

(c) The statement of profit or loss charge in respect of tax was Rs. 22,000,000.

(d) The premium paid on redemption of the long-term loan was Rs. 2,000,000, which has been written off to the statement of profit or loss.

(e) The proposed dividend for 2014 had been paid during the year.

(f) Interest received during the year was Rs. 450,000. Interest charged in the statement of profit or loss for the year was Rs. 6,400,000. Accrued interest of Rs. 440,000 is included in payables at 31 December 2014 (nil at 31 December 2015).

(g) The government stock is a long term investment.

Required

Prepare a cash flow statement for the year ended 31 December 2015, together with notes as required by IAS 7. (20)

Page 32: CAF7-Financial Accounting and Reporting II_Questionbank

Financial accounting and reporting II

© Emile Woolf International 22 The Institute of Chartered Accountants of Pakistan

3.3 FALLEN

Fallen has prepared the following rough draft accounts for the year ended 31 December 2015.

Statement of profit or loss

Rs. in ‘000

Revenue 11,563 Cost of sales (5,502) ——— Gross profit 6,061 Distribution costs (402) Administration expenses (882) Interest payable (152) ——— Operating profit before tax 4,625 Taxation (35%) including deferred tax (1,531) ——— Profit after tax 3,094 Dividends (700) ——— Retained profit 2,394 ———

Statements of financial position

31 December 2015 2014 Rs. in ‘000 Rs. in ‘000

Leasehold premises (net) 6,600 5,700 Plant, machinery and equipment (net) 5,040 3,780 Investments at cost 2,406 2,208 Inventories 2,880 1,986 Receivables 2,586 1,992 Bank – 576 ——— ——— 19,512 16,242 ——— ———

Page 33: CAF7-Financial Accounting and Reporting II_Questionbank

Questions

© Emile Woolf International 23 The Institute of Chartered Accountants of Pakistan

31 December 2015 2014 Rs. in ‘000 Rs. in ‘000

Share capital 2,280 1,800 Share premium 2,112 1,800 Profit and loss account 9,108 6,714 Deferred taxation 202 138 Long-term loan (10%) 1,240 1,800 Provision for deferred repairs 1,202 1,016 Payables 1,026 702 Overdraft 222 – Taxation Corporation tax 1,730 2,038 Proposed dividends 390 234 ——— ——— 19,512 16,242 ——— ———

The following data is relevant.

(1) The 10% long-term loan were redeemed at par.

(2) Plant and equipment with a written down value of Rs. 276,000 was sold for Rs. 168,000. New plant was purchased for Rs. 2,500,000.

(3) Leasehold premises costing Rs. 1,300,000 were acquired during the year.

(4) The investments are highly liquid securities held for the short term.

Required

Prepare the cash flow statement and supporting notes in accordance with IAS 7 for Fallen Inc for 2015. (20)

Page 34: CAF7-Financial Accounting and Reporting II_Questionbank

Financial accounting and reporting II

© Emile Woolf International 24 The Institute of Chartered Accountants of Pakistan

3.4 BIN QASIM MOTORS LIMITED

The summarised financial statements of Bin Qasim Motors Limited for the year to 30 September 2015, together with a comparative balance sheet, are:

Statement of profit or loss Rs. 000

Sales revenue 7,482

Cost of sales (4,284)

Gross profit 3,198

Operating expenses (1,479)

Interest payable (260)

Investment income 120

Profit before tax 1,579

Income tax (520)

Profit for the period 1,059

Statement of financial position as at 30 September

2015 2014

Rs. in ‘000 Rs. in ‘000

Assets

Non-current assets

Property, plant and equipment 2,344 1,908

Investment 690 nil

3,034 1,908

Current assets

Inventory 1,046 785

Trade accounts receivable 935 824

Short term treasury bills 120 50

Bank nil 122

2,101 1,781

Total assets 5,135 3,689

Total equity and liabilities

Equity:

Share capital 1,400 1,000

Reserves:

Share premium 460 60

Retained earnings

At beginning of the year 192 147

Net profit for period 1,059 65

Dividends (180) (20)

At end of the year 1,071 192

2,931 1,252

Page 35: CAF7-Financial Accounting and Reporting II_Questionbank

Questions

© Emile Woolf International 25 The Institute of Chartered Accountants of Pakistan

Revaluation surplus 90 40

Non-current liabilities

Deferred tax 439 400

Deferred income 275 200

10% Convertible loan stock nil 400

714 1,000

Current liabilities

Trade accounts payable 644 760

Accrued interest 40 25

Provision for negligence claim nil 120

Provision for income tax 480 367

Deferred income 100 125

Overdraft 136 nil

1,400 1,397

Total equity and liabilities 5,135 3,689

The following information is relevant

(i) Non-current assets

Property, plant and equipment is analysed as follows: Rs in ‘000

30 September 2015 30 September 2014

Cost/

Depreciation NBV

Cost/

Depreciation NBV Valuation Valuation

Land and buildings 2,000 760 1,240 1,800 680 1,120

Plant 1,568 464 1,104 1,220 432 788

3,568 1,224 2,344 3,020 1,112 1,908

On 1 October 2014 Bin Qasim Motors Limited recorded an increase in the value of its land of Rs. 150,000.

During the year an item of plant that had cost Rs. 500,000 and had accumulated depreciation of Rs. 244,000 was sold at a loss (included in cost of sales) of Rs. 86,000 on its carrying value.

(ii) Deferred income

Bin Qasim Motors Limited sells servicing contracts on certain types of machinery. Payments are received in advance for a service which Bin Qasim Motors Limited must provide over a number of following years. Income that relates to these contracts is deferred and recognised in P&L as the period of service passes.

A credit of Rs. 125,000 for the current year’s recognition of deferred income has been included revenue in this period.

(iii) Share capital and loan stocks

The increase in the share capital during the year was due to the following events:

(1) On 1 January 2015 there was a bonus issue (out of the revaluation surplus) of one bonus share for every 10 shares held.

Page 36: CAF7-Financial Accounting and Reporting II_Questionbank

Financial accounting and reporting II

© Emile Woolf International 26 The Institute of Chartered Accountants of Pakistan

(2) On 1 April 2015 the 10% convertible loan stock holders exercised their right to convert to ordinary shares. The terms of conversion were 25 ordinary shares of Rs. 1 each for each Rs. 100 of 10% convertible loan stock.

(3) The remaining increase in the ordinary shares was due to a stock market placement of shares for cash on 12 August 2015.

(iv) Provision for negligence claim

In June 2015 Bin Qasim Motors Limited made an out of court settlement of a negligence claim brought about by a former employee. The dispute had been in progress for two years and Bin Qasim Motors Limited had made provisions for the potential liability in each of the two previous years. The unprovided amount of the claim at the time of settlement was Rs. 30,000 and this was charged to operating expenses.

Required

Prepare a statement of cash flows for Bin Qasim Motors Limited for the year to 30 September 2015 in accordance with IAS 7 Statement of Cash Flows. (25)

Page 37: CAF7-Financial Accounting and Reporting II_Questionbank

Questions

© Emile Woolf International 27 The Institute of Chartered Accountants of Pakistan

3.5 ITTEHAD MANUFACTURING LTD

The financial statements of Ittehad Manufacturing Ltd for the year to 30 September 2015, together with the comparative statement of financial position (balance sheet) for the year to 30 September 2014 are shown below:

Rs. in million

Sales revenue 3,820

Cost of sales (note 1) (2,620)

Gross profit for period 1,200

Operating expenses (note 1) (280)

920

Interest – Loan note (30)

Profit before tax 890

Taxation (270)

Net profit for the period 620

Statement of financial position as at 30 September:

2015 2016

Rs. in million Rs. in million

Non-current assets

Property, plant and equipment 1,890 1,830

Intangible assets (note 2) 670 300

2,560 2,130

Current assets

Inventory 1,420 940

Accounts receivable 990 680

Cash 70 nil

2,480 1,620

Total assets 5,040 3,750

Equity and liabilities

Ordinary shares of Rs. 1 each 750 500

Reserves

Share premium 350 100

Revaluation 190 nil

Retained earnings 1,860 1,600

3,150 2,200

Non-current liabilities (note 3) 610 240

Current liabilities (note 4) 1,280 1,310

Total equity and liabilities 5,040 3,750

Extract from statement of changes in equity

2015 2014

Rs. in million

Retained earnings – brought forward 1,600 1,000

Profit for the year 620 800

Dividends (320) (200)

Bonus issue (50)

Transfer from revaluation surplus 10 –

Retained earnings – carried forward 1,860 1,600

Page 38: CAF7-Financial Accounting and Reporting II_Questionbank

Financial accounting and reporting II

© Emile Woolf International 28 The Institute of Chartered Accountants of Pakistan

Notes to the financial statements:

(1) Cost of sales includes depreciation of property, plant and equipment of Rs. 320 million and a loss on the sale of plant of Rs. 50 million.

2015 2014

Rs. in million

(2) Intangible non-current assets:

Deferred development expenditure 470 100

Goodwill 200 200

670 300

(3) Non-current liabilities:

10% loan note 300 100

Deferred tax 310 140

610 240

(4) Current liabilities:

Accounts payable 875 730

Bank overdraft nil 115

Accrued loan interest 15 5

Deferred income 260 300

Taxation 130 160

1,280 1,310

The following additional information is relevant:

(i) Intangible non-current assets:

The company successfully completed the development of a new product during the current year, capitalising a further Rs. 500 million before amortisation charges for the period.

(ii) Property, plant and equipment/revaluation surplus:

The company revalued its buildings by Rs. 200 million on 1 October 2014. The surplus was credited to a revaluation surplus.

New plant was acquired during the year at a cost of Rs. 250.

Rs. 10 million has been transferred from the revaluation surplus to retained earnings as a year-end adjustment in respect of the additional depreciation created by the revaluation.

The remaining movement on property, plant and equipment was due to the disposal of obsolete plant.

(iii) Share issues:

On 1 October 2014 a bonus issue of 1 new share for every 10 held was made from retained earnings. Ittehad Manufacturing Ltd made a further issue of ordinary shares for cash during the year.

Required

(a) A statement of cash flows for Ittehad Manufacturing Ltd for the year to 30 September 2015 prepared in accordance with IAS 7 Statement of Cash Flows. (20)

(b) Comment briefly on the financial position of Ittehad Manufacturing Ltd as portrayed by the information in your statement of cash flows. (5)

(25)

Page 39: CAF7-Financial Accounting and Reporting II_Questionbank

Questions

© Emile Woolf International 29 The Institute of Chartered Accountants of Pakistan

3.6 WASEEM INDUSTRIES LIMITED

The following statements of financial position relate to Waseem Industries Limited for the years ended December 31:

2015 2014 Rs. in

million

Rs. in million

ASSETS Non-current assets Fixed assets Property, plant and equipment 242 182 Capital work-in-progress 20 18

262 200 Long term investments 75 100 Long term deposits 13 13

Total non-current assets 350 313 Current assets

Stocks-in-trade 55 48 Trade debts 51 38 Advances, prepayments and other receivables 37 40 Cash and bank balances 11 20

Total current assets 154 146

TOTAL ASSETS 504 459

EQUITY AND LIABILITIES Shareholders' equity

Share capital 150 125 Share premium 55 80 Unappropriated profit 85 50

290 255 Non-current liabilities

Long term finances - Secured 94 118 Deferred liability - Gratuity (unfunded) 16 12

110 130 Current liabilities

Current portion of long term finances 25 22 Short term finances 13 6 Trade and other payables 66 46

104 74

TOTAL EQUITY AND LIABILITIES 504 459

Page 40: CAF7-Financial Accounting and Reporting II_Questionbank

Financial accounting and reporting II

© Emile Woolf International 30 The Institute of Chartered Accountants of Pakistan

Other relevant information is as follows:

(i) An interim bonus issue of one for five ordinary shares was made during the year out of share premium. The company also approved final cash dividend of 10% (2014: 8%), in its annual general meeting.

(ii) During the year, the company provided Rs. 17 million (2014: Rs. 13 million) on account of depreciation. The details relating to disposal of property, plant and equipment are as follows:

Carrying amount Sale proceeds

Rs. m Rs. m

Plant and machinery 20 22

Vehicles 3 4

(iii) Advances, prepayments and other receivables include advance tax of Rs. 10 million (2014: Rs. 7 million).

(iv) In 2015, the company paid Rs. 6 million on account of gratuity.

(v) Accrued mark-up on long term finances amounting to Rs. 7 million (2014: Rs. 9 million) is included in trade and other payables. Financial charges included in the profit and loss account are Rs. 16 million (2014 : Rs. 14 million).

(vi) Income tax expense for the year 2015 amounted to Rs. 19 million (2014: Rs. 13 million).

Required

Prepare a cash flow statement in accordance with the requirements of IAS 7 Cash Flow Statement” using the indirect method. (20)

Page 41: CAF7-Financial Accounting and Reporting II_Questionbank

Questions

© Emile Woolf International 31 The Institute of Chartered Accountants of Pakistan

3.7 JALIB INDUSTRIES LIMITED

Jalib Industries Limited is a listed company. The relevant information contained in the financial statements for the year ended December 31, 2015 is as follows:

Statement of Financial Position

2015 2014

Rupees in million

Non-current assets

Property, plant and equipment 129.40 100.60

Capital work in progress 22.50 37.00

151.90 137.60

Current assets

Stock in trade 531.80 451.00 Trade debts 28.50 24.70 Advances and other receivables 37.40 42.00 Cash and bank 12.00 3.00

609.70 520.70

761.60 658.30

Equity Issued, subscribed and paid-up capital 396.00 300.00

Share premium 45.00 12.00

Unappropriated profit 142.60 163.00

583.60 475.00

Non-current liabilities Deferred liabilities 40.80 27.50 Long term loans 80.00 100.00

120.80 127.50

Current liabilities

Current portion of long term loans 18.00 20.00 Creditors, accrued and other liabilities 36.20 34.40 Dividend payable 3.00 1.40

57.20 55.80

761.60 658.30

Statement of profit or loss 2015

Rupees in million

Sales 2,535.00

Cost of goods sold (1,774.50)

Gross profit 760.50

Operating expenses (554.00)

Financial charges (10.50)

Loss on sale of fixed assets (4.60)

(569.10)

Profit before tax 191.40

Tax expense - Current (104.60)

- Deferred (2.20)

(106.80)

Profit after tax 84.60

Page 42: CAF7-Financial Accounting and Reporting II_Questionbank

Financial accounting and reporting II

© Emile Woolf International 32 The Institute of Chartered Accountants of Pakistan

The following supporting information is available:

(i) During the year, an amount of Rs. 42 million was transferred from capital work in progress to property, plant and equipment.

(ii) The company sold property, plant and equipment having book value of Rs. 15 million for Rs. 10.4 million.

(iii) Depreciation for the year amounted to Rs. 27.7 million.

(iv) Trade debts written off during the year amounted to Rs. 1 million. It is the policy of the company to maintain the provision for doubtful debts at 5% of trade debts.

(v) Advances and other receivables include advance tax of Rs. 3.6 million (2014: Rs. 2.2 million).

(vi) Deferred liabilities include deferred tax and provision for gratuity. There was no deferred tax liability at the beginning of the year. Provision for gratuity made during the year amounted to Rs. 15.5 million.

(vii) Creditors, accrued and other liabilities include accrued financial charges amounting to Rs. 5 million (2014: Rs. 6 million).

(viii) On January 15, 2016, the company declared final dividend for the year ended December 31, 2015 comprising 7.5% (2014: 25%) cash dividend and 12.5% (2014:10%) bonus shares, for its ordinary shareholders.

Required

Prepare a statement of cash flow for the year ended December 31, 2015 in accordance with the requirements of International Accounting Standards. Show all necessary workings. (23)

3.8 APOLLO INDUSTRY LIMITED

Following are the relevant extracts from the financial statements of Apollo Industry Limited, a listed company, for the year ended December 31, 2015.

Statement of financial position as at December 31, 2015

2015 2014

Rs. 000 Rs. 000

Issued, subscribed and paid up capital 25,000 20,000 Unappropriated profit 20,900 22,000

45,900 42,000

Surplus on revaluation of property, plant & equipment 7,000 8,000

Non-current liabilities

Staff gratuity Deferred tax liability- net

1,400 590

1,190 -

1,990 1,190 Trade and other payables 4,200 6,250

59,090 57,440

Page 43: CAF7-Financial Accounting and Reporting II_Questionbank

Questions

© Emile Woolf International 33 The Institute of Chartered Accountants of Pakistan

2015 2014

Rs. 000 Rs. 000 Property, plant and equipment

35,000

25,500

Capital work in progress 5,500 10,000 Intangible assets 1,100 1,140

41,600 36,640 Deferred tax asset- net - 350 Long term deposits and prepayments 400 300

Current Assets

42,000 37,290

Tax refundable 950 800 Other current assets 15,700 12,125 Cash and bank balances 440 7,225

17,090 20,150

59,090 57,440

Statement of comprehensiveiIncome for the year ended December 31, 2015

2015

Rs. 000

Sales 146,700

Cost of sales (127,500)

Gross profit 19,200

Operating expenses (15,000)

Financial charges (500)

Other income 2,800

(12,700)

Profit before tax 6,500

Tax expense - current (4,660)

- deferred (940)

Tax for the year (5,600)

Profit after tax 900

Other relevant information is as under:

(i) During the year, the company has issued 10% bonus shares.

(ii) Depreciation and amortization for the year amounted to Rs. 7 million.

(iii) WDV of assets disposed off during the year amounted to Rs. 1.2 million. (The assets had not been revalued)

(iv) Other income includes interest earned on short term placements, amounting to Rs. 1 million. The remaining amount represents gain on disposal of property, plant and equipment.

(v) Gratuity of Rs. 0.3 million was paid to outgoing employees.

(vi) Intangible assets worth Rs. 50 thousand were acquired during the year.

Required

Prepare the Statement of Cash Flows for the year ended December 31, 2015 in accordance with the requirements of IAS - 7 (Statement of Cash Flows) using ‘indirect method’. (22)

Page 44: CAF7-Financial Accounting and Reporting II_Questionbank

Financial accounting and reporting II

© Emile Woolf International 34 The Institute of Chartered Accountants of Pakistan

3.9 MARVEL ENGINEERING LIMITED

Following are the extracts from the draft financial statements of Marvel Engineering Limited (MEL), a listed company, for the year ended 30 June 2015:

Statement of Financial Position

Rs. in million

2015 2014 2015 2014 Non current assets Share capital and

reserves

Property, plant and equipment

633 410 Share capital (Rs. 10 each)

494 440

Long term investments

130 100 Share premium 8 -

763 510 Retained earnings 133 110 Current assets 635 550

Stock-in-trade 97 68 Non current liabilities

Trade debts 133 57 Long term loans 330 110 Other receivables 100 120 Gratuity payable 55 50 Cash at bank 31 39 Deferred taxation 15 21

361 284 400 181 Current liabilities

Trade and other payables

73 56

Tax payable - net 12 5 Dividend payable 4 2

89 63

1,124 794 1,124 794

Statement of profit or loss 2015 Rs. in million

Revenue 654 Cost of sales (458)

Gross profit 196

Operating expenses (68) Financial charges (75) Other income 35

(108)

Profit before tax 88 Income tax expense (21)

Profit after tax 67

Additional information:

(i) During the year, the company recognised a provision for impairment in respect of one of its plant, amounting to Rs. 11 million. Total depreciation for the year amounted to Rs. 50 million.

(ii) It is the policy of the company to maintain a provision for doubtful debts at 5% of trade debts. During the year, trade debts amounting to Rs. 6 million (2014: Rs. 2 million) were written off.

(iii) Trade and other payables include accrued financial charges amounting to Rs. 7 million (2014: Rs. 3 million).

Page 45: CAF7-Financial Accounting and Reporting II_Questionbank

Questions

© Emile Woolf International 35 The Institute of Chartered Accountants of Pakistan

(iv) On 15 July 2015, MEL’s board of directors proposed a final dividend of 10% for the year ended 30 June 2015 (2014: 5% cash dividend and 5% bonus declared on 20 July 2014).

(v) Other income comprises of the following:

Rs. m

Dividend income 30 Gain on sale of vehicles (carrying value of Rs. 5 million) 2 Gain on sale of investments (carrying value of Rs. 10 million) 3

35

(vi) Gratuity paid during the year amounted to Rs. 6 million.

Required

Prepare the statement of cash flows for Marvel Engineering Limited for the year ended 30 June 2015. (24)

Page 46: CAF7-Financial Accounting and Reporting II_Questionbank

Financial accounting and reporting II

© Emile Woolf International 36 The Institute of Chartered Accountants of Pakistan

CHAPTER 4 – CONSOLIDATED ACCOUNTS: STATEMENTS OF FINANCIAL

POSITION – BASIC APPROACH

4.1 HALL

Statements of financial position at 31 December 2015

Hall Stand Rs. 000 Rs. 000 Assets Non-current assets Property, plant and equipment 35,000 20,000 Investment in Stand 12,000 – Current assets 16,000 14,000 ——— ——— 63,000 34,000 ——— ——— Equity and liabilities Capital and reserves Share capital 10,000 4,000 Retained earnings 13,000 12,000 ——— ——— 23,000 16,000 Non-current liabilities 8% Debenture loans 20,000 9,000 Current liabilities 20,000 9,000 ——— ——— 63,000 34,000 ——— ———

On 1 January 2013 Hall acquired 75% of Stand for Rs. 12,000,000. At that date the balance on Stand’s retained earnings was Rs. 8,000,000.

Required

Prepare the consolidated statement of financial position of Hall as at 31 December 2015. (6)

Page 47: CAF7-Financial Accounting and Reporting II_Questionbank

Questions

© Emile Woolf International 37 The Institute of Chartered Accountants of Pakistan

4.2 HASSLE

Statements of financial position at 31 December 2015 Hassle Strife Rs. Rs. Investment in Strife 60,000 – Sundry assets 247,500 226,600 ———– —–—— 307,500 226,600 ———– ——–— Share capital 120,000 50,000 Retained earnings 87,500 70,000 Liabilities 100,000 106,600 ———– ——–— 307,500 226,600 ———– ——–—

Hassle bought 80% of Strife when the balance on Strife’s retained profit was Rs. 50,000.

Required

Prepare the consolidated statement of financial position at 31 December 2015. (8)

4.3 HYMN

The following are the summarised statements of financial position of a group of companies as at 31 December 2015.

Hymn Psalm Rs. Rs.

Assets Non-current assets Property, plant and equipment 105,000 65,000 Investment 85,000 Current assets 220,000 55,000 ———– ———–

410,000 120,000 ———– ———– Equity and liabilities Equity Share capital 100,000 50,000 Retained earnings 155,000 49,000

———– ———– 255,000 99,000

Current liabilities 155,000 21,000 ———– ———– 410,000 120,000

———– ———–

Hymn purchased 80% of Psalm’s shares on 1 January 2015 when there was a credit balance on that company’s retained earnings of Rs. 20,000.

Required

Prepare the Hymn group consolidated statement of financial position as at 31 December 2015. (6)

Page 48: CAF7-Financial Accounting and Reporting II_Questionbank

Financial accounting and reporting II

© Emile Woolf International 38 The Institute of Chartered Accountants of Pakistan

4.4 HANG

On 31 December 2012, Hang acquired 60% of Swing for Rs. 140,000. At that date Swing had a retained earnings balance of Rs. 50,000 and a share premium account balance of Rs. 49,000.

The following statements of financial position have been prepared as at 31 December 2015.

Hang Swing Rs. Rs.

Assets Non-current assets Property, plant and equipment 240,000 180,000 Investment in Swing 140,000 Current assets 250,000 196,000

———– ———–

630,000 376,000

———– ———–

Equity and liabilities Equity Share capital 200,000 90,000 Share premium 25,000 49,000 Retained earnings 180,000 80,000

———– ———–

405,000 219,000 Current liabilities 225,000 157,000

———– ———–

630,000 376,000

———– ———–

Required

Prepare the consolidated statement of financial position of Hang and its subsidiary as at 31 December 2015. (6)

4.5 HASH

Statements of financial position at 31 December 2015 Hash Stash Rs. 000 Rs. 000 Investment in Stash (80%) 100,000 – Sundry assets 207,500 226,600 ———– —–—— 307,500 226,600 ———– ——–— Share capital 120,000 50,000 Retained earnings 87,500 70,000 Liabilities 100,000 106,600 ———– ——–— 307,500 226,600 ———– ——–— Hash purchased the shares in Stash on 30th September 2015.

Stash’s retained profit for the year ended 31st December 2015 was Rs. 24,000,000.

Required

Prepare the consolidated statement of financial position at 31 December 2015. (8)

Page 49: CAF7-Financial Accounting and Reporting II_Questionbank

Questions

© Emile Woolf International 39 The Institute of Chartered Accountants of Pakistan

CHAPTER 5 – CONSOLIDATED ACCOUNTS: STATEMENTS OF FINANCIAL

POSITION – COMPLICATIONS

5.1 HAIL

The following are the draft statements of financial position of Hail and its subsidiary Snow as at 31 December 2015.

Hash Stash Rs. 000 Rs. 000

Assets Non-current assets Property, plant and equipment 161,000 85,000 Investments 68,000 Current assets Cash 7,700 25,200 Trade receivables 92,500 45,800 Snow current account 15,000 - Inventory 56,200 36,200

———– ——–— 400,400 192,200

———– ——–— Equity and liabilities Shareholders’ equity Share capital 100,000 50,000 Retained earnings 185,400 41,200 Share premium - 5,000 Capital reserve - 20,000

———– ——–— 285,400 116,200 Current liabilities 115,000 68,000 Hail current account - 8,000

———– ——–— 400,400 192,200

———– ——–— Notes

(1) Snow has 50,000 shares in issues. Hail acquired 45,000 of these on 1 January 2012 for a cost of Rs. 65,000,000 when the balances on Snow’s reserves were

Rs. 000 Share premium account 5,000 Capital reserve – Retained earnings 10,000

(2) Hail declared a dividend of Rs. 3,000,000 before the year end and Snow declared one of Rs. 2,000,000. These transactions have not been accounted for.

(3) The current account difference is due to cash in transit.

Required

Prepare the consolidated statement of financial position as at 31 December 2015 of Hail. (12)

Page 50: CAF7-Financial Accounting and Reporting II_Questionbank

Financial accounting and reporting II

© Emile Woolf International 40 The Institute of Chartered Accountants of Pakistan

5.2 HAIRY

The summarised statements of financial position of Hairy and Spider as at 31 December 2015 were as follows.

Hairy Spider Rs. 000 Rs. 000

Assets Non-current assets Property, plant and equipment 120,000 60,000 Investments 55,000 –

Current assets Cash 11,000 4,000 Investments – 3,000 Trade receivables 72,600 19,100 Current account – Hairy – 3,200 Inventory 17,000 11,000

———– ———–

275,600 100,300 ———– ———–

Equity and liabilities Share capital 100,000 60,000 Share premium 20,000 – Capital reserve 23,000 16,000 Retained earnings 91,900 7,300 Trade payables 38,000 17,000 Current account – Spider 2,700 –

———– ———–

275,600 100,300 ———– ———–

The following information is relevant.

(1) On 31 December 2012, Hairy acquired 48,000 shares in Spider for Rs. 55,000,000 cash. Spider has 60,000 shares in total.

(2) The inventory of Hairy includes Rs. 4,000,000 goods from Spider invoiced to Hairy at cost plus 25%.

(3) The difference on the current account balances is due to cash in transit.

(4) The balance on Spider’s retained earnings was Rs. 2,300,000 at the date of acquisition. There has been no movement in the balance on Spider’s capital reserve since the date of acquisition.

Required

Prepare the consolidated statement of financial position of Hairy and its subsidiary Spider as at 31 December 2015. (12)

Page 51: CAF7-Financial Accounting and Reporting II_Questionbank

Questions

© Emile Woolf International 41 The Institute of Chartered Accountants of Pakistan

5.3 HARD

On 31 December 2011, Hard acquired 60% of the ordinary share capital of Soft for Rs. 110 million. At that date Soft had a retained earnings balance of Rs. 50 million and a share premium account balance of Rs. 10 million.

The following statements of financial position have been prepared as at 31 December 2015.

Hard Soft Rs. 000 Rs. 000

Assets Non-current assets Property, plant and equipment 225,000 175,000 Investments in Soft 110,000

Current assets 271,000 157,000

———– ———–

606,000 332,000

———– ———–

Equity and liabilities Capital and reserves Share capital 100,000 100,000 Share premium 15,000 10,000 Retained earnings 260,000 80,000

———– ———–

375,000 190,000 Current liabilities 231,000 142,000

———– ———–

606,000 332,000

———– ———–

During the year to 31 December 2015 Hard sold a tangible asset to Soft for Rs. 50 million. The asset was originally purchased in the year to 31 December 2012 at a cost of Rs. 100 million and had a useful economic life of five years.

Soft’s depreciation policy is 25% per annum based on cost. Both companies charge a full year’s depreciation in the year of acquisition and none in the year of disposal.

Required

Prepare the consolidated statement of financial position of Hard and its subsidiary as at 31 December 2015. (12)

Page 52: CAF7-Financial Accounting and Reporting II_Questionbank

Financial accounting and reporting II

© Emile Woolf International 42 The Institute of Chartered Accountants of Pakistan

5.4 HALE

On 1 July 2012 Hale acquired 128,000 of Sowen’s 160,000 shares. The following statements of financial position have been prepared as at 31 December 2015.

Hale Sowen Rs. 000 Rs. 000

Property, plant and equipment 152,000 129,600 Investment in Sowen 203,000 – Inventory at cost 112,000 74,400 Receivables 104,000 84,000 Bank balance 41,000 8,000

———– ———– 612,000 296,000 ═════ ═════ Hale Sowen Rs. 000 Rs. 000

Share capital 100,000 160,000 Retained earnings 460,000 112,000 Payables 52,000 24,000

———– ———– 612,000 296,000 ═════ ═════

The following information is available.

(1) At 1 July 2012 Sowen had a debit balance of Rs. 11 million on retained earnings.

(2) Property, plant and equipment of Sowen included land at a cost of Rs. 72 million. This land had a fair value of Rs. 100,000 at the date of acquisition.

(3) The inventory of Sowen includes goods purchased from Hale for Rs. 16 million. Hale invoiced those goods at cost plus 25%.

Required

Prepare the consolidated statement of financial position of Hale as at 31 December 2015. (12)

5.5 HELLO

On 1 January 2012, Hello acquired 60% of the ordinary share capital of Solong for Rs. 110,000. At that date Solong had a retained earnings balance of Rs. 60,000.

The following statements of financial position have been prepared as at 31 December 2015.

Hello Solong Rs. Rs.

Assets Non-current assets

Property, plant and equipment 225,000 175,000 Investments in Solong 110,000

Current assets 271,000 157,000 ———– ———–

606,000 332,000 ———– ———–

Page 53: CAF7-Financial Accounting and Reporting II_Questionbank

Questions

© Emile Woolf International 43 The Institute of Chartered Accountants of Pakistan

Equity and liabilities Capital and reserves

Share capital 100,000 100,000 Retained earnings 275,000 90,000

———– ———–

375,000 190,000 Current liabilities 231,000 142,000

———– ———–

606,000 332,000 ———– ———–

The fair value of Solong’s net assets at the date of acquisition was determined to be Rs. 170,000.

The difference between the book value and the fair value of the new assets at the date of acquisition was due to an item of plant which had a useful life of 10 years from the date of acquisition.

Required

Prepare the consolidated statement of financial position of Hello and its subsidiary as at 31 December 2015. (12)

5.6 HASAN LIMITED

On 1 April 2014, Hasan Limited acquired 90% of the equity shares in Shakeel Limited. On the same day Hasan Limited accepted a 10% loan note from Shakeel Limited for Rs. 200,000 which was repayable at Rs. 40,000 per annum (on 31 March each year) over the next five years. Shakeel Limited’s retained earnings at the date of acquisition were Rs. 2,200,000.

Statements of financial position as at 31 March 2015

Hasan Limited

Shakeel Limited

Rs. 000 Rs. 000

Non-current assets

Property, plant and equipment 2,120 1,990

Intangible – software – 1,800

Investments – equity in Shakeel Limited 4,110 –

Investments – 10% loan note Shakeel Limited

200 –

Investments – others 65 210

6,495 4,000

Current assets

Inventories 719 560

Trade receivables 524 328

Shakeel Limited current account 75 –

Cash 20

1,338 888

Total assets 7,833 4,888

Equity and liabilities:

Capital and reserves

Equity shares of Rs. 1 each 2,000 1,500

Share premium 2,000 500

Page 54: CAF7-Financial Accounting and Reporting II_Questionbank

Financial accounting and reporting II

© Emile Woolf International 44 The Institute of Chartered Accountants of Pakistan

Retained earnings 2,900 1,955

6,900 3,955

Non-current liabilities

10% Loan note from Hasan Limited – 160

Government grant 230 40

230 200

Current liabilities

Trade payables 475 472

Hasan Limited current account – 60

Income taxes payable 228 174

Operating overdraft – 27

703 733

Total equity and liabilities 7,833 4,888

The following information is relevant:

(i) Included in Shakeel Limited’s property at the date of acquisition was a leasehold property recorded at its depreciated historical cost of Rs. 400,000. The leasehold had been sub-let for its remaining life of only four years at an annual rental of Rs. 80,000 payable in advance on 1 April each year. The directors of Hasan Limited are of the opinion that the fair value of this leasehold is best reflected by the present value of its future cash flows. An appropriate cost of capital for the group is 10% per annum.

The present value of a Rs. 1 annuity received at the end of each year where interest rates are 10% can be taken as:

3 year annuity Rs. 2.50

4 year annuity Rs. 3.20

(ii) The software of Shakeel Limited represents the depreciated cost of the development of an integrated business accounting package. It was completed at a capitalised cost of Rs. 2,400,000 and went on sale on 1 April 2013. Shakeel Limited’s directors are depreciating the software on a straight-line basis over an eight-year life (i.e. Rs. 300,000 per annum). However, the directors of Hasan Limited are of the opinion that a five-year life would be more appropriate as sales of business software rarely exceed this period.

(iii) The inventory of Hasan Limited on 31 March 2015 contains goods at a transfer price of Rs. 25,000 that were supplied by Shakeel Limited who had marked them up with a profit of 25% on cost. Unrealised profits are adjusted for against the profit of the company that made them.

(iv) On 31 March 2015 Shakeel Limited remitted to Hasan Limited a cash payment of Rs. 55,000. This was not received by Hasan Limited until early April. It was made up of an annual repayment of the 10% loan note of Rs. 40,000 (the interest had already been paid) and Rs. 15,000 of the current account balance.

(v) The accounting policy of Hasan Limited for non-controlling interests (NCI) in a subsidiary is to value NCI at a proportionate share of the net assets.

(v) An impairment test at 31 March 2015 on the consolidated goodwill concluded that it should be written down by Rs. 120,000. No other assets were impaired.

Required:

Prepare the consolidated statement of financial position of Hasan Limited as at 31 March 2015.

(Total: 25 marks)

Page 55: CAF7-Financial Accounting and Reporting II_Questionbank

Questions

© Emile Woolf International 45 The Institute of Chartered Accountants of Pakistan

CHAPTER 6 – CONSOLIDATED ACCOUNTS: STATEMENTS OF

COMPREHENSIVE INCOME

6.1 HARRY

The following are the statements of profit or loss for the year ended 31 December 2015 of Harry and its subsidiary Sally.

Harry Sally Rs. 000 Rs. 000

Revenue 1,120 390 Cost of sales (610) (220)

Gross profit 510 170 Distribution costs (50) (40) Administration costs (55) (45)

Operating profit 405 85 Investment income 20 4 Finance costs (18) (4)

Profit before tax 407 85 Income tax expense (140) (25)

Profit for the year 267 60

Rs. 000 Rs. 000

Retained profit brought forward 100 45 Profit for year 267 60 Dividends paid and proposed (50) (20)

Retained profit carried forward 317 85

The following information is relevant.

(1) Harry acquired 75% of Sally six years ago when Sally’s retained earnings were Rs. 9,000.

(2) Harry made sales to Sally totalling Rs. 100,000 in the year. At the year end the statement of financial position of Sally included inventory purchased from Harry. Harry had taken a profit of Rs. 3,000 on this inventory.

(3) Harry’s investment income includes Rs. 15,000 being its share of Sally’s dividends.

Required

Prepare a consolidated statement of profit or loss and a working showing the movement on consolidated retained profit for the year ended 31 December 2015.(10)

Page 56: CAF7-Financial Accounting and Reporting II_Questionbank

Financial accounting and reporting II

© Emile Woolf International 46 The Institute of Chartered Accountants of Pakistan

6.2 HORNY

Statements of profit or loss for the year ended 31 December 2015.

Horny Smooth Rs. 000 Rs. 000 Revenue 304,900 195,300 Cost of sales (144,200) (98,550)

Gross profit 160,700 96,750 Operating costs (76,450) (52,100)

Operating profit 84,250 44,650 Investment income 10,500 2,600

Profit before tax 94,750 47,250 Income tax expense(42,900) (16,500)

Profit for the year 51,850 30,750

Statement of changes in equity (extracts) for the year ended 31 December 2015.

Horny Smooth Rs. 000 Rs. 000 Retained earnings brought forward 80,200 31,000 Profit for the year 51,850 30,750 Proposed ordinary dividend (20,000) -

112,050 61,750

The following information is also available.

(1) Horny acquired 75% of the share capital of Smooth on 31 August 2015.

(2) Negative goodwill of Rs. 3.8 million arose on the acquisition.

(3) Profits of both companies are deemed to accrue evenly over the year except for the investment income of Smooth all of which was received in November 2015.

(4) Horny has bought goods from Smooth throughout the year at Rs. 2 million per month. At the year-end Horny does not hold any inventory purchased from Smooth.

Required

Prepare the consolidated statement of profit or loss and a working showing the movement on consolidated retained profit for the year ended 31 December 2015.(10)

Page 57: CAF7-Financial Accounting and Reporting II_Questionbank

Questions

© Emile Woolf International 47 The Institute of Chartered Accountants of Pakistan

6.3 HERON

Statements of financial position as at 30 June 2015 Heron Stork Assets Rs. 000 Rs. 000 Non-current assets

Property, plant and equipment 31,000 15,000 Investment in Stork ( 1,000 ordinary shares) 1,000

32,000 15,000 Current assets 23,000 11,000

55,000 26,000

Shareholders’ equity and liabilities

Share capital (Rs. 0001 ordinary shares) 10,000 1,500 Share premium 5,000 – Retained earnings 20,000 18,500

35,000 20,000 Non-current liabilities 15,000 – Current liabilities 5,000 6,000

55,000 26,000

Heron acquired its shares in Stork when the balance on the retained earnings was Rs. 000nil.

Statements of profit or loss for the year ended 30 June 2015 Heron Stork Rs. 000 Rs. 000 Revenue 30,000 25,000 Cost of sales (9,000) (10,000) Gross profit 21,000 15,000 Distribution costs (3,000) (1,200) Administrative expenses (1,000) (2,800) Finance costs (2,000) – Profit before tax 15,000 11,000 Income tax expense (3,000) (3,000) Profit for the period 12,000 8,000 ════ ════

Statement of changes in equity for the year ended 30 June 2015 (extract)

Retained earnings brought forward 8,000 10,500 Profit for the financial year 12,000 8,000 ——— ——— Retained earnings carried forward 20,000 18,500 ════ ════

Required

Prepare Heron’s consolidated statement of profit or loss, consolidated statement of financial position and a working showing the movement on consolidated retained profit for Heron for the year ended 30 June 2015. (12)

Page 58: CAF7-Financial Accounting and Reporting II_Questionbank

Financial accounting and reporting II

© Emile Woolf International 48 The Institute of Chartered Accountants of Pakistan

6.4 HANKS

Statements of financial position as at 31 December 2015

Hanks Streep Scott Rs. 000 Rs. 000 Rs. 000 Assets Non-current assets Property, plant and equipment 32,000 25,000 20,000 Investments 33,500 – – ———– ——— ———

65,500 25,000 20,000 Current assets Cash at bank and in hand 9,500 2,000 4,000 Trade receivables 20,000 8,000 17,000 Inventory 30,000 18,000 18,000 ———– ——— ———

125,000 53,000 59,000 ———– ——— ———

Equity and liabilities Share capital 40,000 10,000 15,000 Share premium account 6,500 – – Retained earnings 55,000 37,000 27,000 ———– ——— ———

101,500 47,000 42,000

Current liabilities 23,500 6,000 17,000 ———– ——— ———

125,000 53,000 59,000 ═════ ═════ ═════

Statements of profit or loss for the year ended 31 December 2015

Hanks Streep Scott Rs. 000 Rs. 000 Rs. 000 Revenue 125,000 117,000 82,000

Cost of sales (65,000) (64,000) (42,000) ———– ———– ——— Gross profit 60,000 53,000 40,000 Distribution costs (21,000) (14,000) (16,000) Administrative expenses (14,000) (8,000) (7,000) ———– ———– ——— Profit before taxation 25,000 31,000 17,000 Income tax expense (10,000) (9,000) (5,000) ———– ———– ——— Profit after tax 15,000 22,000 12,000 ═════ ═════ ═════

Statement of changes in equity (extract) for the year ending 31 December 2015

Hanks Streep Scott Rs. 000 Rs. 000 Rs. 000 Retained earnings brought forward 40,000 15,000 15,000 Retained profit for the financial year 15,000 22,000 12,000 Dividends – – – ———– ———– ——— Retained earnings carried forward 55,000 37,000 27,000 ═════ ═════ ═════

Page 59: CAF7-Financial Accounting and Reporting II_Questionbank

Questions

© Emile Woolf International 49 The Institute of Chartered Accountants of Pakistan

You are given the following additional information

(1) Hanks owns 80% of Streep’s shares. These were purchased in 2012 for Rs. 20.5 million cash, when the balance on Streep’s retained earnings stood at Rs. 7million.

(2) In 2010 Hanks purchased 60% of the shares of Scott by the issue of shares with a nominal value of Rs. 6.5 million. These shares were issued at a premium of Rs. 6.5 million. At that date the retained earnings of Scott stood at Rs. 3 million and the fair value of the net assets of Scott was Rs. 24 million. It was agreed that any undervaluation of the net assets should be attributed to land. This land was still held at 31 December 2015.

(3) Included in the inventory of Scott and Streep at 31 December 2015 are goods purchased from Hanks for Rs. 5.2 million and Rs. 3.9 million respectively. Hanks aims to earn a profit of 30% on cost. Total sales from Hanks to Scott and to Streep were Rs. 8 million and Rs. 6 million respectively.

(4) Hanks and Streep each proposed a dividend before the year end of Rs. 2 million and Rs. 2.5 million respectively. No accounting entries have yet been made for these.

(5) Hanks has carried out annual impairment tests on goodwill in accordance with IFRS 3 and IAS 36. The estimated recoverable amount of goodwill at 31 December 2012 was Rs. 5 million and at 31 December 2015 was Rs. 4.5 million.

Required

Prepare the consolidated statement of profit or loss and consolidated statement of changes in equity for the year ended 31 December 2015 and the consolidated statement of financial position at that date. (20)

Page 60: CAF7-Financial Accounting and Reporting II_Questionbank

Financial accounting and reporting II

© Emile Woolf International 50 The Institute of Chartered Accountants of Pakistan

CHAPTER 7 – TANGIBLE NON-CURRENT ASSETS (IAS 16: PROPERTY,

PLANT AND EQUIPMENT AND IAS 23: BORROWING COSTS)

7.1 ROONEY

(a) Rooney has recently finished building a new item of plant for its own use. The item is a press for use in the manufacture of industrial diamonds. Rooney commenced construction of the asset on 1st April 2013 and completed it on 1st April 2015.

1st January 2013, Rooney took out a loan to finance the construction of the

asset. Interest is charged on the loan at the rate of 5% per annum. The annual interest must be paid in four equal instalments at the end of each quarter. Rooney capitalises interest on manufactured assets in accordance with the rules in IAS 23 Borrowing costs.

The costs (excluding finance costs) of manufacturing the asset were Rs. 28

million.

Required

State the IAS 23 rules on the capitalisation of borrowing costs, calculate the cost of the asset on initial recognition and explain the amount of borrowing cost capitalised. (6)

(b) The press comprises two significant parts, the hydraulic system and the

‘frame’. The hydraulic system has a three year life and the ‘frame’ has an eight year life. Rooney depreciates plant on a straight line basis. The cost of the hydraulic system is 30% of the total cost of manufacture.

Rooney uses the IAS 16 revaluation model in accounting for diamond presses

and revalues these assets on an annual basis. Revaluation surpluses or deficits are apportioned between the hydraulic

system and the ‘frame’ on the basis of their year end book values before the revaluation.

Required

Explain the IAS 16 rules on accounting for significant parts of property, plant and equipment and show the accounting treatment of the diamond press in the financial statements for the financial years ending:

(i) 31st March 2016 (assume that the press has a fair value of Rs. 21 million)

(ii) 31st March 2017 (assume that the press has a fair value of Rs. 19.6 million). (13)

(25)

Page 61: CAF7-Financial Accounting and Reporting II_Questionbank

Questions

© Emile Woolf International 51 The Institute of Chartered Accountants of Pakistan

7.2 EHTISHAM

The following information relates to the financial statements of Ehtisham for the year to 31 March 2015.

The head office of Ehtisham was acquired on 1 April 2012 for Rs. 1million. Ehtisham intend to occupy the building for 25 years. On 31 March 2014 it was revalued to Rs. 1.15 million. On 31 March 2015, a surplus of vacant commercial property in the area had led to a fall in property prices and the fair value was now only Rs. 0.8 million.

Required

Explain the correct accounting treatment for the above (with calculations if appropriate). (10)

7.3 CARLY

The following is an extract from the financial statements of Carly on 31 December 2014.

Property, plant and equipment

Land and buildings

Plant and equipment Computers

Total

Rs. Rs. Rs. Rs.

Cost

On 31 December 2014 1,500,000 340,500 617,800 2,458,300

Accumulated depreciation

On 31 December 2014 600,000 125,900 505,800 1,231,700

Carrying amount

On 31 December 2014 900,000 214,600 112,000 1,226,600

Accounting policies

Depreciation

Depreciation is provided at the following rates.

On land and buildings 2% per annum straight line on buildings only

On plant and equipment 25% reducing balance

On computers 33.33% per annum straight line

During 2015 the following transactions took place.

(1) On 31 December the land and buildings were revalued to Rs. 1,750,000. Of this amount, Rs. 650,000 related to the land (which had originally cost Rs. 500,000). The remaining useful life of the buildings was assessed as 40 years.

(2) A machine which had cost Rs. 80,000 and had accumulated depreciation of Rs. 57,000 at the start of the year was sold for Rs. 25,000 in the first week of the year.

Page 62: CAF7-Financial Accounting and Reporting II_Questionbank

Financial accounting and reporting II

© Emile Woolf International 52 The Institute of Chartered Accountants of Pakistan

(3) A new machine was purchased on 31 March 2015. The following costs were incurred:

Rs.

Purchase price, before discount, inclusive of reclaimable

sales tax of Rs. 3,000 20,000

Discount 1,000

Delivery costs 500

Installation costs 750

Interest on loan taken out to finance the purchase 300

(4) On 1 January it was decided to change the method of providing depreciation on computer equipment from the existing method to 40% reducing balance.

Required

Produce the analysis of property, plant and equipment as it would appear in the financial statements of Carly for the year ended 31 December 2015.

7.4 ADJUSTMENTS LIMITED

Adjustments Limited has carried out a review of its non-current assets.

(a) A lathe was purchased on 1 January 2009 for Rs. 150,000. The plant had an estimated useful life of twelve years, residual value of nil. Depreciation is charged on the straight line basis. On 1 January 2015, when the asset’s net book value is Rs. 75,000, the directors decide that the asset’s total useful life is only ten years.

(b) A grinder was purchased on 1 January 2012 for Rs. 100,000. The plant had an estimated useful life of ten years and a residual value of nil. Depreciation is charged on the straight line basis. On 1 January 2015, when the asset’s net book value is Rs. 70,000, the directors decide that it would be more appropriate to depreciate this asset using the sum of digits approach. The remaining useful life is unchanged.

(c) The company purchased a fifty year lease some years ago for Rs. 1,000,000. This was being depreciated over its life on a straight line basis. On 1 January 2015, when the net book value is Rs. 480,000 and twenty-four years of the lease are remaining, the asset is revalued to Rs. 1,500,000. This revised value is being incorporated into the accounts.

Required

Explain the effects of these changes on the depreciation for the year to 31 December 2015. (15)

Page 63: CAF7-Financial Accounting and Reporting II_Questionbank

Questions

© Emile Woolf International 53 The Institute of Chartered Accountants of Pakistan

7.5 FAM

Fam had the following tangible fixed assets at 31 December 2014.

Cost Depreciation NBV Rs. 000 Rs. 000 Rs. 000 Land 500 – 500 Buildings 400 80 320 Plant and machinery 1,613 458 1,155 Fixtures and fittings 390 140 250 Assets under construction 91 – 91 ——— —— ——— 2,994 678 2,316 ════ ════ ════

In the year ended 31 December 2015 the following transactions occur.

(1) Further costs of Rs. 53,000 are incurred on buildings being constructed by the company. A building costing Rs. 100,000 is completed during the year.

(2) A deposit of Rs. 20,000 is paid for a new computer system which is undelivered at the year end.

(3) Additions to plant are Rs. 154,000.

(4) Additions to fixtures, excluding the deposit on the new computer system, are Rs. 40,000.

(5) The following assets are sold.

Cost Depreciation Proceeds brought forward Rs. 000 Rs. 000 Rs. 000 Plant 277 195 86 Fixtures 41 31 2

(6) Land and buildings were revalued at 1 January 2015 to Rs. 1,500,000, of which land is worth Rs. 900,000. The revaluation was performed by Messrs Jackson & Co, Chartered Surveyors, on the basis of existing use value on the open market.

(7) The useful economic life of the buildings is unchanged. The buildings were purchased ten years before the revaluation.

(8) Depreciation is provided on all assets in use at the year end at the following rates.

Buildings 2% per annum straight line

Plant 20% per annum straight line

Fixtures 25% per annum reducing balance

Required

Show the disclosure under IAS 16 in relation to fixed assets in the notes to the published accounts for the year ended 31 December 2015. (14)

Page 64: CAF7-Financial Accounting and Reporting II_Questionbank

Financial accounting and reporting II

© Emile Woolf International 54 The Institute of Chartered Accountants of Pakistan

7.6 IMRAN LIMITED

On January 1, 2015, Imran Limited started the construction of its new factory. The construction period is approximately 15 months and the cost is estimated at Rs. 80 million. The work has been divided into 5 phases and payment to contractor shall be made on completion of each phase.

In the year the company had the following sources of finance available.

(i) Rights i ssue o f shares amounting to Rs. 15 million on January 1, 2015. The company usually pays a dividend of 10% each year.

(ii) Bank loan of Rs. 32 million carrying a mark-up of 13% was raised on March 1, 2015. (This loan was outstanding for 306 days in the year).

(iii) On August 1, 2015, Rs. 10 million were borrowed from the bank. Interest thereon, is payable at the rate of 11%. (This loan was outstanding for 153 days in the year).

Investment income on temporary investment of the borrowings amounted to Rs. 0.5 million.

The details of bills submitted by the contractor, during the year are as follows:

Particulars Date of payment Rupees

On completion of 1st phase March 1, 2015 20,000,000

On completion of 2nd phase April 1, 2015 18,000,000

On completion of 3rd phase October 1, 2015 16,000,000

On completion of 4th phase Payment not yet made 17,000,000

On June 1, 2015, the Building Control Authority issued instructions for stoppage of work on account of certain discrepancies in the completion plan. The company filed a petition in the Court and the matter was decided in the company’s favour on July 31, 2015. Work recommenced after a delay of 61 days.

The following periods may be relevant:

Period Days

March 1 to December 31 306

April 1 to December 31 275

August 1 to December 31 153

October 1 to December 31 92

Required

a) Assuming that the loans were taken specifically for the project, calculate the amount of borrowing costs that s h ou l d be capitalised in the per iod end ing December 31, 2015 in accordance with the requirements of IAS 23 Borrowing Costs.

b) Assuming that the loans constituted general finance, calculate the amount of borrowing costs that s ho u l d be capitalised in the per iod end ing December 31, 2015 in accordance with the requirements of IAS 23 Borrowing Costs.

Page 65: CAF7-Financial Accounting and Reporting II_Questionbank

Questions

© Emile Woolf International 55 The Institute of Chartered Accountants of Pakistan

7.7 HUMAYUN CHEMICALS LIMITED

(a) On July 1, 2013, Humayun Chemicals Limited acquired a machine at a cost of Rs. 10 million. The useful life of the machine and its salvage value was estimated at 5 years and Rs. 3.0 million respectively. The cost of machine is being depreciated under the straight line method.

Based on the practice followed by similar type of companies, the company has determined that the remaining useful economic life of the machine is six years. It has also been established that the residual value at the end of the useful life will be equal to 10% of the cost of machine.

Required

Compute the depreciation expenses and other adjustments (if any) required to be made in the financial statements of the company for the year ended June 30, 2015 under each of the following assumptions:

(i) the review of useful life and residual value was carried out on June 30, 2015;

(ii) the review of useful life and residual value was carried out on June 30, 2014 but in the financial statements for the year then ended the depreciation expense was erroneously recorded on the previous basis.

(11)

(b) Discuss the requirements of International Accounting Standard(s) in respect of estimation and revision of useful life of an item of property, plant and equipment. (04)

7.8 FARADAY PHARMACEUTICAL LIMITED

Faraday Pharmaceutical Limited (FPL) acquired a building for Rs. 200 million on July 1, 2011. The following information relating to the building is available:

(i) It is being depreciated on the straight line basis, over 20 years.

(ii) FPL uses the revaluation model for subsequent measurement of its property, plant and equipment and accounts for revaluations on the net replacement value method. The details of revaluation carried out by the independent valuers during the past years are as follows:

Revaluation date Fair value

Rupees in million

July 1, 2012 230

July 1, 2013 170

July 1, 2014 180

(iii) FPL transfers the maximum possible amount from the revaluation surplus to retained earnings on an annual basis.

(iv) There is no change in the useful life of the building.

Required

Prepare the journal entries to record the above transactions from the date of acquisition of the building to the year ended June 30, 2015. (Ignore deferred tax) (16)

Page 66: CAF7-Financial Accounting and Reporting II_Questionbank

Financial accounting and reporting II

© Emile Woolf International 56 The Institute of Chartered Accountants of Pakistan

7.9 SPIN INDUSTRIES LIMITED

On September 1, 2014, Spin Industries Limited (SIL) started construction of its new office building and completed it on May 31, 2015. The payments made to the contractor were as follows:

Date of Payment Rupees

September 1, 2014 10,000,000

December 1, 2014 15,000,000

February 1, 2015 12,000,000

June 1, 2015 9,000,000

In addition to the above payments, SIL paid a fee of Rs. 8 million on September 1, 2014 for obtaining a permit allowing the construction of the building.

The project was financed through the following sources:

(i) On August 1, 2014 a medium term loan of Rs. 25 million was obtained specifically for the construction of the building. The loan carried mark up of 12% per annum payable semi-annually. A commitment fee @ 0.5% of the amount of loan was charged by the bank.

Surplus funds were invested in savings account @ 8% per annum. On February 1, 2015 SIL paid the six monthly interest plus Rs. 5 million towards the principal.

(ii) Existing running finance facilities of SIL

Running finance facility of Rs. 28 million from Bank A carrying mark up of 13% payable annually. The average outstanding balance during the period of construction was Rs. 25 million.

Running finance facility of Rs. 25 million from Bank B. The mark up accrued during the period of construction was Rs. 3 million and the average running finance balance during that period was Rs. 20 million.

Required

Calculate the amount of borrowing costs to be capitalised on June 30, 2015 in accordance with the requirements of International Accounting Standards. (Borrowing cost calculations should be based on number of months). (18)

7.10 SCIENTIFIC PHARMA LIMITED

Scientific Pharma Limited (SPL) is a manufacturer of pharmaceutical products. In January 2015, one of its plants suffered a major break down. It was repaired at a cost of Rs. 1.5 million but the production capacity was reduced significantly. The plant was ready for production on June 30, 2015. At that time the company’s engineers advised that the plant could be used at a reduced level for 3 years only. The factory was estimated to have a recoverable amount of Rs. 19,277,000 at June 30, 2015

Other related information is as under:

(i) The plant was imported at FOB price of US$ 800,000. The payment was made at the time of shipment on July 1, 2005 at Rs. 52 per US$. Other charges including installation cost amounted to Rs. 7 million. Installation of the plant was completed on December 31, 2005 and commercial production commenced from April 1, 2006.

Page 67: CAF7-Financial Accounting and Reporting II_Questionbank

Questions

© Emile Woolf International 57 The Institute of Chartered Accountants of Pakistan

(ii) The company uses straight line method of deprecation. Depreciation is charged from the month the asset is available for use upto the month prior to disposal. At the time of purchase, the estimated useful life of the plant was estimated at 15 years whereas the salvage value was estimated at Rs. 2.0 million.

(iii) Based on the report of a professional independent valuer, the plant was revalued on July 1, 2010 at Rs. 45 million. There was however, no change in estimated useful life of the plant.

(iv) The factory remained closed from April 1, to June 30, 2012 due to law and order situation.

(v) The salvage value has not changed since it was first estimated at the time of purchase.

Required

Prepare accounting entries for the year ended June 30, 2015. Give all the necessary calculations.

(Ignore taxation) (20)

7.11 QURESHI STEEL LIMITED

On July 1, 2014, Qureshi Steel Limited (QSL) signed an agreement with Pak Construction Limited for construction of a factory building at a cost of Rs. 100 million. It was agreed that the factory would be ready for use from January 1, 2016. The terms of payments were agreed as under:

(i) 10% advance payment would be made on signing of the agreement. The advance paid would be adjusted at 10% of the quarterly progress bills.

(ii) 5% retention money would also be deducted from the progress bills. Retention money will be refunded one year after completion of the factory building.

(iii) Progress bills will be raised on last day of each quarter and settled on 15th of the next month.

The under mentioned progress bills were received and settled by QSL as per the agreement:

Invoice date Amount (Rs. )

September 30, 2014 30 million

December 31, 2014 20 million

March 31, 2015 10 million

June 30, 2015 15 million

On April 30, 2015 an invoice of Rs. 1.5 million was raised by the contractor for

damages sustained at the site, on account of rains. After negotiations, QSL finally agreed to make additional payment of Rs. 1.0 million to compensate the contractor. The amount was paid on May 15, 2015. It is expected that 75% of the payment would be recovered from the insurance company.

The cost of the project has been financed through the following sources:

(i) Issue of right shares amounting to Rs. 15 million, on September 1, 2014. The company has been following a policy of paying dividend of 20% for the past many years.

Page 68: CAF7-Financial Accounting and Reporting II_Questionbank

Financial accounting and reporting II

© Emile Woolf International 58 The Institute of Chartered Accountants of Pakistan

(ii) Bank loan of Rs. 25 million obtained on December 1, 2014. The loan carries a markup of 13% per annum. The principal is repayable in 5 half yearly equal instalments of Rs. 5 million each along with the interest, commencing from May 31, 2015. Loan processing charges of Rs. 0.5 million were deducted by the bank at the time of disbursement of loan. Surplus funds, when available, were invested in short term deposits at 8% per annum.

(iii) Cash withdrawals from the existing running finance facility provided by a bank. Average running finance balance for the year was Rs. 60 million. Markup charged by the bank for the year was Rs. 9 million.

Required

Compute cost of capital work in progress for the factory building as of June 30, 2015 in accordance with the requirements of relevant IFRSs.

(Borrowing costs calculations should be based on number of months) (18)

7.12 GRANITE CORPORATION

On 1 March 2014, Granite Corporation (GC) started the construction of a new plant to meet the growing demand for its products. The new plant was completed at a cost of Rs. 100 million on 31 May 2015.

GC financed the cost of the project from the following sources:

(i) On 1 March 2014, a 7-year loan of Rs. 70 million was obtained specifically for the construction of the plant. The loan carried mark-up @ 13% per annum payable semi-annually. An arrangement fee @ 1% of the loan amount was paid to the bank.

Two instalments, each comprising of repayment of principal of Rs. 5 million with interest, were paid on 31 August 2014 and 28 February 2015.

(ii) GC also has a running finance facility of Rs. 100 million carrying mark-up @ 14% per annum. Average utilization of this facility, prior to commencement of construction was Rs. 10 million. Any additional amount required for the project was provided through this facility.

(iii) Surplus funds were used to reduce the running finance utilization or invested in savings account @ 8% per annum.

Payments made to the contractor were as follows:

Payment date Rs. m

01 March 2014 25

31 January 2015 65

30 September 2015 10

The construction work was suspended from 1 February 2015 to 28 February 2015. The suspension was caused due to delay in shipment of essential components for the installation of the plant.

Required

Calculate the amount of borrowing costs that may be capitalised during the years ended 30 June 2014 and 2015 in accordance with the requirements of International Financial Reporting Standards. (20)

Page 69: CAF7-Financial Accounting and Reporting II_Questionbank

Questions

© Emile Woolf International 59 The Institute of Chartered Accountants of Pakistan

CHAPTER 8 – IAS 38: INTANGIBLE ASSETS

8.1 FAZAL

The following information relates to the financial statements of Fazal for the year to 31 March 2015.

The IT division has begun a training course for all managers in a new programming language at a cost of Rs. 200,000. The consultants running the training course have quantified the present value of the training benefits over the next two years to be Rs. 400,000. The project cost has been included in the statement of financial position as a current asset. The accounting policy note identifies that the costs will be written off over the next two years to match the benefits.

Required

Explain the correct accounting treatment for the above (with calculations if appropriate). (3)

8.2 HENRY

During 2015 Henry has the following research and development projects in progress.

Project A was completed at the end of 2014. Development expenditure brought forward at the beginning of 2015 was Rs. 412,500 on this project. Savings in production costs arising from this project are first expected to arise in 2015. In 2015 savings are expected to be Rs. 100,000, followed by savings of Rs. 300,000 in 2016 and Rs. 200,000 in 2017.

Project B commenced on 1 April 2015. Costs incurred during the year were Rs. 56,000. In addition to these costs a machine was purchased on 1 April 2015 for Rs. 30,000 for use on the project. This machine has a useful life of five years. At the end of 2015 there were still some uncertainties surrounding the completion of the project.

Project C had been started in 2014. In 2014 the costs relating to this project of Rs. 36,700 had been written off, as at the end of 2014 there were still some uncertainties surrounding the completion of the project. Those uncertainties have now been resolved and a further Rs. 45,000 costs incurred during the year.

Required

Show how the above would appear in the financial statements (including notes to the financial statements) of Henry as of 31 December 2015.

Page 70: CAF7-Financial Accounting and Reporting II_Questionbank

Financial accounting and reporting II

© Emile Woolf International 60 The Institute of Chartered Accountants of Pakistan

8.3 TOBY

Toby entered into the following transactions during the year ended 31 December 2015. The directors of Toby wish to capitalise all assets wherever possible.

(1) On 1 January Toby acquired the net assets of George for Rs. 105,000. The assets acquired had the following book and fair values.

Book value Fair value

Rs. Rs. Goodwill 5,000 5,000 Patents 15,000 20,000 Non-current assets 40,000 50,000 Other sundry net assets 30,000 25,000 ––––––– –––––––– 90,000 100,000 ═════ ═════

The patent expires at the end of 2022. The goodwill arising from the above had a recoverable value at the end of 2015 of Rs. 7,000.

(2) On 1 April Toby acquired a brand from a competitor for Rs. 50,000. The directors of Toby have assessed the useful life of the brand as five years.

(3) During the year Toby spent Rs. 40,000 on developing a new brand name. The development was completed on 30 June. The useful life of this brand has been assessed as eight years.

(4) The directors of Toby believe that there is total goodwill of Rs. 2 million within Toby and that this has an indefinite useful life.

Required

Prepare the note to the financial statements for intangible assets as at 31 December 2015.

8.4 BROOKLYN

Brooklyn is a bio-technology company performing research for pharmaceutical companies. The finance director has contacted your financial consulting company to arrange a meeting to discuss issues relevant to the preparation of the financial statements for the year to 30th June 2015. Your initial telephone conversation has provided the necessary background information.

1 On 1st August 2014 Brooklyn began investigating a new bio-process. On 1st September 2015, the new process was widely supported by the scientific community and the feasibility project was approved. A grant was then obtained relating to future work. Several pharmaceutical companies have expressed an interest in buying the ‘know how’ when the project completes in June 2016. The nominal ledger account set up for the project shows that the expenditure incurred between 1st August 2014 and 30th June 2015 was Rs. 300,000 per month.

2 In August 2015, an employee lodged a legal claim against the company for damage to his health as a result of working for the company for the two years through to 31st March 2014 when he had to retire due to ill health. He has argued that his health deteriorated as a result of the stress from his position in the organisation. Brooklyn has denied the claim and has appointed an employment lawyer to assist with contesting the case. The lawyer has advised that there is a 25% chance that the claim will be rejected, 50% chance that the damages will be Rs. 600,000 and 25% chance of Rs. 1 million. The company has an insurance policy that will pay 10% of any damages to the company.

Page 71: CAF7-Financial Accounting and Reporting II_Questionbank

Questions

© Emile Woolf International 61 The Institute of Chartered Accountants of Pakistan

The lawyer has said that the case could take until 30th June 2018 to resolve. The present value of the estimated damages discounted at 8% is Rs. 476,280 and Rs. 793,800 respectively.

3 Brooklyn owns several buildings, which include an administrative office in the centre of London. The company has revalued these on a regular basis every five years and the next valuation is due on 30th June 2017. Property prices have increased since the last review and particularly for the London premises. The cost of engaging a professionally qualified valuer is very expensive and so to reduce costs the finance director is proposing that the property manager, who is a professionally qualified valuer, should value the London property and that the increase in value should be included in the financial statements. The finance director is of the opinion that the property prices may fall next year.

Required

Prepare notes for your meeting with the finance director which explain and justify the accounting treatment of these issues, preparing calculations where appropriate and identifying matters on which your require further information. (25)

8.5 ZOUQ INC

Zouq Inc. is a multinational company. As part of its vision to expand its business in South Asia, it purchased a 90% share of a locally incorporated company, Momin Limited. Following are the brief details of the acquisition:

Date of acquisition January 1, 2014

Total paid up capital of Momin Limited (Rs. 10 each) 500,000,000

Purchase price per share Rs. 30

Net assets of Momin Limited (as per 2013 audited financial statements) 650,000,000

Fair value of net assets (other than intangible assets) of Momin Limited 1,100,000,000

Momin Limited has an established line of products under the brand name of “Badar”. On behalf of Zouq Inc., a firm of specialists has valued the brand name at Rs. 100 million with an estimated useful life of 10 years at January 1, 2014. It is expected that the benefits will be spread equally over the brand’s useful life.

An impairment test of goodwill and brand was carried out on December 31, 2014 which indicated an impairment of Rs. 50 million in the value of goodwill.

An impairment test carried out on December 31, 2015 indicated a decrease of Rs. 13.5 million in the carrying value of the brand.

Required:

(a) What are the requirements of International Accounting Standards relating to amortization of intangible assets having finite life?

(b) Prepare the ledger accounts for goodwill and the brand, showing initial recognition and all subsequent adjustments. (15)

Page 72: CAF7-Financial Accounting and Reporting II_Questionbank

Financial accounting and reporting II

© Emile Woolf International 62 The Institute of Chartered Accountants of Pakistan

8.6 STAR-BRIGHT PHARMACEUTICAL LIMITED

Star-Bright Pharmaceutical Limited (SPL), a listed company, purchased a brand on January 1, 2010 at a cost of Rs. 382 million. It has incurred a substantial amount on further development of the brand, in subsequent years.

It is the policy of SPL to amortise the development expenditures which meet the recognition criteria as given in IAS-38 ‘Intangible Assets’, over a period of ten years. The amortization commences when the development expenditures first meet the recognition criteria. However, it was discovered during the year 2015 that the development expenditure incurred after acquisition had erroneously been written-off to the profit and loss account, details of which are as follows:

Year ended Rs. m

December 31, 2012 24

December 31, 2013 54

December 31, 2014 38

December 31, 2015 43

The draft financial statements (before correction of error) show that retained earnings as at December 31, 2015 was Rs. 1,950 million (2014: Rs. 1,785 million).

Required

In accordance with the requirements of International Financial Reporting Standards, prepare relevant extracts of the Statement of Financial Position along with the note on intangible assets after incorporating the required corrections. (Ignore tax) (16)

8.7 RAISIN INTERNATIONAL

(a) Discuss the criteria that should be used while recognizing intangible assets arising from research and development work. (05 marks)

(b) Raisin International (RI) is planning to expand its line of products. The related information for the year ended 31 December 2015 is as follows:

(i) Research and development of a new product commenced on 1 January 2015. On 1 October 2015, the recognition criteria for capitalization of an internally generated intangible asset were met. It is estimated that the product would have a useful life of 7 years. Details of expenditures incurred are as follows:

Rs. m

Research work 4.50

Development work 9.00

Training of production staff 0.50

Cost of trial run 0.80

Total costs 14.80

Page 73: CAF7-Financial Accounting and Reporting II_Questionbank

Questions

© Emile Woolf International 63 The Institute of Chartered Accountants of Pakistan

(ii) The right to manufacture a well-established product under a patent for a period of five years was purchased on 1 March 2015 for Rs. 17 million. The patent has an expected remaining useful life of 10 years. RI has the option to renew the patent for a further period of five years for a sum of Rs. 12 million.

(iii) RI has acquired a brand at a cost of Rs. 2 million. The cost was incurred in the month of June 2015. The life of the brand is expected to be 10 years. Currently, there is no active market for this brand. However, RI is planning to launch an aggressive marketing campaign in February 2016.

(iv) In September 2014, RI developed a new production process and capitalised it as an intangible asset at Rs. 7 million. The new process is expected to have an indefinite useful life. During 2015, RI incurred further development expenditure of Rs. 3 million on the new process which meets the recognition criteria for capitalization of an intangible asset.

Required

In the light of International Financial Reporting Standards, explain how each of the above transaction should be accounted for in the financial statements of Raisin International for the year ended 31 December 2015. (11)

Page 74: CAF7-Financial Accounting and Reporting II_Questionbank

Financial accounting and reporting II

© Emile Woolf International 64 The Institute of Chartered Accountants of Pakistan

CHAPTER 9 – IAS 17: LEASES

9.1 DAWOOD

The following information relates to the financial statements of Dawood for the year to 31 March 2015.

On 1 October 2014, Dawood entered into a 5 year lease for a machine from Narbonne, agreeing to make payments every 6 months of Rs. 29,500 beginning on the 1 October. The cash price of the machine is Rs. 250,000 and the machine is believed to have a useful life of 5 years. Dawood has treated the arrangement as a finance lease. Any finance costs are to be treated using the sum-of-digits method.

Required

Explain the correct accounting treatment for the above (with calculations if appropriate). (7)

9.2 FINLEY

On 1 January 2015, Finley entered into an agreement to lease a boat. The fair value of the boat was Rs. 36,000 and the term of the lease was four years. Annual lease payments of Rs. 10,000 are payable in advance. The interest rate implicit in the lease is 7.5%. Finley is responsible for insuring and maintaining the boat throughout the term of the lease.

Required

Show how this lease would be presented in the statement of profit or loss of Finley for the year ended 31 December 2015 and the statement of financial position as at that date. Detailed disclosure notes are not required.

9.3 FABIAN

In the year ended 31 December 2015, Fabian leased two assets.

(1) A car was leased on 1 July 2015 via a three year lease agreement. Fabian paid a deposit of Rs. 7,500 followed by 36 monthly payments of Rs. 700 each on the 1st of each month. At the end of the three years Fabian will return the car. The car has a useful life of eight years.

(2) A machine was leased on 1 January 2015 via a four year lease. The machine has a fair value of Rs. 130,000 and Fabian is responsible for its upkeep. Lease payments of Rs. 40,000 are payable in arrears annually. The interest rate implicit in the lease is 10% and the present value of the minimum lease payments is Rs. 126,760.

Required

Show how the two lease agreements would be presented in the statement of profit or loss for 2015 and the statement of financial position at 31 December 2015. Notes to the financial statements are not required.

Page 75: CAF7-Financial Accounting and Reporting II_Questionbank

Questions

© Emile Woolf International 65 The Institute of Chartered Accountants of Pakistan

9.4 XYZ INC

A lessor, ABC Inc, leases an asset, which it purchased for Rs. 4,400,000, to XYZ Inc under a finance lease. It estimates that its residual value after five years will be Rs. 400,000 and after seven years will be zero.

The lease is for five years at a rental of Rs. 600,000 per half year in advance, with an option of two more years at nominal rental. The lease commences on 1 January 2015. The directors of XYZ Inc consider that the asset has a useful life of seven years. The finance charge is to be allocated using the sum of digits (“rule of 78”) method. Title to the asset will pass to XYZ at the end of seven years if the option is exercised. It is likely that it will be.

Required

(a) Show the relevant extracts from the accounts of XYZ Inc at 31 December 2015. (5)

(b) Show the allocation of the finance charge for XYZ Inc using the actuarial before tax method (using the interest rate implicit in the lease). Compare this with the sum of the digits allocation in (a) above. (14)

The rate of interest implicit in the lease is 7.68% per half year.

9.5 SNOW INC

On 1 January 2015, Snow Inc entered into the following finance lease agreements.

(a) Snowplough

To lease a snowplough for 3 years from Ice Inc. The machine had cost Ice Inc Rs. 35,000,000.

A deposit of Rs. 2,000,000 was payable on 1 January 2015 followed by 6 half yearly instalments of Rs. 6,500,000 payable in arrears, commencing on 30 June 2015. Finance charges are to be allocated on a sum of digits basis.

(b) Snow machine

To lease a snow machine for 5 years from Slush Inc. The snow machine cost Slush Inc Rs. 150,000 and is estimated to have a useful life of 5 years.

Snow Inc has agreed to make 5 annual instalments of Rs. 35,000,000, payable in advance, commencing on 1 January 2015.

The interest rate implicit in the lease is 8.36%.

Required

Show the relevant extracts from the accounts of Snow Inc for year ended 31 December 2015. (15)

Page 76: CAF7-Financial Accounting and Reporting II_Questionbank

Financial accounting and reporting II

© Emile Woolf International 66 The Institute of Chartered Accountants of Pakistan

9.6 MIRACLE TEXTILE LIMITED

On 1 July 2013, Miracle Textile Limited (MTL) acquired a machine on lease, from a bank.

Details of the lease are as follows:

(i) Cost of machine is Rs. 20 million.

(ii) The lease term and useful life is 4 years and 10 years respectively.

(iii) Instalment of Rs. 5.80 million is to be paid annually in advance on 1 July.

(iv) The interest rate implicit in the lease is 15.725879%.

(v) At the end of lease term, MTL has an option to purchase the machine on payment of Rs. 2 million. The fair value of the machine at the end of lease term is expected to be Rs. 3 million.

MTL depreciates the machine on the straight line method to a nil residual value.

Required

Prepare relevant extracts of the statement of financial position and related notes to the financial statements for the year ended 30 June 2015 along with comparative figures. Ignore taxation (16)

9.7 SHOAIB LEASING LIMITED

Shoaib Leasing Limited (the lessor) has entered into a three year agreement with Sarfaraz Limited (the lessee) to lease a machine with an expected useful life of 4 years. The cost of machine is Rs. 2,100,000.

The following information relating to lease transaction is available:

(i) Date of commencement of lease is July 1, 2015.

(ii) The lease contains a purchase bargain option at Rs. 100,000. At the end of the lease term, the value of the machine will be Rs. 300,000.

(iii) Lease instalments of Rs. 860,000 are payable annually, in arrears, on June 30.

(iv) The implicit interest rate is 12.9972%.

Required

(a) Prepare the journal entries for the years ending June 30, 2016, 2017 and 2018 in the books of lessor. Ignore tax.

(b) Produce extracts from the statement of financial position including relevant notes as at June 30, 2016 to show how the transactions carried out in 2016 would be reflected in the financial statements of the lessor.

(Disclosure of accounting policy is not required.) (20)

Page 77: CAF7-Financial Accounting and Reporting II_Questionbank

Questions

© Emile Woolf International 67 The Institute of Chartered Accountants of Pakistan

9.8 NEPTUNE LIMITED

Neptune Limited (NL) had established its business in December 2014 as a supplier of plant and machinery. During the year ended December 31, 2015 the company sold two machines under lease arrangements. The details are as under:

A B

Date of commencement of lease January 1, 2015 January 1, 2015

Lease period 6 years 3 years

Lease instalments payable annually in advance

Rs. 2,000,000 Rs. 4,000,000

(to be reduced annually by 5%)

Cost of machine Rs. 6,963,448 Rs. 15,000,000

Economic life 6 years 6 years

NL sells machines on cash at cost plus 25%. It depreciates its assets under straight line method with no residual value. Fair market annual interest rate is 15%.

Required

(a) Prepare journal entries to record the above transactions.

(b) Prepare notes to the financial statements for the year ended December 31, 2015 in accordance with the requirements of IAS - 17 (Leases). (19)

(Ignore taxation and comparative figures)

9.9 QUARTZ AUTO LIMITED

Quartz Auto Limited (QAL) is engaged in the business of manufacturing of trucks. Since a number of the prospective customers do not have adequate funds to purchase the vehicles against full payment, QAL provides lease financing facility to its customers. It expects to receive a return at the rate of 15% per annum on the amount of lease finance.

On 1 July 2014, QAL sold seven trucks to Emerald Goods Transport Company (EGTC) on lease. The terms of the lease and related information are as follows:

(i) The lease period is 4 years, extendable up to the expected useful life of the trucks i.e. 5 years.

(ii) EGTC has guaranteed a residual value of Rs. 360,000 for each truck, till the end of the fourth year. However, the guarantee would lapse if the lease term is extended to the fifth year. EGTC will return the truck at the end of the lease term.

(iii) Lease rentals amount to Rs. 2,715,224 per annum and are payable in arrears i.e. on 30 June.

(iv) The cost of each truck is Rs. 900,000. Price in case of outright sale is Rs. 1,350,000 per truck.

(v) The expected residual value of each truck at the end of the 4th and 5th year is Rs. 150,000 and Rs. 100,000 respectively.

Required

Assuming that QAL and EGTC intend to extend the lease for a period of five years, prepare:

(a) Journal entries to record the transactions for the year ended 30 June 2015. (08)

(b) A note for inclusion in the financial statements, for the year ended 30 June 2015, in accordance with the requirements of IAS 17 ‘Leases’. (07)

Page 78: CAF7-Financial Accounting and Reporting II_Questionbank

Financial accounting and reporting II

© Emile Woolf International 68 The Institute of Chartered Accountants of Pakistan

9.10 LODHI TEXTILE MILLS LIMITED

Lodhi Textile Mills Limited is facing severe financial difficulties. To improve the cash flows, the management has decided to sell and lease back three power generators of the company under three different sale and lease back arrangements which were signed on August 15, 2015. The company has assessed that all the leases shall qualify as finance leases.

The related information as on August 15, 2015 is given below:

Cost Book Value

Fair Value Value in Use

Amount of Financing

Rs. 000 Rs. 000 Rs. 000 Rs. 000 Rs. 000

Generator A 10,000 7,500 6,000 6,500 6,000

Generator B 12,000 6,000 4,000 5,000 6,000

Generator C 10,000 7,000 10,000 12,000 8,000

Required

Prepare the accounting entries that should be recorded by the company on August 15, 2015 in respect of the above transactions. (13)

Note: Ignore tax and deferred tax implications, if any.

9.11 NOMAN ENGINEERING LIMITED

Noman Engineering Limited (NEL) manufactures auto parts. On July 1, 2014 it finalised a lease agreement with a bank for sale and leaseback of one of its plants costing Rs. 18.75 million.

Relevant information is as under:

(i) Proceeds from the bank amounting to Rs. 20 million which represent the prevailing market value of such type and age of plant, were received on July 1, 2014.

(ii) The plant had a book value of Rs. 15 million at the time of commencement of the lease.

(iii) The remaining life of the plant on July 1, 2014 was estimated at 8 years.

(iv) The lease period is 6 years. Lease instalments of Rs. 2.5 million each are payable semi-annually in arrears from December 31, 2014.

(v) NEL has the option to purchase the plant at market value at the end of the lease term. No final decision has yet been made by NEL, in this regard.

(vi) The rate of interest implicit in the lease is 13.731% per annum.

Required

Pass journal entries in respect of the lease, for the year ended June 30, 2015. (12)

Page 79: CAF7-Financial Accounting and Reporting II_Questionbank

Questions

© Emile Woolf International 69 The Institute of Chartered Accountants of Pakistan

CHAPTER 10 – IAS 37: PROVISIONS CONTINGENT LIABILITIES AND

CONTINGENT ASSETS AND IAS 10: EVENTS OCCURRING AFTER THE

REPORTING DATE

10.1 BADAR

The following information relates to the financial statements of Badar for the year to 31 March 2015.

The mining division of Badar has a 3 year operating licence from an overseas government. This allows it to mine and extract copper from a particular site. When the licence began on 1 April 2014, Badar started to build on the site. The cost of the construction was Rs. 500,000.

The overseas country has no particular environmental decommissioning laws. In its past financial statements Badar has given information about the company’s environmental policy and has provided examples to demonstrate that it is a responsible company that believes in restoring mining sites at the end of the extraction period. The cost of removing the construction at the end of the three years is estimated to be Rs. 100,000.

The cost of the site currently shown in the trial balance is Rs. 500,000. The company has a cost of borrowing of 10%.

Required

Explain the correct accounting treatment for the above (with calculations if appropriate). (6)

10.2 GEORGINA

Georgina Company is preparing its financial statements for the year ended 30 September 2015. The following matters are all outstanding at the year end.

(1) Georgina is facing litigation for damages from a customer for the supply of faulty goods on 1 September 2015. The claim, which is for Rs. 500,000, was received on 15 October 2015. Georgina’s legal advisors consider that Georgina is liable and that it is likely that this claim will succeed. On 25 October 2015 Georgina sent a counter-claim to its suppliers for Rs. 400,000. Georgina’s legal advisors are unsure whether or not this claim will succeed.

(2) Georgina’s sales director, who was dismissed on 15 September, has lodged a claim for Rs. 100,000 for unfair dismissal. Georgina’s legal advisors believe that there is no case to answer and therefore think it is unlikely that this claim will succeed.

(3) Although Georgina has no legal obligation to do so, it has habitually operated a policy of allowing customers to return goods within 28 days, even where those goods are not faulty. Georgina estimates that such returns usually amount to 1% of sales. Sales in September 2015 were Rs. 400,000. By the end of October 2015, prior to the drafting of the financial statements, goods sold in September for Rs. 3,500 had been returned.

(4) On 15 September 2015 Georgina announced in the press that it is to close one of its divisions in January 2016. A detailed closure plan is in place and the costs of closure are reliably estimated at Rs. 300,000, including Rs. 50,000 for staff relocation.

Required

State, with reasons, how the above should be treated in Georgina’s financial statements for the year ended 30 September 2015.

Page 80: CAF7-Financial Accounting and Reporting II_Questionbank

Financial accounting and reporting II

© Emile Woolf International 70 The Institute of Chartered Accountants of Pakistan

10.3 EARLEY INC

Earley Inc is finalising its accounts for the year ended 31 December 2014. The following events have arisen since the year end and the financial director has asked you to comment on the final accounts.

(a) At 31 December 2014 trade receivables included a figure of Rs. 250,000 in respect of Nedengy Inc. On 8 March 2015, when the current debt was Rs. 200,000, Nedengy Inc went into receivership. Recent correspondence with the receiver indicates that no dividend will be paid to unsecured creditors.

(b) On 15 March 2015 Earley Inc sold its former head office building, Whitley Wood, for Rs. 2.7 million. At the year end the building was unoccupied and carried at a value of Rs. 3.1 million.

(c) Inventories at the year-end included Rs. 650,000 of a new electric tricycle, the Opasney. In January 2015 the European Union declared the tricycle to be unsafe and prohibited it from sale. An alternative market, in Bongolia, is being investigated, although the current price is expected to be cost less 30%.

(d) Stingy Inc, a subsidiary in Outer Sonning, was nationalised in February 2015. The Outer Sonning authorities have refused to pay any compensation. The net assets of Stingy Inc have been valued at Rs. 200,000 at the year end.

(e) Freak floods caused Rs. 150,000 damage to the Southcote branch of Earley Inc in January 2015. The branch was fully insured.

(f) On 1 April 2015 Earley Inc announced a 1 for 1 rights issue aiming to raise Rs. 15 million.

Required

Explain how you would respond to the matters listed above. (13)

10.4 ACCOUNTING TREATMENT

You have been asked to advise on the appropriate accounting treatment for the following situations arising in the books of various companies. The year end in each case can be taken as 31 December 2015 and you should assume that the amounts involved are material in each case.

(a) At the year end there was a debit balance in the books of a company for Rs. 15,000, representing an estimate of the amount receivable from an insurance company for an accident claim. In February 2016, before the directors had agreed the final draft of the published accounts, correspondence with lawyers indicated that Rs. 18,600 might be payable on certain conditions.

(b) A company has an item of equipment which cost Rs. 400,000 in 2012 and was expected to last for ten years. At the beginning of the 2015 financial year the book value was Rs. 280,000. It is now thought that the company will soon cease to make the product for which the equipment was specifically purchased. Its recoverable amount is only Rs. 80,000 at 31 December 2015.

(c) On 30 November a company entered into a legal action defending a claim for supplying faulty machinery. The company’s solicitors advise that there is a 20% probability that the claim will succeed. The amount of the claim is Rs. 500,000.

Page 81: CAF7-Financial Accounting and Reporting II_Questionbank

Questions

© Emile Woolf International 71 The Institute of Chartered Accountants of Pakistan

(d) An item has been produced at a manufacturing cost of Rs. 1,800 against a customer’s order at an agreed price of Rs. 2,300. The item was in inventory at the year-end awaiting delivery instructions. In January 2016 the customer was declared bankrupt and the most reasonable course of action seems to be to make a modification to the unit, costing approximately Rs. 300, which is expected to make it marketable with other customers at a price of about Rs. 1,900.

(e) At 31 December a company has a total potential liability of Rs. 1,000,400 for warranty work on contracts. Past experience shows that 10% of these costs are likely to be incurred, that 30% may be incurred but that the remaining 60% is highly unlikely to be incurred.

Required

For each of the above situations outline the accounting treatment you would recommend and give the reasoning of principles involved. The accounting treatment should refer to entries in the books and/or the year-end financial statements as appropriate. (12)

10.5 J-MART LIMITED

(a) Explain the terms “adjusting events” and “non-adjusting events” and give three examples of each. (05)

(b) J-Mart Limited, a chain of departmental stores has distributed its operations into four Divisions i.e. Food, Furniture, Clothing and Household Appliances. The following information has been extracted from the records:

(i) The company allows the dissatisfied customers to return the goods within 30 days. It is estimated that 5% of the sales made in June 2015 will be refunded in July 2015.

(ii) On June 2, 2015, three employees were seriously injured as a result of a fire at the company’s warehouse. They have lodged claims seeking damages of Rs. 2.0 million from the company. The company’s lawyers have advised that it is probable that the court may award compensation of Rs. 400,000.

(iii) Under a new legislation, the company is required to fit smoke detectors at all the stores by December 31, 2015. The company has not yet installed the smoke detectors.

(iv) On June 20, 2015, the board of directors decided to close down the Household Appliances Division. However, the decision was made public after June 30, 2015.

(v) The company has a large warehouse in Lahore which was acquired under a three-year rent agreement signed on April 1, 2014. The agreement is non- cancellable and the company cannot sub-let the warehouse. However, due to operational difficulties, the company shifted the warehouse to a new location.

(vi) A 15% cash dividend was declared on July 5, 2015.

Required

Describe how each of the above issue should be dealt with in the financial statements for the year ended June 30, 2010. Support your point of view in the light of relevant International Accounting Standards. (15)

Page 82: CAF7-Financial Accounting and Reporting II_Questionbank

Financial accounting and reporting II

© Emile Woolf International 72 The Institute of Chartered Accountants of Pakistan

10.6 AKBER CHEMICALS LIMITED

Akber Chemicals Limited is engaged in the business of manufacture and sale of different type of chemicals. The following transactions have not yet been incorporated in the financial statements for the year ended June 30, 2015:

(a) On June 15, 2015, one of its tankers carrying chemicals fell into a canal, thus polluting the water. The company has never faced such a situation before. The company has neither any legal obligation to clean the canal nor does it have any published environmental policy. In a meeting held on July 26, 2015 the Board of Directors decided to clean the canal, which is estimated to cost Rs. 5.5 million.

(b) During the second week of July 2015, a significant decline in the demand for company’s products was observed which also led to a decrease in net realizable value of finished goods. It was estimated that goods costing Rs. 25 million as at June 30, 2015 would only fetch Rs. 23 million.

(c) On June 21, 2015, a customer lodged a claim of Rs. 2 million with the company as a consignment dispatched on June 1, 2015 was not according to the agreed specifications. The company’s inspection team found that this defect arose because of inferior quality of raw materials supplied by the vendor. On June 28, 2015, the company lodged a claim for damages of Rs. 5.0 million, with its vendor, which include reimbursement of the cost of raw materials. The company anticipates that it will have to pay compensation to its customer and would be able to recover 50% of the amount claimed from the vendor.

Required

Discuss how Akber Chemicals (Pvt.) Limited would deal with the above situations in its financial statements for the year ended June 30, 2015. Explain your point of view with reference to the guidance contained in the International Financial Reporting Standards. (13)

10.7 QALLAT INDUSTRIES LIMITED

The following information pertains to Qallat Industries Limited (QIL) for its financial year ended June 30, 2015:

(i) QIL sells all its products on one-year warranty which covers all types of defects. Previous history indicates that 2% of the products contain major defects whereas 10% have minor defects. It is estimated that if major defects were detected in all the products sold, repair cost of Rs. 150 million would result. If minor defects were detected in all products sold, repair cost of Rs. 70 million would result. Total sales for the year are amounted to Rs. 830 million.

(ii) QIL has two large warehouses, A and B. These were acquired under non-cancellable lease agreements. Details are as follows:

Warehouse A Warehouse B

Effective date of agreement July 1, 2010 January 1, 2013

Lease period 10 years 8 years

Rental amount per month Rs. 450,000 Rs. 300,000

Page 83: CAF7-Financial Accounting and Reporting II_Questionbank

Questions

© Emile Woolf International 73 The Institute of Chartered Accountants of Pakistan

On account of serious operating difficulties, QIL vacated both the warehouses on January 1, 2015 and moved to a warehouse situated close to its factory. On the same day QIL sub-let Warehouse A at Rs. 250,000 per month for the remaining lease period. Warehouse B was sub-let on March 1, 2015 for Rs. 350,000 per month for the remaining lease period.

(iii) On July 18, 2015, QIL was sued by an employee claiming damages for Rs. 6 million on account of an injury caused to him due to alleged violation of safety regulations on the part of the company, while he was working on the machine on June 15, 2015. Before filing the suit, he contacted the management on June 29, 2015 and asked for compensation of Rs. 4 million which was turned down by the management. The lawyer of the company anticipates that the court may award compensation ranging between Rs. 1.5 million to Rs. 3 million. However, in his view the most probable amount is Rs. 2 million.

(iv) On November 1, 2014 a new law was introduced requiring all factories to install specialised safety equipment within four months. The Equipment costing Rs. 5.0 million was ordered on December 15, 2014 against 100% advance payment but the supplier delayed installation to July 31, 2015. On August 5, 2015 the company received a notice from the authorities levying a penalty of Rs. 0.4 million i.e. Rs. 0.1 million for each month during which the violation continued. QIL has lodged a claim for recovery of the penalty from the supplier of the equipment.

Required

Describe how each of the above issues should be dealt with in the financial statements for the year ended June 30, 2015. Support your answer in the light of relevant International Accounting Standards and quantify the effect where possible. (14)

10.8 SKYLINE LIMITED

The following information pertains to Skyline Limited (SL) for the financial year ended December 31, 2015:

(i) A customer who owed Rs. 1 million was declared bankrupt after his warehouse was destroyed by fire on February 10, 2016. It is expected that the customer would be able to recover 50% of the loss from the insurance company.

(ii) An employee of SL forged the signatures of directors and made cash withdrawals of Rs. 7.5 million from the bank. Of these, Rs. 1.5 million were withdrawn before December 31, 2015. Investigations revealed that an employee of the bank was also involved and therefore, under a settlement arrangement, the bank paid 60% of the amount to SL on January 27, 2016.

(iii) SL has filed a claim against one of its vendors for supplying defective goods. SL’s legal consultant is confident that damages of Rs. 1 million would be paid to SL. The supplier has already reimbursed the actual cost of the defective goods.

(iv) A suit for infringement of patents, seeking damages of Rs. 2 million, was filed by a third party. SL’s legal consultant is of the opinion that an unfavourable outcome is most likely. On the basis of past experience he has advised that there is 60% probability that the amount of damages would be Rs. 1 million and 40% likelihood that the amount would be Rs. 1.5 million.

Required

Advise SL about the amount of provision that should be incorporated and the disclosures that are required to be made in the financial statements for the year ended December 31, 2015. (16)

Page 84: CAF7-Financial Accounting and Reporting II_Questionbank

Financial accounting and reporting II

© Emile Woolf International 74 The Institute of Chartered Accountants of Pakistan

10.9 WALNUT LIMITED

Walnut Limited (WL) is engaged in the business of import and distribution of electronic appliances.

The following events took place subsequent to the reporting period i.e. 31 December 2015:

(i) On 15 January 2016, one of WL’s competitors announced launching of an upgraded version of DVD players. WL’s inventories include a large stock of existing version of DVD players which are valued at Rs. 15 million. Because of the introduction of the upgraded version, the net realizable value of the existing version in WL’s inventory at 31 December 2015 has reduced to Rs. 12.5 million.

(ii) On 20 December 2015, the board of directors decided to close down the division which imports and sells mobile sets. This decision was made public on 29 December 2015. However, the business was actually closed on 29 February 2016.

Net costs incurred in connection with the closure of this division were as follows:

Rs. m Redundancy costs 1.50 Staff training 0.15 Operating loss from 1 July 2015 to closure of division 0.80 Less: Profit on sale of remaining mobile sets (0.50)

1.95

(iii) On 16 January 2016, LED TV sets valuing Rs. 3 million were stolen from a warehouse. These sets were included in WL’s inventory as at 31 December 2015.

(iv) WL owns 9,000 shares of a listed company whose price as on 31 December 2015 was Rs. 22 per share. During February 2016, the share price declined significantly after the government announced a new legislation which would adversely affect the company’s operations. No provision in this regard has been made in the draft financial statements.

(v) On 31 January 2016, a customer announced voluntary liquidation. On 31 December 2015, this customer owed Rs. 1.5 million.

(vi) On 15 February 2016, WL announced final dividend for the year ended 31 December 2015 comprising 20% cash dividend and 10% bonus shares, for its ordinary shareholders.

Required

Describe how each of the above transactions should be accounted for in the financial statements of Walnut Limited for the year ended 31 December 2015. Support your answer in the light of relevant International Financial Reporting Standards. (16)

Page 85: CAF7-Financial Accounting and Reporting II_Questionbank

Questions

© Emile Woolf International 75 The Institute of Chartered Accountants of Pakistan

10.10 ATTOCK TECHNOLOGIES LIMITED

Attock Technologies Limited (ATL) manufactures five hi-tech products, each on a different plant. It is in the process of preparing its financial statements for the year ended June 30, 2015. As the CFO of the company, the following matters are under your consideration:

(i) Inventory carried at Rs. 25 million on June 30, 2015 was sold for Rs. 15 million after it had been damaged in a flood, in July 2015.

(ii) On July 5, 2015 one of ATL’s corporate customers declared bankruptcy. The liquidator announced on August 25, 2015 that 20% of the debt would be paid on liquidation.

(iii) A new product introduced by a competitor on August 1, 2015 had caused a significant decline in the market demand of one of ATL’s major products. As a result, ATL is considering a reduction in price and a cut in production.

(iv) On August 18, 2015 the government announced a retrospective increase in the tax rate applicable to the company.

(v) The directors of ATL declared a dividend of Rs. 3 per share on August 28, 2015.

Required

State how the above events should be treated in ATL’s financial statements for the year ended June 30, 2015. You may assume that all the above events are material to the company. (11)

Page 86: CAF7-Financial Accounting and Reporting II_Questionbank

Financial accounting and reporting II

© Emile Woolf International 76 The Institute of Chartered Accountants of Pakistan

CHAPTER 11 – IAS 8: ACCOUNTING POLICIES, CHANGES IN ACCOUNTING

ESTIMATES AND ERRORS

11.1 WONDER LIMITED

Wonder Limited (WL) is engaged in the manufacturing and sale of textile machinery. Following are the draft extracts of the statement of financial position and the statement of profit or loss for the year ended 30 June 2015:

Statement of Financial Position

2015 2014

Rs. m Rs. m

Property, plant and equipment 189 130

Retained earnings 166 108

Deferred tax liability 45 27

Statement of profit or loss

2015 2014

Rs. m Rs. m

Profit before taxation 90 120 Taxation 32 42

Profit after taxation 58 78

Following additional information has not been taken into account in the preparation of the above financial statements:

(i) Cost of repairs amounting to Rs. 20 million was erroneously debited to the machinery account on 1 October 2013. The estimated useful life of the machine is 10 years.

(ii) On 1 July 2014, WL reviewed the estimated useful life of its plant and revised it from 5 years to 8 years. The plant was purchased on 1 July 2013 at a cost of Rs. 70 million.

Depreciation is provided under the straight line method. Applicable tax rate is 30%.

Required

Prepare relevant extracts (including comparative figures) for the year ended 30 June 2015 related to the following:

(a) Statement of financial position

(b) Statement of profit or loss

(c) Statement of changes in equity

(d) Correction of error note (20)

Page 87: CAF7-Financial Accounting and Reporting II_Questionbank

Questions

© Emile Woolf International 77 The Institute of Chartered Accountants of Pakistan

11.2 DUNCAN

Duncan Company has previously written off any expenditure on borrowing costs in the period in which it was incurred.

The company has appointed new auditors this year. They have expressed the view that the previous recognition of borrowing costs in the statement of profit or loss was in error. The company has decided to correct the error retrospectively in accordance with IAS 8.

The financial statements for 2014 and the 2015 draft financial statements, both reflecting the old policy, show the following.

Statement of changes in equity (extract)

2014 2015

Retained earnings

Retained earnings

Rs. 000 Rs. 000

Opening balance 22,500 23,950

Profit after tax for the period 3,200 4,712

Dividends paid (1,750) (2,500) ––––––– –––––––

Closing balance 23,950 26,162 ═════ ═════

Borrowing costs written off were Rs. 500,000 in 2014 and Rs. 600,000 in 2015.

The directors have calculated that borrowing costs, net of depreciation which should have been included in property, plant and equipment had the correct policy been applied, are as follows.

Rs. 000

At 30 December 2013 400

At 31 December 2014 450

At 31 December 2015 180

Had the correct policy been in force depreciation of Rs. 450,000 would have been charged in 2014 and Rs. 870,000 in 2015.

Required

Show how the change in accounting policy must be reflected in the statement of changes in equity for the year ended 31 December 2015. Work to the nearest Rs. 000.

Page 88: CAF7-Financial Accounting and Reporting II_Questionbank

Financial accounting and reporting II

© Emile Woolf International 78 The Institute of Chartered Accountants of Pakistan

11.3 MOHANI MANUFACTURING LIMITED

Mohani Manufacturing Limited is engaged in manufacturing of spare parts for motor car assemblers. The audited financial statements for the year ended December 31, 2014 disclosed that the profit and retained earnings were Rs. 21 million and Rs. 89 million respectively. The draft financial statements for the year show a profit of Rs. 15 million. However, following adjustments are required to be made:

(i) The management of the company has decided to change the method for valuation of raw materials from FIFO to weighted average. The value of inventory under each method is as follows:

FIFO Weighted Average

Rs. m Rs. m

December 31, 2013 37.0 35.5

December 31, 2014 42.3 44.5

December 31, 2015 58.4 54.4

(ii) In 2014, the company purchased a plant for Rs. 100 million. Depreciation on plant was recorded at Rs. 25 million instead of Rs. 10 million. This error was discovered after the publication of financial statements for the year ended December 31, 2014. The error is considered to be material.

Required

Produce an extract showing the movement in retained earnings, as would appear in the statement of changes in equity for the year ended December 31, 2015. (11)

Page 89: CAF7-Financial Accounting and Reporting II_Questionbank

Questions

© Emile Woolf International 79 The Institute of Chartered Accountants of Pakistan

CHAPTER 12 – IAS 12: INCOME TAXES

12.1 FRANCESCA

On 30 June 2014 Francesca Company had a credit balance on its deferred tax account of Rs. 1,340,600 all in respect of the difference between depreciation and capital allowances. During the year ended 30 June 2015 the following transactions took place.

(1) Rs. 45 million was charged against profit in respect of depreciation. The tax computation showed capital allowances of Rs. 50 million.

(2) Interest receivable of Rs. 50,000 was reflected in profit for the period. However, only Rs. 45,000 of interest was actually received during the year. Interest is not taxed until it is received.

(3) Interest payable of Rs. 32,000 was treated as an expense for the period. However, only Rs. 28,000 of interest was actually paid during the year. Interest is not an allowable expense for tax purposes until it is paid.

(4) During the year Francesca incurred development costs of Rs. 500,600, which it has capitalised. Development costs are an allowable expense for tax purposes in the period in which they are paid.

(5) Land and buildings with a net book value of Rs. 4,900,500 were revalued to Rs. 6 million.

The tax rate is 30%. Francesca has a right of offset between its deferred tax liabilities and its deferred tax assets.

Required

Calculate the deferred tax liability on 30 June 2015. Show where the increase or decrease in the liability in the year would be charged or credited.

12.2 SHEP (I)

Shep was incorporated on 1 January 2015. In the year ended 31 December 2015 the company made a profit before taxation of Rs. 121,000

During the period Shep made the following capital additions.

Rs.

Plant 48,000

Motor vehicles 12,000

During the period:

Accounting depreciation 11,000

Tax depreciation 15,000

Tax is chargeable at a rate of 30%.

Required

(a) Calculate the corporate income tax liability for the year ended 31st December 2015.

(b) Calculate the deferred tax balance that is required in the statement of financial position as at 31st December 2015.

(c) Prepare a note showing the movement on the deferred tax account and thus calculate the deferred tax charge for the year ended 31st December 2015

(d) Prepare the statement of profit or loss note which shows the compilation of the tax expense for the year ended 31st December 2015.

Page 90: CAF7-Financial Accounting and Reporting II_Questionbank

Financial accounting and reporting II

© Emile Woolf International 80 The Institute of Chartered Accountants of Pakistan

12.3 SHEP (II)

Continuing from the previous year. The following information is relevant for the year ended 31st December 2016.

(a) Capital transactions

Rs.

Depreciation charged 14,000 Tax allowances 16,000

(b) Interest payable

On 1st April 2016 the company issued Rs. 25,000 of 8% convertible loan stock. Interest is paid in arrears on 30th September and 30th March. Assume that tax relief on interest expense is only given when the interest is paid.

(c) Interest receivable

On 1st April Shep purchased debentures having a nominal value of Rs. 4,000. Interest at 15% pa is receivable on 30th September and 30th March. Assume that interest income is not taxed until the cash is actually received.

(d) Provision for warranty

In preparing the financial statements for the year to 31st December 2016, Shep has recognised a provision for warranty payments in the amount of Rs. 1,200. This has been correctly recognised in accordance with IAS 37 and the amount has been expensed. Assume that tax relief on the warranty cost is only given when the expense is paid.

(e) Fine

During the period Shep has paid a fine of Rs. 6,000. The fine is not tax deductible.

(f) Further information

The accounting profit before tax for the year was Rs. 125,000.

Tax is chargeable at a rate of 30%.

Required

(a) Calculate the corporate income tax liability for the year ended 31st December 2016.

(b) Calculate the deferred tax balance that is required in the statement of financial position as at 31st December 2016.

(c) Prepare a note showing the movement on the deferred tax account and thus calculate the deferred tax charge for the year ended 31st December 2016

(d) Prepare the statement of profit or loss note which shows the compilation of the tax expense for the year ended 31st December 2016.

(e) Prepare a note to reconcile the product of the accounting profit and the tax rate to the tax expense for year ended 31st December 2016.

Page 91: CAF7-Financial Accounting and Reporting II_Questionbank

Questions

© Emile Woolf International 81 The Institute of Chartered Accountants of Pakistan

12.4 SHEP (III)

Continuing from the previous year. The following information is relevant for the year ended 31st December 2017.

(a) Interest payable/Interest receivable

Shep still has Rs. 25,000 of 8% convertible loan stack in issue and still retains its holding in the debentures purchased in 2004.

(b) Provision for warranty

During the year Shep had paid out Rs. 500 in warranty claims and provided for a further Rs. 2,000.

(d) Development costs

During 2017 Shep has capitalised development expenditure of Rs. 17,800 in accordance with the provisions of IAS 38. Assume that tax relief on this expenditure is taken in full in the period in which it is incurred.

(e) Further information

Rs.

Profit before taxation 175,000 Depreciation charged 18,500 Tax allowable depreciation 24,700

(f) Entertainment

Shep paid for a large office party during 2017 to celebrate a successful first two years of the business. This cost Rs. 20,000. Assume that this expenditure is not tax deductible.

Tax is chargeable at a rate of 30%.

Required

(a) Calculate the corporate income tax liability for the year ended 31st December 2017.

(b) Calculate the deferred tax balance that is required in the statement of financial position as at 31st December 2017.

(c) Prepare a note showing the movement on the deferred tax account and thus calculate the deferred tax charge for the year ended 31st December 2017

(d) Prepare the statement of profit or loss note which shows the compilation of the tax expense for the year ended 31st December 2017.

(e) Prepare a note to reconcile the product of the accounting profit and the tax rate to the tax expense for year ended 31st December 2017.

Page 92: CAF7-Financial Accounting and Reporting II_Questionbank

Financial accounting and reporting II

© Emile Woolf International 82 The Institute of Chartered Accountants of Pakistan

12.5 SHEP (IV)

Using the information provided in “Shep III” and assume that Shep is subject to a higher tax rate of 34% in 2017.

Required

(a) Calculate the corporate income tax liability for the year ended 31st December 2017.

(b) Calculate the deferred tax balance that is required in the statement of financial position as at 31st December 2017.

(c) Prepare a note showing the movement on the deferred tax account and thus calculate the deferred tax charge for the year ended 31st December 2017

(d) Prepare the statement of profit or loss note which shows the compilation of the tax expense for the year ended 31st December 2017.

(e) Prepare a note to reconcile the product of the accounting profit and the tax rate to the tax expense for year ended 31st December 2017.

12.6 WAQAR LIMITED

Waqar Limited has provided you the following information for determining its tax and deferred tax expense for the year 2014 and 2015:

(i) During the year ended December 31, 2015, the company’s accounting profit before tax amounted to Rs. 40 million (2014: Rs. 30 million). The profit includes capital gains amounting to Rs. 10 million (2014: Rs. 8 million) which are exempt from tax.

(ii) The accounting written down values of the fixed assets, as at December 31, 2013 were as follows:

Cost

Accumulated

Depreciation

Written down value

Rs. m Rs. m Rs. m

Machinery 200 25 175

Furniture and fittings 50 10 40

No additions or disposals of fixed assets were made in the years 2014 and 2015.

(iii) Machinery was acquired on January 1, 2013 and is being depreciated on straight- line basis over its estimated useful life of 8 years. The tax base of machinery as at December 31, 2013 was Rs. 90 million.

(iv) Furniture and fittings are also depreciated on the straight line basis at the rate of 10% per annum. The tax base of furniture and fittings as at December 31, 2013 was Rs. 40.5 million.

(v) Normal rate of tax depreciation on both types of assets is 10% on written down value.

(vi) The tax rates for 2013, 2014 and 2015 were 35%, 35% and 30% respectively.

Page 93: CAF7-Financial Accounting and Reporting II_Questionbank

Questions

© Emile Woolf International 83 The Institute of Chartered Accountants of Pakistan

Required

For each year:

(a) Calculate the corporate income tax liability for the year.

(b) Calculate the deferred tax balance that is required in the statement of financial position as at the year end.

(c) Prepare a note showing the movement on the deferred tax account and thus calculate the deferred tax charge for the year.

(d) Prepare the statement of profit or loss note which shows the compilation of the tax expense.

(e) Prepare a note to reconcile the product of the accounting profit and the tax rate to the tax expense. (25)

12.7 SHAKIR INDUSTRIES

Given below is the statement of profit or loss of Shakir Industries for the year ended December 31, 2015:

2015

Rs. m

Sales 143.00

Cost of goods sold (96.60)

Gross profit 46.40

Operating expenses (28.70)

Operating profit Other income 17.70

Profit before interest and tax 3.40

Financial charges 21.10

Profit before tax (5.30)

15.80

Following information is available:

(i) Operating expenses include an amount of Rs. 0.7 million paid as penalty to SECP on non-compliance of certain requirements of the Companies Ordinance, 1984.

(ii) During the year, the company made a provision of Rs. 2.4 million for gratuity. The actual payment on account of gratuity to outgoing members was Rs. 1.6 million.

(iii) Lease payments made during the year amounted to Rs. 0.65 million which include financial charges of Rs. 0.15 million. As at December 31, 2015, obligations against assets subject to finance lease stood at Rs. 1.2 million. The movement in assets held under finance lease is as follows:

Rs. m

Opening balance – 01/01/2015 2.50

Depreciation for the year (0.7)

Closing balance – 31/12/2015 1.80

Page 94: CAF7-Financial Accounting and Reporting II_Questionbank

Financial accounting and reporting II

© Emile Woolf International 84 The Institute of Chartered Accountants of Pakistan

(iv) The details of owned fixed assets are as follows:

Accounting Tax

Rs. m Rs. m

Opening balance – 01/01/2015 12.50 5.30

(1.10)

10.20

5.30

(1.65) Purchased during the year 5.3 5.3

Depreciation for the year (1.1) 5.30

(1.10)

(1.65)

5.30

(1.65) Closing balance – 31/12/2015 16.70 13.85

(v) Capital work-in-progress as on December 31, 2015 include financial charges of Rs. 2.3 million which have been capitalised in accordance with IAS-23 “Borrowing Costs”. However, the entire financial charges are admissible, under the Income Tax Ordinance, 2001.

(vi) Deferred tax liability and provision for gratuity as at January 1, 2015 was Rs. 0.55 million and Rs. 0.7 million respectively.

(vii) Applicable income tax rate is 35%.

Required

Based on the available information, compute the current and deferred tax expenses for the year ended December 31, 2015. (15)

12.8 MARS LIMITED

Mars Limited (ML) is engaged in the manufacturing of chemicals. On July 1, 2014 it obtained a motor vehicle on lease from a bank. Details of the lease agreement are as follows:

(i) Cost of motor vehicle is Rs. 1,600,000.

(ii) Instalments of Rs. 480,000 are to be paid annually in advance.

(iii) The lease term and useful life is 4 years and 5 years respectively.

(iv) The interest rate implicit in the lease is 13.701%.

ML follows a policy of depreciating the motor vehicles over their useful life, on the straight-line method. However, the tax department allows only the lease payments as a deduction from taxable profits.

The tax rate applicable to the company is 30%. ML’s accounting profit before tax for the year ended June 30, 2015 is Rs. 4,900,000.

There are no temporary differences other than those evident from the information provided above.

Required

(a) Prepare journal entries in the books of Mars Limited for the year ended June 30, 2015 to record the above transactions including tax and deferred tax.

(b) Prepare a note to the financial statements related to disclosure of finance lease liability, in accordance with the requirements of IFRS. (18)

(Ignore comparative figures.)

Page 95: CAF7-Financial Accounting and Reporting II_Questionbank

Questions

© Emile Woolf International 85 The Institute of Chartered Accountants of Pakistan

12.9 BILAL ENGINEERING LIMITED

Bilal Engineering Limited earned profit before tax amounting to Rs. 50 million during the year ended December 31, 2015. The accountant of the company has submitted draft accounts to the Finance Manager along with the following information which he believes could be useful in determining the amount of taxation:

(i) Accounting deprecation for the year is Rs. 10 million which includes Rs. 1 million charged on the difference between cost and revalued amount.

(ii) A motor vehicle costing Rs. 1 million was taken on lease in 2014. Related clauses of the lease agreement are as under:

Annual instalment of Rs. 0.3 million is payable annually in advance.

The lease term and useful life is 4 years and 5 years respectively.

The interest rate implicit in the lease is 13.701% per annum.

Accounting depreciation on the leased vehicle is included in the depreciation referred to in para (i) above.

(iii) Tax depreciation on the assets owned by the company is Rs. 7 million.

(iv) Research and development expenses of Rs. 15 million were incurred in 2013 and are being amortised over a period of 15 years. For tax purposes research and development expenses are allowed to be written off in 10 years. However, 10% of these expenses were not verifiable and have not been claimed.

(v) Expenses amounting to Rs. 0.25 million were disallowed in 2012. Out of these Rs. 0.15 million were allowed in appeal, during the current year. The company had initially expected that the full amount would be allowed but has decided not to file a further appeal.

(vi) The applicable tax rate is 35%.

Required

(a) Prepare journal entries in respect of taxation, for the year ended December 31, 2015.

(b) Prepare a reconciliation to explain the relationship between tax expense and accounting profit as is required to be disclosed under IAS 12 Income Taxes. (18)

12.10 GALAXY INTERNATIONAL

The following information relates to Galaxy International (GI), a listed company, which was incorporated on January 1, 2014.

(i) The (loss) / profit before taxation for the years ended December 31, 2014 and 2015 amounted to (Rs. 1.75 million) and Rs. 23.5 million respectively.

(ii) The details of accounting and tax depreciation on fixed assets is as follows:

2015 2014 Rs. m Rs. m Accounting depreciation 15 15 Tax depreciation 6 45

(iii) In 2014, GI accrued certain expenses amounting to Rs. 2 million which were disallowed by the tax authorities. However, these expenses are expected to be allowed on the basis of payment in 2015.

Page 96: CAF7-Financial Accounting and Reporting II_Questionbank

Financial accounting and reporting II

© Emile Woolf International 86 The Institute of Chartered Accountants of Pakistan

(iv) GI earned interest on Special Investment Bonds amounting to Rs. 1.0 million and Rs. 1.25 million in the years 2014 and 2015 respectively. This income is exempt from tax.

(v) GI operates an unfunded gratuity scheme. The provision during the years 2014 and 2015 amounted to Rs. 1.7 million and Rs. 2.2 million respectively. No payment has so far been made on account of gratuity.

(vi) The applicable tax rate is 35%.

Required

Prepare a note on taxation for inclusion in the company’s financial statements for the year ended December 31, 2015 giving appropriate disclosures relating to current and deferred tax expenses including a reconciliation to explain the relationship between tax expense and accounting profit. (20)

12.11 APRICOT LIMITED

The following information relates to Apricot Limited (AL), a listed company, for the financial year ended 31 December 2015:

(i) The profit before tax for the year amounted to Rs. 60 million (2014: Rs. 45 million).

(ii) The accounting and tax written down value of fixed assets as on 31 December 2014 was Rs. 95 million and Rs. 90 million respectively. Accounting depreciation for the year is Rs. 10 million (2014: Rs. 9 million) whereas tax depreciation for the year is Rs. 8 million (2014: Rs. 7 million).

(iii) During the year, AL sold a machine for Rs. 3 million and recognised a profit of Rs. 0.5 million. The tax written down value of the machine as on 31 December 2014 was Rs. 2 million. There were no other additions/disposals of fixed assets in 2014 and 2015.

(iv) AL earned capital gain of Rs. 6 million (2014:Nil) on sale of shares of a listed company. This income is exempt from tax.

(v) Bad debt expenses recognised during the year was Rs. 5 million (2014: Rs. 7 million).

(vi) Bad debts written off during the year amounted to Rs. 3 million (2014: Rs. 4 million).

(vii) Deferred tax liability and provision for bad debts as on 31 December 2011 was Rs. 18.90 million and Rs. 9 million respectively.

(viii) The company’s assessed brought forward losses up to 31 December 2011 amounted to Rs. 19.25 million.

(ix) Applicable tax rate is 35%.

Required

Prepare a note on taxation for inclusion in AL’s financial statements for the year ended 31 December 2015 giving appropriate disclosures relating to current and deferred tax expenses including comparative figures for 2014 and a reconciliation to explain the relationship between 2015 tax expense and 2015 accounting profit. (21)

Page 97: CAF7-Financial Accounting and Reporting II_Questionbank

Questions

© Emile Woolf International 87 The Institute of Chartered Accountants of Pakistan

CHAPTER 13 – RATIO ANALYSIS

13.1 WASIM

Wasim is an importer and retailer of vegetable oils. Extracts from the financial statements for this year and last are set out below.

Income statements for the years ended 30 September

Year 7 Year 6

Rs.000 Rs.000 Revenue

2,160 1,806

Cost of sales (1,755) (1,444) –––––– –––––– Gross profit 405 362 Distribution costs (130) (108) Administrative expenses (260) (198) –––––– –––––– Profit before tax 15 56 Income tax expense (6) (3) –––––– –––––– Profit for the period 9 53 –––––– ––––––

Statements of financial position as of 30 September

Year 7 Year 6

Rs.‘000 Rs.‘000 Assets Non-current assets Property, plant and equipment 78 72 Current assets Inventories 106 61 Trade receivables 316 198 Cash - 6 ––––– –––––

422 265 ––––– ––––– Total assets 500 337 ––––– ––––– Equity and liabilities Equity Ordinaryshares 110 85 Preference shares 23 11 Share premium 15 - Revaluation reserve 20 20 Retained earnings 78 74

246 190 Current liabilities Bank overdraft 49 - Trade payables 198 142 Current tax payable 7 5 ––––– –––––

254 147 ––––– ––––– Total equity and liabilities 500 337 ––––– –––––

Page 98: CAF7-Financial Accounting and Reporting II_Questionbank

Financial accounting and reporting II

© Emile Woolf International 88 The Institute of Chartered Accountants of Pakistan

Required

Define and calculate the following ratios:

a) Gross profit percentage.

b) Net profit percentage

c) Return on capital employed

d) Asset turnover

e) Current ratio

f) Quick ratio

g) Average receivables collection period

h) Average payables period

i) Inventory turnover

13.2 AMIR AND MO

The income statements and statements of financial position of two manufacturing companies in the same sector are set out below.

Amir Mo

Rs. Rs.

Revenue 150,000 700,000

Cost of sales (60,000) (210,000) –––––––– ––––––––

Gross profit 90,000 490,000

Interest payable (500) (12,000)

Distribution costs (13,000) (72,000)

Administrative expenses (15,000) (35,000) –––––––– ––––––––

Profit before tax 61,500 371,000

Income tax expense (16,605) (100,170) –––––––– ––––––––

Profit for the period 44,895 270,830 –––––––– ––––––––

Assets

Non-current assets

Property - 500,000

Plant and equipment 190,000 280,000

––––––– –––––––

190,000 780,000

Current assets

Inventories 12,000 26,250

Trade receivables 37,500 105,000

Cash at bank 500 22,000 ––––––– –––––––

50,000 153,250

––––––– –––––––

Total assets 240,000 933,250 ––––––– –––––––

Page 99: CAF7-Financial Accounting and Reporting II_Questionbank

Questions

© Emile Woolf International 89 The Institute of Chartered Accountants of Pakistan

Equity and liabilities

Equity

Share capital 156,000 174,750

Retained earnings 51,395 390,830 ––––––– –––––––

207,395 565,580

Non-current liabilities

Long-term debt 10,000 250,000

Current liabilities

Trade payables 22,605 117,670 ––––––– –––––––

Total equity and liabilities 240,000 933,250 ––––––– –––––––

Required

Define and calculate the following ratios for each company:

a) Gross profit percentage.

b) Net profit percentage

c) Return on capital employed

d) Asset turnover

e) Current ratio

f) Quick ratio

g) Average receivables collection period

h) Average payables period

i) Inventory turnover

Page 100: CAF7-Financial Accounting and Reporting II_Questionbank

Financial accounting and reporting II

© Emile Woolf International 90 The Institute of Chartered Accountants of Pakistan

CHAPTER 14 – ETHICAL ISSUES IN FINANCIAL REPORTING

14.1 ETHICAL ISSUES

Waheed is a chartered accountant, recently employed by AA plc as deputy to the finance director, Arif (also a chartered accountant). AA plc is listed on the Lahore stock exchange.

On Waheed’s first day on the job he met with Arif who said ‘Look, keep it to yourself but I’m having a second interview next week for a new job. The first thing that I need you to do is to review the financial statements before the auditors arrive. I qualified a few years ago and am not up to date on all of the little technicalities in IFRS. You should now these better than me and you’ll know more about what the auditors might focus on. We must do our best to present the financial statements in the most favourable light as the bonus paid to employees (including me) depends on profit being more than 10% bigger than last year’s and remember that you qualify for this too. Keep this in mind when you carry out the review as we do not really want to find anything. Do well at this and I might put in a good word for you when I leave as I’m sure you’ll be a great replacement for me.”

Required

Explain the ethical issues inherent in the above conversation and what Waheed should do about them.

14.2 SINDH INDUSTRIES LTD

Jafar has recently been appointed as financial controller to Sindh Industries Ltd. Until a month ago, Sindh Industries had a finance director, who resigned suddenly, due to ill health. Since Jafar joined the company, he has learned that his resignation was related to stress caused by a series of disagreements with the managing director about the performance of the business.. The directors have not yet appointed a replacement.

It is now March 2016 and you have been asked to finalise the financial statements for the year ended 31 December 2015. The draft statement of profit or loss extract and statement of financial position are shown below:

Draft statement of profit or loss for the year ended 31 December 2015

Rs. 000

Profit before tax 2,500

Draft statement of financial position at 31 December 2015

Rs. 000 Property, plant and equipment 12,000 Current assets 3,500

Total assets 15,500

Share capital 2,000 Retained earnings 6,000

Equity 8,000 Non-current liabilities 5,000 Current liabilities 2,500

Total equity and liabilities 15,500

Page 101: CAF7-Financial Accounting and Reporting II_Questionbank

Questions

© Emile Woolf International 91 The Institute of Chartered Accountants of Pakistan

During the year ended 31 December 2015 Sindh Industries entered into the following transactions.

(1) Just before the year end Sindh Industries signed a contract to deliver consultancy services for a period of 2 years at a fee of Rs. 500,000 per annum. The full amount of this fee has been paid in advance and is non-refundable.

(2) Sindh Industries has constructed a new factory. The construction has been financed from the pool of existing borrowings. Land at a cost of Rs. 1.8 million was acquired on 1 February 2015 and construction began on 1 June 2015. Construction was completed on 30 September 2015 at an additional cost of Rs. 2.7 million. Although the factory was usable from that date, full production did not commence until 1 December 2015. Throughout the year the company’s average borrowings were as follows:

Annual interest Amount rate Rs. % Bank overdraft 1,000,000 9.75 Bank loan 1,750,000 10 Debenture 2,500,000 8

An amount of Rs. 450,000 has been included in property, plant and equipment in respect of borrowing costs relating to the construction of the factory. The useful life of the factory has been estimated at 20 years. No depreciation has been charged for the year. The reason for this is that the factory has only been in use for one month and that the depreciation charge would be immaterial.

(3) A blast furnace with a carrying amount at 1 January 2015 of Rs. 3.5 million has been depreciated in the draft financial statements on the basis of a remaining life of 20 years. In December 2015 the directors carried out a review of the useful lives of various significant items of plant and machinery, including the blast furnace and came to the conclusion that the useful life of the furnace was 20 years at 31 December 2015. The reasoning behind this judgement was that the lining of the furnace had been replaced in the last week of December 20X6 at a cost of Rs. 1.4 million. Provided that the lining is replaced every five years, the life of the furnace can be extended accordingly. You have found a report, commissioned by the previous finance director and prepared by a firm of asset valuation specialists, which assesses the remaining useful life of the main structure of the furnace at 1 January 2015 at 15 years and the lining of the furnace at 5 years. You have also found evidence that the managing director has seen this report.

Jafar has had a conversation with the managing director who told him, “We need to make the figures look as good as possible so I hope you’re not going to start being difficult. The consultancy fee is non-refundable so there’s no reason why we can’t include it in full. I think we should look at our depreciation policies. We’re writing off our assets over far too short a period. As you know, we’re planning to go for a stock market listing in the near future and being prudent and playing safe won’t help us do that. It won’t help your future with this company either.”

Page 102: CAF7-Financial Accounting and Reporting II_Questionbank

Financial accounting and reporting II

© Emile Woolf International 92 The Institute of Chartered Accountants of Pakistan

Required

(a) Explain the required IFRS accounting treatment of these issues, preparing relevant calculations where appropriate. (17)

(b) Prepare a revised draft of the statement of profit or loss extract for the year ended 31 December 2015 and the statement of financial position at that date. (6)

(c) Discuss the ethical issues arising from your review of the draft financial statements and the actions that you should consider. (5)

Page 103: CAF7-Financial Accounting and Reporting II_Questionbank

© Emile Woolf International 93 The Institute of Chartered Accountants of Pakistan

Certificate in Accounting and Finance Financial accounting and reporting II

S E

C T

I O

N

B

Answers

Page 104: CAF7-Financial Accounting and Reporting II_Questionbank

Financial accounting and reporting II

© Emile Woolf International 94 The Institute of Chartered Accountants of Pakistan

CHAPTER 2 – IAS 1: PRESENTATION OF FINANCIAL STATEMENTS

2.1 LARRY

Statement of profit or loss For the year ended 31 December 2015 Rs. in

million

Revenue 3,304 Cost of sales (2,542 + 118 – 127) (2,533)

Gross profit 771 Other income 20 Distribution costs (175) Administrative expenses (342)

Profit before tax 274 Income tax expense (75)

Profit for the period 199

Statement of financial position As at 31 December 2015

Assets Rs. in million

Non-current assets Property, plant and equipment (2,830 – 918) 1,912 Intangible assets (26 – 5) 21

1,933 Current assets

Inventories 127 Trade and other receivables 189 Cash (89 +2) 91

407

Total assets 2,340

Equity and liabilities Equity

Share capital 400 Retained earnings (1,562 + 199) 1,761

2,161 Non-current liabilities Long-term borrowings (18 x 2/3) 12 Current liabilities

Trade and other payables 86 Current portion of long-term borrowing (18 ÷ 3) 6 Current tax payable 75

167

Total equity and liabilities 2,340

Page 105: CAF7-Financial Accounting and Reporting II_Questionbank

Answers

© Emile Woolf International 95 The Institute of Chartered Accountants of Pakistan

2.2 MINGORA IMPORTS LIMITED

Statement of profit or loss for the year ended 31 December 2015

Rs. in million

Revenue 1,740 Change in inventories of finished goods and work-in-progress (W3) 40 Staff costs (W3) (620) Depreciation and other amortisation expense (W3) (42) Other expenses (W3) (359)

Profit before tax 759 Income tax expense (120)

Profit for the period 639

Statement of financial position as at 31 December 2015

Assets Rs. in million

Non-current assets Property, plant and equipment (W1) 368 Intangible assets (W2) 40

408 Current assets

Inventories (180 + 140) 320 Trade and other receivables (420 x 95%) 399 Cash 440

1,159

Total assets 1,567

Equity and liabilities Equity

Share capital 600 Other reserves 120 Retaind earnings 635

1,355 Current liabilities

Trade and other payables 92 Current tax payable 120

212

Total equity and liabilities 1,567

Statement of changes in equity for the year ended 31 December 2015

Amounts in Rs. million

Share capital

Revaluation

reserve

Retained earnings

Total

Balance at 31 December 2014 620 121 721 Dividends paid (125) (125) Net revaluation surplus in the year (360 – (300 – 60))

120 - 120

Profit after tax for the period - - 639 639

Balance at 31 December 2015 620 120 635 1,355

Page 106: CAF7-Financial Accounting and Reporting II_Questionbank

Financial accounting and reporting II

© Emile Woolf International 96 The Institute of Chartered Accountants of Pakistan

Workings

(1) Property, plant and equipment

Rs. in million

Cost brought forward Leasehold 300 Computers 50

Revaluation 60

Cost carried forward 410

Accumulated depreciation brought forward (60 + 20) 80 Revaluation (60) Charge for the year

Leasehold (360 ÷ 30) 12 Computers (50 ÷ 5) 10

Accumulated depreciation carried forward 42

Carrying amount carried forward 368

(2) Intangible assets

Rs. in million

Cost 60 Amortisation (60 ÷ 3) (20)

Carried forward 40

(3) Allocation of costs

Amounts in Rs. million

Change in

inventories

Staff costs

Depreciation etc

Other expenses

Work-in-progress (140 – 125)

(15)

Staff costs 260 Finished goods (180 – 155)

(25)

Consultancy fees 44 Directors’ salaries 360

Doubtful receivables (420 5%)

21

Sundry 294 Amortisation of patent (W2)

20

Depreciation (12 + 10) (W1)

22

(40) 620 42 359

Page 107: CAF7-Financial Accounting and Reporting II_Questionbank

Answers

© Emile Woolf International 97 The Institute of Chartered Accountants of Pakistan

2.3 BARRY

Barry Statement of profit or loss For the year ended 31st August 2015

Rs. in million

Revenue 30,000

Cost of sales (W1) (19,650)

Gross profit 10,350

Distribution costs (W1) (1,370)

Administrative expenses (W1) (1,930)

Profit from operations 7,050

Finance costs (350)

Profit before tax 6,700

Tax (W2) (2,500)

Profit after tax 4,200

Barry Statement of financial position As at 31st August 2015 Rs. in

million

ASSETS

Non-current assets

Property, plant and equipment 39,600

Current assets

Inventory 4,600

Trade and other receivables (7,400 + 200) 7,600

Cash and cash equivalents 700

12,900

Total assets 52,500

EQUITY AND LIABILITIES

Capital and reserves

Equity shares 21,000

Share premium 2,000

Accumulated profits (W3) 11,800

Total equity 34,800

Revaluation reserve (W4) 4,700

Non current liabilities

Borrowings 5,200

Current liabilities

Trade and other payables 5,300

Taxation (2,100 + 400) 2,500

7,800

Total equity and liabilities 52,500

Page 108: CAF7-Financial Accounting and Reporting II_Questionbank

Financial accounting and reporting II

© Emile Woolf International 98 The Institute of Chartered Accountants of Pakistan

Reconciliation of opening and closing property, plant and equipment

Rs. in ‘000

Land Buildings Plant &

machinery

Fixtures &

fittings

Assets under

construction

Total

Cost/ Valuation

At 1 Sept 2014 10,000 9,000 20,100 10,000 400 49,500

Additions - - - - 50 50

Reclassification - - 450 - (450) -

Revaluation 1,000 1,000 - - - 2,000

At 31 Aug 2015 11,000 10,000 20,550 10,000 - 51,550

Depreciation

At 1 Sept 2014 - 3,000 4,000 3,700 - 10,700

Revaluation - (3,000) - - - (3,000)

Charge for year - 1,000 2,550 700 - 4,250

At 31 Aug 2015 - 1,000 6,550 4,400 - 11,950

Net book value

At 31 Aug 2015 11,000 9,000 14,000 5,600 - 39,600

At 1 Sept 2014 10,000 6,000 16,100 6,300 400 38,800

Workings

1 Allocation of expenses Rs.in ‘000

Cost of sales

Admin Distrib

Raw materials consumed 9,500

Manufacturing overheads 5,000

Increase in inventories (1,400)

Staff costs (70%/20%/10%) 3,290 940 470

Distribution costs 900

Depreciation

Building (50%/50%) 500 500

Plant and machinery 2,550

Fixtures and fittings (30%/70%) 210 490

19,650 1,930 1,370

2 Tax charge

Rs. in ‘000

Current year 2,100

Under provision from previous year 400

2,500

Page 109: CAF7-Financial Accounting and Reporting II_Questionbank

Answers

© Emile Woolf International 99 The Institute of Chartered Accountants of Pakistan

3 Accumulated profits carried forward Rs. in ‘000

Accumulated profits carried forward per question 14,000

Less tax charge

- Current year estimate 2,100

- Under-provision in previous year 400

(2,500)

Add transfer of excess depreciation on revalued building 300

11,800

4 Revaluation reserve carried forward

Revaluation reserve per question 5,000

Add transfer of excess depreciation on revalued building (300)

4,700

2.4 OSCAR INC

(a) Statement of profit or loss For the year ended 31 March 2015

Rs. in ‘000

Sales 2,010 Operating costs (140 + 960 – 150 + 420 + 210 + 16) (1,596) ———— Operating profit before interest 414 Income from investments 75 ———— Profit before taxation 489 Income tax (49) ———— 440 ———— Statement of financial position

As at 31 March 2015 Assets Non-current assets Tangible assets 530 Investments 560

———— 1,090 Current assets Inventory 150 Receivables 470

———— 620

———— 1,710

———— Equity and liabilities Capital and reserves Share capital 600 Retained earnings 500

———— 1,100 Current liabilities 414 Provisions for liabilities and charges 196

———— 1,710

————

Page 110: CAF7-Financial Accounting and Reporting II_Questionbank

Financial accounting and reporting II

© Emile Woolf International 100 The Institute of Chartered Accountants of Pakistan

Workings (1) Income tax Rs. in ‘000

Income tax (current year) 74 Over provision for tax in the previous year (25) —— 49 —— (2) Tangible assets – plant and machinery Rs. in ‘000

Cost at 1 April 2014 and 31 March 2015 750 —— Accumulated depreciation At 31 March 2014 188 Provided during the year (27 + 5) 32 —— At 31 March 2015 220 ——

Net book value at 31 March 2015 530 ——

(3) Current liabilities Rs. in ‘000

Trade payables 260 Mainstream corporation tax 74 Bank overdraft 80 —— 414 —— (4) Provisions for liabilities and charges Rs. in ‘000 At 1 April 2014 180 Provided in the year 16 —— At 31 March 2015 196 —— (5) Retained earnings Rs. in ‘000

Retained earnings 440 Opening retained earnings 180 Dividends (120) ——— Closing retained earnings 500 ———

Page 111: CAF7-Financial Accounting and Reporting II_Questionbank

Answers

© Emile Woolf International 101 The Institute of Chartered Accountants of Pakistan

2.5 CLIFTON PHARMA LIMITED

(a) Clifton Pharma Limited

Statement of profit or loss for the year ended 30 September 2015

Rs. in ‘000

Revenue 338,300 Cost of sales: see working (1) (180,000)

Gross profit 158,300 Operating expenses: see working (2) (36,600) Investment income 2,000

Finance costs: Loan notes – see working (3) (3,000) Finance lease – see working (2) (1,700)

(4,700)

Profit before tax 119,000 Income tax expense: see working (4) (21,000)

Profit for the period 98,000

(b) Clifton Pharma Limited

Statement of financial position as at 30 September 2015

Non-current assets

Property, plant and equipment: see working (5) 358,000 Investments 92,400

450,400 Current assets

Inventory 23,700 Trade receivables 76,400 Bank 12,100

112,200

Total assets 562,600

Equity and liabilities Capital and reserves Share capital 280,000 Share premium 20,000 Retained earnings: see working (6) 117,300

417,300 Revaluation surplus 20,000 Non-current liabilities

3% loan notes: see working (3) 51,500 Deferred tax: see working (4) 23,000 Finance lease obligation: see working (2) 11,700

86,200 Current liabilities

Trade payables 14,100 Accrued lease finance costs: see working (2) 1,700 Finance lease obligation: see working (2) 5,300 Income tax payable 18,000

39,100

Total equity and liabilities 562,600

Page 112: CAF7-Financial Accounting and Reporting II_Questionbank

Financial accounting and reporting II

© Emile Woolf International 102 The Institute of Chartered Accountants of Pakistan

Workings

(1) Cost of sales Rs. in ‘000

As given in the trial balance 134,000

Depreciation of plant and equipment: 20% (197,000 – 47,000)

30,000

Depreciation of leased vehicles: 24,000/4 years 6,000

Amortisation of leasehold property: 250,000/25 years 10,000

180,000

(2) Vehicle rentals and finance lease. Operating expenses

Rental costs given in the trial balance 8,600

Relating to finance lease (7,000)

Balance: relating to operating lease – operating expense 1,600

Other operating expenses (trial balance in question) 35,000

Total operating expenses 36,600

Finance lease

Fair value of leased assets 24,000

Less: First rental payment, paid in advance 1 October 2014

(7,000)

Remaining obligation, 1 October 2014 17,000

Interest at 10% to 30 September 2015 (current liability) 1,700

Lease payment due 1 October 2015 7,000

Capital repayment due (= balance, current liability) (5,300)

Remaining lease obligation = non-current liability 11,700

(3) Loan notes The effective interest rate is 6%. Actual interest paid was Rs.1,500,000 (in trial balance); therefore the balancing Rs.1,500,000 should be added to the loan notes obligation, to make the total loan notes liability Rs.50 million + Rs.1,500,000 = Rs.51.5 million.

(4) Taxation

Deferred tax liability b/f 20,000

Deferred tax: credit in the statement of profit or loss 2,000

Deferred tax liability c/f (92,000 25%) 23,000

Tax expense

Income tax on profits for the year 18,000

Deferred tax movement 3,000

Tax charge in the statement of profit or loss 21,000

Page 113: CAF7-Financial Accounting and Reporting II_Questionbank

Answers

© Emile Woolf International 103 The Institute of Chartered Accountants of Pakistan

(5) Non-current assets and depreciation

Leasehold property Rs. in ‘000

Carrying value in the trial balance (250,000 – 40,000) 210,000

Amortisation charge for the year to 30 September 2015 (10,000)

200,000

Re-valued amount 220,000

Transfer to revaluation reserve 20,000

The annual depreciation charges for plant and equipment and the leased vehicles are shown in workings (1) Rs. in ’000

Cost or valuation

Accumulated depreciation

Carrying amount

Leasehold property 220,000 0 220,000

Plant and equipment (non-leased)

197,000 77,000 120,000

Leased vehicles 24,000 6,000 18,000

441,000 83,000 358,000

(6) Retained profits

At 1 October 2014 (trial balance) 19,300

Profit for the year 98,000

Retained profits at 30 September 2015 117,300

2.6 SARHAD SUGAR LIMITED

(a) Sarhad Sugar Limited – Statement of profit or loss for the year ended 30 September 2015

Revenue (300,000 – 2,500) 297,500 Cost of sales (w (i)) (225,400)

Gross profit 72,100 Distribution costs (14,500) Administrative expenses (22,200 – 400 + 100 see note below) (21,900) Finance costs (1,000)

Profit before tax 34,700 (Income tax expense (11,400 + (6,000 – 5,800 deferred tax)) (11,600)

Profit for the year 23,100

(b) Sarhad Sugar Limited

Statement of financial position as at 30 September 2015

Assets Non-current assets (w (ii)) Property, plant and equipment (43,000 + 38,400) 81,400 Development costs 14,800

96,200 Current assets

Inventory 20,000 Trade receivables 43,100

Page 114: CAF7-Financial Accounting and Reporting II_Questionbank

Financial accounting and reporting II

© Emile Woolf International 104 The Institute of Chartered Accountants of Pakistan

63,100

Total assets 159,300

Equity and liabilities:

Equity

Share capital 70,000 Retained earnings (w (iii)) 41,600

117,100 Revaluation reserve (w (iii)) 5,500 Non-current liabilities Deferred tax 6,000 Current liabilities

Trade payables (23,800 – 400 + 100 – re legal action) 23,500 Bank overdraft 1,300 Current tax payable 11,400

36,200

Total equity and liabilities 159,300

Note: As it is considered that the outcome of the legal action against Sarhad Sugar Limited is unlikely to succeed (only a 20% chance) it is inappropriate to provide for any damages. The potential damages are an example of a contingent liability which should be disclosed (at Rs.2 million) as a note to the financial statements. The unrecoverable legal costs are a liability (the start of the legal action is a past event) and should be provided for in full.

Workings (figures in brackets in Rs.000)

(i) Cost of sales: Rs. in ‘000

Per trial balance 204,000

Depreciation (w (iii)) – leasehold property 2,500

– plant and equipment 9,600

Loss on disposal of plant (4,000 – 2,500) 1,500

Amortisation of development costs (w (iii)) 4,000

Research and development expensed (1,400 + 2,400 (w (iii)) 3,800 –––––––– 225,400 ══════

(ii) Non-current assets:

Leasehold property

Valuation at 1 October 2014 50,000

Depreciation for year (20 year life) (2,500) –––––––– Carrying amount at date of revaluation 47,500

Valuation at 30 September 2015 (43,000) –––––––– Revaluation deficit 4,500 ══════

Plant and equipment per trial balance (76,600 – 24,600) 52,000

Disposal (8,000 – 4,000) (4,000) –––––––– 48,000

Depreciation for year (20%) (9,600) –––––––– Carrying amount at 30 September 2015 38,400 ══════

Page 115: CAF7-Financial Accounting and Reporting II_Questionbank

Answers

© Emile Woolf International 105 The Institute of Chartered Accountants of Pakistan

Capitalised/deferred development costs Rs. in ‘000

Carrying amount at 1 October 2014 (20,000 – 6,000) 14,000

Amortised for year (20,000 x 20%) (4,000)

Capitalised during year (800 x 6 months) 4,800 –––––––– Carrying amount at 30 September 2015 14,800 ══════

Note: development costs can only be treated as an asset from the point where they meet the recognition criteria in IAS 38 Intangible assets. Thus development costs from 1 April to 30 September 2015 of Rs.4·8 million (800 x 6 months) can be capitalised. These will not be amortised as the project is still in development.

The research costs of Rs.1·4 million plus three months’ development costs of Rs.2·4 million (800 x 3 months) (i.e. those incurred before 1 April 2015) are treated as an expense.

(iii) Movements on reserves

Revaluation surplus

Retained earnings

Rs. in ‘000 Balances at 1 October 2014 10,000 24,500 Dividend (6,000) Comprehensive income 23,100 Revaluation loss (4,500)

Balances at 30 September 2015 5,500 41,600

2.7 BSZ LIMITED

BSZ Limited Statement of financial position as at June 30, 2015

Note Rs. in million

ASSETS Fixed Assets Property, plant & equipment 1 576 Intangible assets 2 8

584

Long term advances – considered good 4

Current assets

Stocks in trade 90 Accounts receivable 3 57

Advances, deposits, prepayments and other receivables 4 45

Cash at banks 5 29

221

809

Page 116: CAF7-Financial Accounting and Reporting II_Questionbank

Financial accounting and reporting II

© Emile Woolf International 106 The Institute of Chartered Accountants of Pakistan

Rs. in

million EQUITY AND LIABILITIES Share capital and reserves Authorized share capital 50,000,000 shares of Rs. 10 each 500

Issued, subscribed and paid up capital 40,000,000 shares of Rs. 10 each 400 Unappropriated profit 65

465

Surplus on revaluation of fixed assets 120

Non-current liabilities

Deferred taxation 40

Current liabilities

Short term loan 85 Account and other payables 6 82 Provision for taxation 17

184

809

Notes Rs. in

million 1. Property, plant and equipment Operating assets 556 Capital work in progress – building 20

576

1.1 Operating assets Rs. in million

Cost/revalued amount Freehold

land Building Machines Fixtures Total As of July 01 2014 375.0 130.0 100.0 19.0 624.0 Additions - - - 8.0 8.0 Disposals - - (15.0) - (15.0)

As at June 30 2015 375.0 130.0 85.0 27.0 617.0

Accumulated depreciation

As of July 01 2014 - 19.5 22.5 5.9 47.9 For the year - 6.5 18.1

(105 × 85) + 10% × 15 × 8/12)

9.5

(105 × 19) + 10% × 8 × 3/12) 2.1 Disposals - - (5.0) - (5.0)

As at June 30 2015 - 26.0 27.0 8.0 61.0

Carrying amount 375.0 104.0 58.0 19.0 556.0

Depreciation rate - 5% 10% 10%

Page 117: CAF7-Financial Accounting and Reporting II_Questionbank

Answers

© Emile Woolf International 107 The Institute of Chartered Accountants of Pakistan

1.2 Revaluation

During the year 2011, the first revaluation of freehold land was carried out. The valuation was carried out under market value basis by an independent valuer, Mr. Dee, Chartered Civil Engineer of M/s SSS Consultants (Pvt.) Ltd., Islamabad. It resulted in a surplus of Rs. 120 million over book values which was credited to surplus on revaluation of fixed assets. Had there been no revaluation, the value of freehold land would be Rs. 255 million.

1.3 Disposal of machine

Rs. in million

Proceeds 13.0

Cost 15.0 Accumulated depreciation (5.0)

Carrying amount (10.0)

Profit on disposal 3.0

Note 2015

Rs. in

million 2. Intangible Assets Cost of computer software/license 10.0

Accumulated Amortization as of July 1, 2014 1.0 Amortization for the year 1.0

Accumulated Amortization as of June 30, 2015 2.0

Carrying value as at June 30, 2015 8.0

Amortization rate 10%

3. Accounts Receivable Considered good

- Secured 30 - Unsecured 27

57 Considered doubtful 3

60 Less: Provision for bad debts 3.1 3

57

3.1 Provision for bad debts Balance as at July 1, 2014 3.4

Provision made during the year 1.0 Amount written off during the year (1.4)

Balance as at June 30, 2015 (Rs. 30 million x 10%) 3.0

4 Advances, Deposits, Prepayments and Other Receivables Advances - suppliers - considered good 12 - staffs 6

18 Deposits 11 Prepayments 4 Sales tax receivable 12

45

Page 118: CAF7-Financial Accounting and Reporting II_Questionbank

Financial accounting and reporting II

© Emile Woolf International 108 The Institute of Chartered Accountants of Pakistan

5 Cash at banks Cash at banks - current accounts 7 saving accounts 5.1 22

29

5.1: It carries interest / mark up ranging from 3% to 7% per annum.

6 Accounts and other payables Accounts payable 75 Accrued liabilities 7

82

2.8 YASIR INDUSTRIES LIMITED

Yasir Industries Limited Statement of Financial Position as at June 30, 2015

Assets

Rs. in million

Non-current assets Property, plant and equipment (W2) 351.00 Intangible assets (20 – 12) 8.00 359.00 Current assets Inventories (W6) 64.50 Trade receivables (W5) 39.00 103.50

462.50

Equity and Liabilities Equity Issued, subscribed and paid up capital 120.00 Retained earnings (W4) 87.10 207.10 Revaluation surplus 41.25 Non-current liabilities Redeemable preference shares 40.00 Debentures 80.00 Deferred taxation (W 10) 9.00

129.00 Current liabilities Trade payables 30.40 Accrued expenses (W3) 25.00 Taxation 16.50 Bank overdraft 13.25 85.15

Total equity and liabilities 462.50

Page 119: CAF7-Financial Accounting and Reporting II_Questionbank

Answers

© Emile Woolf International 109 The Institute of Chartered Accountants of Pakistan

Yasir Industries Limited Statement of profit or loss for the year ended June 30, 2015

Rs. in million

Sales revenue (W5) 445.40 Cost of sales (W7) (250.72)

Gross profit 194.68 Distribution costs (W8) (20.05) Administrative expenses (W8) (40.38) Financial charges (W9) (9.10)

125.15 Loss due to fraud (30.00)

Profit before tax 95.15 Income tax expense (W10) (19.50)

Profit for the year 75.65

Workings

(W1) Leasehold property Annual depreciation before the revaluation (230 ÷ 40 years) = Rs. 5.75 million per

annum. Depreciation this year has been charged incorrectly on cost (whereas it should

have been on the revalued amount). This year’s charge must be added back

Dr Cr Accumulated depreciation 5.75 Cost of sales (50%) 2.88 Administrative expenses (30%) 1.72 Distribution costs (20%) 1.15

Rs. in

million Carrying amount at the 30 June (as per trial balance)(230.00 – 40.25) 189.75 Add back depreciation incorrectly charged (see above) 5.75

Carrying amount of property at the start of the year 195.5

Revaluation surplus Rs. in

million Revalued amount of leasehold property 238.00 Less: WDV of leasehold property at revaluation 195.50

Revaluation surplus arising in the year 42.50 Transfer to retained earnings in respect of incremental depreciation

(Rs. 7 million – Rs. 5.75 million) (1.25)

41.25

Depreciation of revalued property Number of years depreciation by the year end: (40.25 ÷ 5.75) = 7 years.

Therefore, remaining useful life as at the year-end = 33 years Revaluation was at the start of the year

Remaining useful life at the start of the year = 34 years

Depreciation charge based on the revalued amount (238/34 years) = Rs. 7 million

Page 120: CAF7-Financial Accounting and Reporting II_Questionbank

Financial accounting and reporting II

© Emile Woolf International 110 The Institute of Chartered Accountants of Pakistan

Dr Cr Cost of sales (50%) 3.5 Administrative expenses (30%) 2.1 Distribution costs (20%) 1.4 Accumulated depreciation 7.00

(W2) Property, plant and equipment Rs. in

million Leasehold property (Rs. 238m – 7) 231 Machines (Rs. 168.6 – Rs. 48.6m) 120

351

(W3) Accrued Expenses Rs. in

million As per trial balance 15.00 Accrued interest on debentures (Rs. 80m × 12% × 6/12) 4.80 Dividend on preference shares (Rs. 40m × 10%) 4.00

23.80

(W4) Retained earnings

Rs. in million

Balance as per trial balance 10.20

Profit for the year 75.65 Transfer from revalution surplus 1.25

87.10

(W5) Sales and receivables

Sales. Rec.

Rs. in million

Rs. m

Given in the trial balance 478.40 66.00

Deduct revenue incorrectly recognised (sale or return) (27.00) (27.00)

Cost of sales 451.40 39.00

(W6) Closing inventory

Rs. in million

Given in the question 42.00

Add back inventory held by customer on sale or return (100/120 27) 22.50

Cost of sales 64.50

(W7) Cost of sales

Rs. in million

Opening inventory as of July 1, 2014 38.90

Purchases 175.70

Direct labour 61.00

Manufacturing overheads excluding incremental depreciation 39.00

Less: Closing inventory (64.50)

Deduct depreciation incorrectly charged on cost (2.88)

Add depreciation charged on revalued amount 3.50

Cost of sales 250.10

Page 121: CAF7-Financial Accounting and Reporting II_Questionbank

Answers

© Emile Woolf International 111 The Institute of Chartered Accountants of Pakistan

(W8) Administrative expenses and distribution costs

Admin. DIst/

Rs. in million

Rs. m

Given in the trial balance 40.00 19.80

Deduct depreciation incorrectly charged on cost (1.72) (1.15)

Add depreciation charged on revalued amount 2.10 1.40

Cost of sales 40.38 20.05

(W9) Financial charges

Rs. in million

Balance as per trial balance 0.30

Accrued interest on debentures (Rs. 80m × 12% × 6/12) 4.80

Preference dividend for the year (Rs. 40m × 10%) 4.00

9.10

(W10) Taxation

Deferred taxation Rs. in million

Balance b/f 6.00

Charge for the year (balancing figure) 3.00

Balance c/f (30% Rs. 30 million temporary difference) 9.00

Tax expense Rs. in million

Current tax 16.50

Deferred tax (see above) 3.00

19.50

Page 122: CAF7-Financial Accounting and Reporting II_Questionbank

Financial accounting and reporting II

© Emile Woolf International 112 The Institute of Chartered Accountants of Pakistan

2.9 SHAHEEN LIMITED

Shaheen Limited

Statement of financial position

As of June 30, 2015

Assets Rs. in ‘000

Non-current assets

Property, plant and equipment (86,000 12,000 4,500) 69,500

Intangible assets (6,000 600) 5,400

74,900

Current assets

Stock in trade 30,000

Trade receivables (37,800 10,000) 27,800

Other receivables and prepayments (14,000 + 6,000) 20,000

Cash and bank balances 4,725

82,525

157,425

Equity and liabilities

Share capital and reserves

issued, subscribed and paid up capital 60,000

Unappropriated profit 35,372

95,372

Non-current liabilities

Long term borrowings (31,525 6,000) 25,525

Deferred taxation (5,000 1,470) 3,530

29,055

Current liabilities

Trade payables 12,000

Current portion of long term borrowings 6,000

Provision for litigation 5,000

Provision for taxation (2,000 + 9,988 2,000) 9,998

32,998

157,425

Shaheen Limited

Statement of profit or loss and other comprehensive income

As of June 30, 2015 Rs. in ‘000

Sales revenue 200,000

Cost of sales (W2) (104,708

)

Gross profit

95,292

Selling and distribution expenses (W2) (36,275)

Administrative expenses (W2) (30,450)

(66,725)

Financial charges (5,000)

Profit before taxation 23,567

Taxation (W3) (6,528)

Profit after taxation 17,039 Other comprehensive income – net of tax -

Total comprehensive income 17,039

Page 123: CAF7-Financial Accounting and Reporting II_Questionbank

Answers

© Emile Woolf International 113 The Institute of Chartered Accountants of Pakistan

Shaheen Limited

Statement of changes in equity 2015

As of June 30, 2015 Rs.000

Issued, subscribed &

paid up capital

Retained earnings

Balance July 1, 2014 60,000 32,000*

Correction of prior year error (10,000 20/120) (1,667)

Balance July 1, 2014 (restated) 60,000 30,333

Comprehensive income for the year 17,039

Dividend for the year ended June 30, 2014 (60,000*0.20) (12,000)

Balance June 30, 2015 60,000 35,372

*Retained earnings as at 01-07-09 = 20,000+ (20% of 60,000)=32,000

Workings

W1 Depreciation for the year

On building (36,000/20) 1,800

On plant and equipment (30,000 3,000)/10 2,700

Total

4,500

W2 Costs Cost of sales

Selling and distribution

costs

Administrative costs

Opening inventory 23,000

Costs as per Trial balance 100,000 35,000 30,000

Closing inventory (30,000)

Depreciation (75%, 15%, and 10% of Rs. 4,500) 3,375 675 450

Adjustment for goods sent on sale or return, erroneously booked as sales last year now returned during the year. (10,000/1.2) 8,333

Amortization of export license (6,000/5*0.5)

600

104,708 36,275 30,450

W3:Taxation

profit before tax

23,567

Disallowances and add backs

5,000

Taxable income

28,567

Current For the year (28,567*0.35) 9,998

For prior years (7,000 5,000) (2,000)

Deferred For the year (5,000 800)*0.35 (1,470)

6,528

Page 124: CAF7-Financial Accounting and Reporting II_Questionbank

Financial accounting and reporting II

© Emile Woolf International 114 The Institute of Chartered Accountants of Pakistan

2.10 MOONLIGHT PAKISTAN LIMITED

(a) Moonlight Pakistan Limited

Statement of Financial Position

As at December 31, 2015

Rs. in million

ASSETS

Non-current assets

Property, plant and equipment (W2) 3,472

Current assets

Stocks in trade 758

Trade receivables 702

Cash and bank 354

1,814

5,286

EQUITY

Issued, subscribed and paid-up capital (W3) 1,750

Share premium (420 x 2/12) 70

Retained earnings (W3) 876

2,696

Surplus on revaluation of fixed assets 240

LIABILITIES

Non-current liabilities

Long term loan 1,600

Deferred tax (22 + 80 x 35%) 50

Provision for gratuity 23

1,673

Current liabilities

Creditor and other liabilities (544 + 96) 640

Income tax payable 37

677

5,286

(b) Moonlight Pakistan Limited

Statement of profit or loss

For the year ended December 31, 2015

Rs. in million

Sales 3,608

Cost of sales (W1) (2,149)

Gross profit 1,459

Selling expenses (W1) 252

Administrative expenses (W1) 270

522

937

Financial charges (210 + 1,600 x 12% x 6/12) 306

Profit before taxation 631

Taxation (37 + 80 x 35%) 65

Profit after taxation 566

Page 125: CAF7-Financial Accounting and Reporting II_Questionbank

Answers

© Emile Woolf International 115 The Institute of Chartered Accountants of Pakistan

W1: Cost of sales/selling expenses/admin expenses

Cost of sales

Selling expenses

Admin. expenses

Rs. in million

As per trial balance 1,784 220 250

Depreciation – building (60% : 25% : 15%) (W2) 69 29 17

Depreciation – plant 287 - -

Provision for gratuity (23-8) x 60%:20%:20% 9 3 3

2,149 252 270

W2: Property, plant and equipment

Land Building Plant Total

Rs. in million

Cost as at January 1, 2015 600 2,000 2,104

4,704

Accumulated depreciation - (400) (670) (1,070)

Revaluation (1,840 - (2,000 - 400 )) - 240 -

240

Current year depreciation - (287) (402) (1,840/16) (115)

600 1,725 1,147

3,472

W3: Share Capital/Retained Earnings

Share capital Retained earnings Rs. in million

As per trial balance 1,200 510

Bonus issue (1200 ÷ 6) 200 (200)

Right issue (420 x 10/12) 350 -

Profit for the year - 566

1,750 876

2.11 FIGS PAKISTAN LIMITED

Figs Pakistan Limited Statement of profit or loss and other comprehensive income For the year ended 31 December 2015 2015

Note Rs. in million

Sales 1 44,758 Cost of sales 2 (26,203) Gross profit 18,555 Distribution costs 3 (6,431) Administrative expenses 4 (752) Other operating expenses 5 (399) Other operating income 6 30 Profit from operations 11,003 Finance costs 7 (166) Profit before tax 10,837 Taxation 8 (2,532) Profit after tax 8,305 Other comprehensive income -

Total comprehensive income for the year 8,305

Page 126: CAF7-Financial Accounting and Reporting II_Questionbank

Financial accounting and reporting II

© Emile Woolf International 116 The Institute of Chartered Accountants of Pakistan

Figs Pakistan Limited Notes to the financial statements For the year ended 31 December 2015

1 Sales Note Rs. in million

Manufactured goods Gross sales 56,528 Sales tax (10,201) 46,327 Imported goods Gross sales 1,078 Sales tax (53) 1,025 Sales discounts (2,594)

44,758

2 Cost of sales Raw material consumed (1,751 + 22,603 - 2,125) 22,229 Stores and spares consumed 180 Salaries, wages and benefits (2,367 × 55%) 2.1 1,302 Utilities (734 × 85%) 624 Depreciation and amortizations (1.287 × 70%) 901 Stationery and office expenses (230 × 25%) 58 Repairs and maintenance (315 × 85%) 268 25,562 Opening work in process 73 Closing work in process (125) 25,510 Opening finished goods (manufactured) 1,210 Closing finished goods (manufactured) (1,153) 25,567 Finished goods (imported) Opening stock 44 Purchases 658 702 Closing stock (66) 636

26,203

2.1 Salaries, wages and benefits include Rs. 30 million (54 × 55%) and Rs. 24

million (44 × 55%) in respect of defined contribution plan and defined benefit plan respectively.

Page 127: CAF7-Financial Accounting and Reporting II_Questionbank

Answers

© Emile Woolf International 117 The Institute of Chartered Accountants of Pakistan

3 Distribution costs

Rs. in million

Advertisement and sales promotion 4,040 Outward freight and handling 1,279 Salaries, wages and benefits (2,367 × 30%) 3.2 710 Utilities (734 × 5%) 37 Depreciation and amortization (1,287 × 20%) 257 Stationery and office expenses (230 × 40%) 92 Repairs and maintenance (315 × 5%) 16

6,431

3.1

Salaries, wages and benefits include Rs. 16 million (54 × 30%) and Rs. 13 million (44×30%) in respect of defined contribution plan and defined benefit plan respectively.

4 Administrative expenses Rs. in

million Salaries, wages and benefits (2,367 × 15%) 4.1 355 Utilities (734 × 10%) 73 Depreciation and amortization (1,287 × 10%) 129 Stationery and office expenses (230 × 35%) 80 Repairs and maintenance (315 × 10%) 31 Legal and professional charges 71 Auditor's remuneration 4.2 13

752

4.1

Salaries, wages and benefits include Rs. 8 million (54 × 15%) and Rs. 7 million (44×15%) in respect of defined contribution plan and defined benefit plan respectively.

4.2 Auditor's remuneration

Rs. in million

Audit fees 8 Taxation services 4 Out of pocket expenses 1

13

5 Other operating expenses Donation 5.1 34 Worker's Profit Participation Fund 257 Worker Welfare Fund 98 Loss on disposal of property, plant and equipment 10

399

5.1 Donations

Donations include Rs. 5 million given to Dates Cancer Foundation (DCF). One of the company’s directors, Mr. Peanut is a trustee of DCH.

Donations other than that mentioned above were not made to any donee in which a director or his spouse had any interest at any time during the year.

Page 128: CAF7-Financial Accounting and Reporting II_Questionbank

Financial accounting and reporting II

© Emile Woolf International 118 The Institute of Chartered Accountants of Pakistan

6 Other operating income

Rs. in million

Income from financial assets Dividend income 12 Return on savings account 2 Income from non-financial assets Scrap sales 16

30

7 Finance costs Finance charges on short term borrowings 133 Exchange loss 22 Finance charges on lease 11

166

8 Taxation Current - for the year 1,440 Deferred (3,120 × 35%) 1,092 2,532

Page 129: CAF7-Financial Accounting and Reporting II_Questionbank

Answers

© Emile Woolf International 119 The Institute of Chartered Accountants of Pakistan

CHAPTER 3 – IAS 7: STATEMENTS OF CASH FLOWS

3.1 KLEA

Statement of cash flows for the year ended 31st March 2015 Rs. in ‘000 Cash flows from operating activities Profit before taxation 1,606 Adjustments for: Depreciation (W4) 800 Finance income (50) Interest expense 320 –––––– 2,676 Increase in trade receivables (400) Increase in inventories (1,200) Increase in trade payables 334 –––––– Cash generated from operations 1,410 Interest paid (320) Income taxes paid (W1) (630) –––––– Net cash from operating activities 460 Cash flows from investing activities Purchase of intangible assets (W2) (300) Purchase of property, plant and equipment (W3) (1,600) Proceeds from sale of equipment 150 Purchase of long-term investments (200) Finance income received 50 –––––– Net cash used in investing activities (1,900) Cash flows from financing activities Proceeds from issue of share capital (1,000 + 278) 1,278 Payments to redeem debentures (400) Dividends paid (400) –––––– Net cash used in financing activities 478 ––––– Net decrease in cash and cash equivalents (962) Cash and cash equivalents at 1 April 2014 580 ––––– Cash and cash equivalents at 31 March 2015 (32 - 414) (382)

(Note: Alternative classifications of the cash flows in accordance with IAS 7 should receive full credit – i.e. interest and dividends received as investing activities or operating cash flows, interest and dividends paid as financing or operating cash flows.)

Notes

(1) Analysis of cash and cash equivalents Rs. in ‘000

2015 2014 Cash on hand and balances with bank 32 580 Bank overdraft (414) - –––– –––– Cash and cash equivalents (382) 580 –––– ––––

Page 130: CAF7-Financial Accounting and Reporting II_Questionbank

Financial accounting and reporting II

© Emile Woolf International 120 The Institute of Chartered Accountants of Pakistan

(2) Material non-cash transactions

During the year land was re-valued upwards by Rs.1million

Workings Rs. in ‘000

(W1) Taxation paid

Taxation creditor brought forward 400 Taxation expense for period 650 –––––– 1,050 Taxation creditor carried forward (420) –––––– Taxation paid in the year 630 ––––––

(W2) Intangible assets

Net book value brought forward 200 Capitalised in the year (from (i)) 300 –––– 500 Amortisation charged in year (from (i)) (200) –––– Intangibles acquired in the year 300 ––––

(W3) Property, plant and equipment

Cost brought forward 3,000 Revaluation in year (from (ii)) 1,000 Disposals (from (iii)) (600) Additions (balancing figure) 1,600 –––––– Cost carried forward 5,000 ––––––

(W4) Depreciation and amortisation

Depreciation (150 movement + 500 on disposal) 650 Amortisation 200 Profit on disposal (W5) (50) –––– Charge shown in statement of profit or loss 800 ––––

Hence add back of depreciation and amortisation also takes account of the profit on disposal of the plant and machinery.

(W5) Disposal

Cost of disposal 600 Accumulated depreciation (500) –––– Net book value 100 Proceeds of sale 150 –––– Profit on sale 50 ––––

Page 131: CAF7-Financial Accounting and Reporting II_Questionbank

Answers

© Emile Woolf International 121 The Institute of Chartered Accountants of Pakistan

3.2 STANDARD INC

Statement of cash flows for the year ended 31 December 2015

Rs. in ‘000 Rs. in ‘000

Cash flows from operating activities

Net profit before tax (W7) 64,000 Adjustments for: Depreciation, loss on sale (W1-5) 20,000 Interest receivable (450) Interest and premium payable 8,400 ———— Operating profit 91,950 Increase in inventories (14,000) Increase in receivables (1,200) Increase in payables 14,440 ———— Cash generated from operations 91,190 Interest paid (6,840) Tax paid (W6) (10,500) ———— Net cash from operating activities 73,850 Cash flows from investing activities Acquisition of long-term investment (4,600) Purchase of property plant and equipment (69,000) Receipt from sale of long-term investment 4,000 Interest received 450 ———— Net cash used in investing activities (69,150) Cash flows from financing activities Proceeds from issuance of shares 70,000 Redemption of long term loan (42,000) Dividends paid (7,500) ———— Net cash used in financing activities 20,500 ———— Net increase in cash and cash equivalents 25,200 ———— WORKINGS

(1) Plant and machinery account – at cost

Rs.000 Rs.000 Balance b/d 120,000 Disposals account 8,000 Additions 39,000 Balance c/d 151,000 ———— ———— 159,000 159,000 ———— ————

Page 132: CAF7-Financial Accounting and Reporting II_Questionbank

Financial accounting and reporting II

© Emile Woolf International 122 The Institute of Chartered Accountants of Pakistan

(2) Fixtures and fittings account – at cost

Rs.000 Rs.000 Balance b/d 24,000 Disposals account 5,000 Additions 10,000 Balance c/d 29,000 ——— ——— 34,000 34,000 ——— ———

Fixed assets – additions summary Rs.000

Freehold property Rs.000(130,000 - 110,000) 20,000 Plant and machinery 39,000 Fixtures and fittings 10,000 ——— 69,000 ——— (3) Plant and machinery account – depreciation

Rs.000 Rs.000 Disposals account 6,000 Balance b/d 45,000 Balance c/d 54,000 Charge for year 15,000 ——— ——— 60,000 60,000 ——— ———

(4) Fixtures and fittings account – depreciation

Rs.000 Rs.000 Disposals account 2,000 Balance b/d 13,000 Balance c/d 15,000 Charge for year 4,000 ——— ——— 17,000 17,000 ——— ———

(5) Fixed assets disposals account

Rs.000 Rs.000 Plant cost 8,000 Plant depreciation 6,000 Fittings cost 5,000 Fittings depreciation 2,000 Cash proceeds Plant 3,000 Fittings 1,000 Depreciation underprovided (bal fig) 1,000 ——— ——— 13,000 13,000 ——— ———

Page 133: CAF7-Financial Accounting and Reporting II_Questionbank

Answers

© Emile Woolf International 123 The Institute of Chartered Accountants of Pakistan

(6) Tax account

Rs.000 Rs.000 Cash paid (bal fig) 10,500 Balance b/f – corporation tax 21,500 Balance c/f – corporation tax 33,000 I&E account – corporation tax 22,000 ——— ——— 43,500 43,500 ——— ———

(7) Net profit before tax

Note As profit before tax is required, reconstruct the statement of profit or loss up to this figure.

Rs. in ‘000

Profit before tax 64,000 Taxation – Corporation tax (22,000) ——— 42,000 Dividends (15,000) ——— Retained profit for year 27,000 Balance b/f 14,000 ——— Balance c/f 41,000 ════

(8) Cash and cash equivalents as shown in the statement of financial position

Cash and cash equivalents consist of cash on hand and balances with banks.

Rs. in ‘000 2015 2014 Change in year Cash at bank 11,400 200 11,200 Bank overdraft – (14,000) 14,000 ——— ——— ——— 11,400 (13,800) 25,200 ════ ════ ════

Page 134: CAF7-Financial Accounting and Reporting II_Questionbank

Financial accounting and reporting II

© Emile Woolf International 124 The Institute of Chartered Accountants of Pakistan

3.3 FALLEN

Statement of cash flows for the year ended 31 December 2015

Cash flows from operating activities Rs. in ‘000

Net profit before tax 4,625 Adjustments for: Depreciation, (W1-3) 1,472 Interest payable 152 ——— Operating profit 6,249 Increase in deferred repairs provision 186 Increase in inventories (894) Increase in receivables (594) Increase in payables 324 ——— Cash generated from operations 5,271 Interest paid (152) Tax paid (W5) (1,775) ——— Net cash from operating activities 3,344 Cash flows from investing activities Acquisition of long-term investment (198) Purchase of property plant and equipment (3,800) Receipt from sale of long-term investment 168 ——— Net cash used in investing activities (3,830) Cash flows from financing activities Proceeds from issuance of shares (W6-7) 792 Redemption of long term loan (560) Dividends paid (544) —— Net cash used in financing activities (312) —— Net increase in cash and cash equivalents (798) ═══

WORKINGS Rs. in ‘000

(1) Leasehold premises (net)

Brought forward 5,700 Depreciation (to balance) 400 Additions 1,300 Carried forward 6,600 ——— ——— 7,000 7,000 ════ ════

Page 135: CAF7-Financial Accounting and Reporting II_Questionbank

Answers

© Emile Woolf International 125 The Institute of Chartered Accountants of Pakistan

(2) Plant (net)

Brought forward 3,780 Disposals 276 Additions 2,500 Depreciation (to balance) 964 Carried forward 5,040 ——— ——— 6,280 6,280 ════ ════

(3) Disposals

Plant 276 Cash 168 Loss on sale (to balance) 108 —–— —–— 276 276 ════ ════

(4) Dividends

Cash (to balance) 544 Brought forward 234 Carried forward 390 I&E account 700 —–— —–— 934 934 ════ ════

(5) Taxation

Cash (to balance) 1,775 Brought forward Carried forward DT 138 DT 202 CT 2,038 CT 1,730 I&E account 1,531 ——— ——— 3,707 3,707 ════ ════

(6) Share capital

Brought forward 1,800 Carried forward 2,280 Cash (to balance) 480 ——— ——— 2,280 2,280 ════ ════

(7) Share premium

Carried forward 2,112 Brought forward 1,800 Cash (to balance) 312 ——— ——— 2,112 2,112 ════ ════

Page 136: CAF7-Financial Accounting and Reporting II_Questionbank

Financial accounting and reporting II

© Emile Woolf International 126 The Institute of Chartered Accountants of Pakistan

(8) Long term loan

Cash (to balance) 560 Brought forward 1,800 Carried forward 1,240 ——— ——— 1,800 1,800 ——— ———

(9) Analysis of the balances of cash and cash equivalents as shown in the statement of financial position

Cash and cash equivalents consist of cash on hand and balances with banks.

Rs. in ‘000

2015 2014 Change in year

Cash at bank and in hand – 576 (576) Bank overdrafts (222) – (222) —— —— —— (222) 576 (798) ═══ ═══ ═══

3.4 BIN QASIM MOTORS LIMITED

Note: figures in brackets are in Rs.000

Bin Qasim Motors Limited

Statement of cash flows for the year to 30 September 2015

Rs.000 Rs.000

Cash flows from operating activities

Net profit before interest and tax (3,198 – 1,479) 1,719

Adjustments for:

Depreciation – buildings (W1) 80

– plant (W1) 276

Loss on disposal of plant (W1) 86 442

442

Amortisation of government grants (W2) (125)

Negligence claim previously provided (120)

Operating profit before working capital changes 1,916

Increase in inventories (1,046 – 785) (261)

Increase in accounts receivable (935 – 824) (111)

Decrease in accounts payable (760 – 644) (116)

Cash generated from operations 1,428

Interest paid (260 + 25 – 40) (245)

Income tax paid (W4) (368)

Dividends paid (180)

Net cash from operating activities 635

Cash flows from investing activities

Page 137: CAF7-Financial Accounting and Reporting II_Questionbank

Answers

© Emile Woolf International 127 The Institute of Chartered Accountants of Pakistan

Bin Qasim Motors Limited

Statement of cash flows for the year to 30 September 2015

Rs.000 Rs.000

Purchase of land and buildings (W1) (50)

Purchase of plant (W1) (848)

Purchase of non-current investments (690)

Purchase of treasury bills (120 – 50) (70)

Proceeds of sale of plant (W1) 170

Receipt of cash on servicing contracts (W2) 175

Investment income 120

Net cash used in investing activities (1,193)

Cash flows from financing activities

Issue of ordinary shares (W3) 300

Net decrease in cash and cash equivalents (258)

Cash and cash equivalents at the beginning of the period 122

Cash and cash equivalents at the end of the period (136)

Workings

(W1) Non-current assets

Rs.000

Land and buildings – cost/valuation

Balance b/f 1,800

Revaluation surplus 150

Balance c/f (2,000)

Difference cash purchase (50)

Plant – cost

Balance b/f 1,220

Disposal (500)

Balance c/f (1,568)

Difference cash purchase (848)

Depreciation of non-current assets:

Building (760 – 680) 80

Plant (464 – (432 – 244)) 276

The plant had a carrying value of Rs.256,000 at the date of its disposal (500 cost – 244 depreciation). As there was a loss on sale of Rs.86,000 (given in question), the sale proceeds must have been Rs.170,000 (i.e. 256 – 86).

(W2) Deferred income

Balances b/f – current (125)

– non-current (200)

(325)

Amortisation credited to cost of sales 125

Balances c/f – current 100

– non-current 275

375

Difference cash receipt 175

Page 138: CAF7-Financial Accounting and Reporting II_Questionbank

Financial accounting and reporting II

© Emile Woolf International 128 The Institute of Chartered Accountants of Pakistan

(W3) Share capital and convertible loan stock

A reconciliation of share capital, share premium and the revaluation reserve shows the shares issued for cash:

Share capital

Share premium

Revaluation reserve

Rs.000 Rs.000 Rs.000

Opening balance (1,000) (60) (40)

Revaluation of land (150)

Bonus issue 1 for 10 (100) 100

Conversion of loan stock (see below) (100) (300)

Closing balance 1,400 460 90 ––––– ––––– –––––

Difference issued for cash 200 100 nil ════ ════ ════

The 10% convertible loan stock had a carrying value of Rs.400,000 at the date of conversion to equity shares. This would be taken as the consideration for

the shares issued which would be 100,000 Rs.1 shares (i.e. 400,000/100 25). This would increase issued share capital by Rs.100,000 and share premium by Rs.300,000.

(W4) Income tax

Rs.000

Tax provision b/f (367)

Deferred tax b/f (400)

Statement of profit or loss tax charge (520)

Tax provision c/f 480

Deferred tax c/f 439

Difference cash paid (368)

Page 139: CAF7-Financial Accounting and Reporting II_Questionbank

Answers

© Emile Woolf International 129 The Institute of Chartered Accountants of Pakistan

3.5 ITTEHAD MANUFACTURING LTD

(a)

Ittehad Manufacturing Ltd

Statement of cash flows for the year to 30 September 2015

Rs.m Rs.m

Cash flows from operating activities

Net profit before interest and tax 920

Adjustments for:

Amortisation – development expenditure (W1) 130

Depreciation – property, plant and equipment 320

Loss on sale of plant 50

Increase in inventory (1,420 – 940) (480)

Increase in accounts receivable (990 – 680) (310)

Increase in accounts payable (875 – 730) 145

Decrease in deferred income (260 – 300) (40)

Cash generated from operations 735

Interest paid (30 – (15 – 5 accrual adjustments)) (20)

Income tax paid (W2) (130)

Net cash from operating activities 585

Cash flows from investing activities

Purchase of property, plant and equipment (W3) (250)

Capitalised development costs (W1) (500)

Proceeds of sale of plant (W3) 20

Net cash from investing activities (730)

Cash flows from financing activities

Issue of ordinary shares (W4) 450

Issue of loan notes (300 – 100) 200

Dividends paid (320)

Net cash generated from financing activities 330

Net increase in cash and cash equivalents 185

Cash and cash equivalents at beginning of period (115)

Cash and cash equivalents at end of period 70

Workings

(W1) Development expenditure

Rs.m

Opening balance 100

Amount capitalised 500

Closing balance (470)

Amortisation: balancing figure 130

Page 140: CAF7-Financial Accounting and Reporting II_Questionbank

Financial accounting and reporting II

© Emile Woolf International 130 The Institute of Chartered Accountants of Pakistan

(W2) Income tax

Rs.m Rs.m

Opening balance: tax provision 160

Opening balance: deferred tax 140

300

Tax charged to statement of profit or loss 270

Closing balance: tax provision (130)

Closing balance: deferred tax (310)

(440)

Tax paid (cash payments) 130

(W3) Property, plant and equipment

Rs.m

Opening balance 1,830

Revaluation surplus 200

Plant acquired 250

Depreciation (320)

1,960

Closing balance 1,890

Disposal at net book value – balancing figure 70

Disposal of plant:

Disposal at net book value (see above) 70

Loss on sale (given in the question) (50)

Difference = Sale proceeds 20

(W4) Share capital

Rs.m

Opening balance, ordinary shares 500

Bonus issue 1 for 10 (from retained earnings) 50

550

Closing balance, ordinary shares 750

Difference: shares issued for cash (nominal value) 200

Plus increase in share premium (350 – 100) 250

Total cash proceeds of issue of ordinary shares 450

(b) The cash flows generated from operations were Rs.685 million and are more than enough to pay the interest costs and taxation, but these cash flows are not as large as the equivalent profit figure. For most companies the operating cash flows are higher than the profit before interest and tax due to the effects of depreciation/amortisation charges (which are not cash flows). In the case of Ittehad Manufacturing Ltd the depreciation/amortisation effect has been more than offset by a much higher investment in working capital of Rs.645 million. Inventory has increased by over 50% and accounts receivable by 45%. This may be an indication of expanding activity, but it could also be an indication of poor inventory management policy and poor credit control, or even the presence of some obsolete inventory or unprovided bad accounts receivable.

Page 141: CAF7-Financial Accounting and Reporting II_Questionbank

Answers

© Emile Woolf International 131 The Institute of Chartered Accountants of Pakistan

A cause of concern is the size of the dividends, which seem high at Rs.320 million. This is a very high distribution ratio, and it seems odd that the company is returning such large amounts to shareholders at the same time as they are raising finance. Rs.450 million has been received from the issue of new shares and Rs.200 million from a further issue of loan notes.

The company has invested considerably in new plant (Rs.250 million) and even more so in development expenditure (Rs.500 million). If management has properly applied the capitalisation criteria in IAS 38 Intangible Assets, then this indicates that they expect good future returns from the investment in new products or processes. The net investment in non-current assets is Rs.680 million which closely correlates to the proceeds from financing of Rs.650 million. In general it is acceptable to finance increases in the capacity of non-current assets by raising additional finance, however operating cash flows should finance replacement of consumed non-current assets.

3.6 WASEEM INDUSTRIES LIMITED

Waseem Industries Limited Statement of cash flows for the year ended December 31, 2015 2015 Workings Rs.m Cash flows from operating activities

Profit before taxation

64 W1 Adjustments for: Depreciation 17

Gain on sale of fixed assets

(3) Provision for gratuity 10 W2 Interest expense 16

104 Increase/decrease in working capital

Increase in stocks-in-trade (7) (55-48) Increase in trade debts (13) (51-38) Decrease in advance, prepayments and other

receivables 6 W3 Increase in trade and other payables 22 W4

8

Cash generated from operations 112 Gratuity paid (6) Interest paid (18) W5 Income taxes paid (22) W6

Net cash from operating activities 66

Page 142: CAF7-Financial Accounting and Reporting II_Questionbank

Financial accounting and reporting II

© Emile Woolf International 132 The Institute of Chartered Accountants of Pakistan

Cash flows from investing activities

Sale proceeds from sale of property, plant and equipment 26

Purchase of property, plant and equipment

(100) W7

Increase in capital work in progress (2) (20 -18) Sale of long term investments 25 (75-100)

Net cash used in investing activities (51) Cash flows from financing activities

Payment of long term finances (21) W8 Increase in short term finances 7 (13 - 6) *Dividend paid (10)

Net cash used in financing activities (24)

Net increase in cash and cash equivalents (9) Cash and cash equivalent at the beginning of the year 20

Cash and cash equivalent at the end of the year 11

W1: Profit before taxation Rs.m Unappropriated profit – closing 85 Income tax expenses for the year 2015 19 Dividend (Rs. 125 million x 8%) 10

114 Less: Unappropriated profit - opening (50)

64

W2: Provision for gratuity Rs.m Provision for gratuity: Closing 16 Paid during the year 2015 6

22 Less: Provision for gratuity - opening 12

Provision for the year 10

W3: Advances, prepayments and other receivables Rs.m

Advances, payments and other receivables – closing 37 Advance Tax - closing (10)

27

Advances, payments and other receivables – opening 40 Advance Tax – opening (7)

33

Decrease in advances, prepayments and other receivables

(6)

Page 143: CAF7-Financial Accounting and Reporting II_Questionbank

Answers

© Emile Woolf International 133 The Institute of Chartered Accountants of Pakistan

W4: Trade and other payables Rs.m

Trade and other payables – closing 66 Accrued mark-up – closing (7)

59

Trade and other payables – opening 46 Accrued mark-up – opening (9)

37

Increase in trade and other payables 22

W5: Interest paid Rs.m Accrued mark up – opening 9 Expense for the year 16

25 Less: Accrued mark-up – closing (7)

Interest paid during the year 18

W6: Income taxes paid Rs.m Advances taxes – closing 10 Provision for the year 19

29 Less: advance taxes - opening (7)

Income taxes paid during the year 22

W7: Fixed assets purchase Rs.m Closing fixed assets 242 Depreciation for the year 17 Carrying amount of disposed off assets 23

302 Less: opening fixed assets (182)

Purchase of fixed assets 100

W8: Payment of long term finances Rs.m Long term finance including current portion – closing

(118 + 22)

140

Long term finance including current portion – Opening (94 + 25)

(119)

Payment during the year 21

Page 144: CAF7-Financial Accounting and Reporting II_Questionbank

Financial accounting and reporting II

© Emile Woolf International 134 The Institute of Chartered Accountants of Pakistan

3.7 JALIB INDUSTRIES LIMITED

Jalib Industries Limited Statement of cash flow for the year ended December 31, 2015

Rs. in million

CASH FLOW FROM OPERATING ACTIVITIES Net profit before tax 191.40 Adjustments for: Depreciation 27.70 Loss on sale of fixed assets 4.60 Provision for gratuity 15.50 Financial charges 10.50 Bad debt expense 1.20 Working 3

250.90 Working Capital Changes

Increase in creditors, accrued and other liabilities ([36.2 - 5] - [34.4 - 6]) 2.80

Increase in stock in trade (80.80) Increase in trade debts (5.00) Working 4

Decrease in advances and other receivables [(42-2.2)-(37.4-3.6)] 6.00

Cash generated from operations 173.90 Gratuity paid (4.40) Working 2 Income tax paid [3.6 + 104.6 - 2.2] (106.00) Financial charges paid (6 + 10.5 - 5) (11.50)

Net cash from operating activities 52.00

CASH FLOW FROM INVESTING ACTIVITIES

Capital expenditure incurred (57.00) Working 1 Proceeds on sale of fixed assets 10.40

Net cash used in investing activities (46.60)

CASH FLOW FROM FINANCING ACTIVITIES

Issue of share capital 99.00 Working 5 Repayment of long term loans (120 - 98) (22.00) Dividend paid (1.4 + 75 - 3) (73.40)

Net cash from financing activities 3.60

Net increase in cash and cash equivalents 9.00 Cash and cash equivalent at the beginning of year 3.00

Cash and cash equivalent at the end of year 12.00

WORKING 1

Rs. in million

Capital expenditure incurred Book value of PPE - Closing 129.40 Book value of CWIP - Closing 22.50 Add: Book value of assets sold during the year 15.00 Add: Depreciation for the year 27.70 Less: Book value of PPE - Opening (100.60) Less: Book value of CWIP - Opening (37.00)

57.00

Page 145: CAF7-Financial Accounting and Reporting II_Questionbank

Answers

© Emile Woolf International 135 The Institute of Chartered Accountants of Pakistan

WORKING 2

Rs. in million

Gratuity paid during the year Opening balance 27.50 Provision for gratuity 15.50

43.00 Less: Closing balance (38.60)

Gratuity paid during the year 4.40

WORKING 3

Bad debts expense for the year Closing balance (28.5 ÷ 0.95) - 28.5 1.50 Less: Opening balance (24.7 ÷ 0.95) - 24.7 (1.30) Add: Bad debts written off 1.00

Bad debts expense for the year 1.20

WORKING 4 Increase in trade debts Closing balance (28.5 ÷ 0.95) 30.00 Less: Opening balance (24.7 / 0.95) (26.00) Add: Bad debts written off 1.00

5.00

WORKING 5 Issue of share capital Closing balance of paid up capital 396.00 Closing balance of share premium 45.00 Less: Opening balance of paid up capital (300.00) Opening balance of share premium (12.00) Issue of bonus shares (300 x 10%) (30.00)

99.00

Page 146: CAF7-Financial Accounting and Reporting II_Questionbank

Financial accounting and reporting II

© Emile Woolf International 136 The Institute of Chartered Accountants of Pakistan

3.8 APOLLO INDUSTRY LIMITED

Apollo Industry Limited Statement of cash flows for the year ended December 31, 2015 Rs. in

‘000 Cash used in operating activities Profit before taxation 6,500 Adjustment for: (non cash items / separately disclosed items) Depreciation for the year (7,000-90-1,000) 5,910 Amortization for the year (1140+50-1100) 90 Provision for staff gratuity (1,400+300-1,190) 510 Profit on sale of fixed assets (2,800-1,000) (1,800) Mark-up on short term placement (1,000)

Operating profit before working capital changes 10,210 Increase in working capital (12,125 – 15,700 + 4,200 – 6,250) (5,625)

Cash generated from operations 4,585 Payment for staff gratuity (300) Payment for taxation (950 + 4,660 – 800) (4,810)

(525) Cash used in investing activities

Capital expenditure incurred Note 1 (13,110) Proceeds from sale of PPE (1,200 + 1,800) 3,000 Acquisition of intangible assets (50) Mark-up received on short term placement 1,000 Long term deposits (400-300) (100)

(9,260) Cash used in financing activities Issue of ordinary share capital (25,000-2,000-20,000) 3,000

Net decrease in cash and cash equivalents (6,785) Opening balance: cash and cash equivalents 7,225

Closing balance: cash and cash equivalents 440

Note 1 Capital expenditure incurred: Rs.000 Opening book value for PPE 25,500 Opening book value for CWIP 10,000

Book value of assets sold during the year (1,200)

Depreciation for the year (7,000-90-1,000) (5,910)

Revaluation reserve adjustment (1,000) Closing book value for PPE (35,000) Closing book value for CWIP (5,500)

(13,110)

Page 147: CAF7-Financial Accounting and Reporting II_Questionbank

Answers

© Emile Woolf International 137 The Institute of Chartered Accountants of Pakistan

3.9 MARVEL ENGINEERING LIMITED

Marvel Engineering Limited Cash Flow Statement For the year ended 30 June 2015

Workings 2015

Cash flows from operating activities

Rs.m

Profit before taxation

88.00

Adjustment for non-cash charges and other items: Depreciation

50.00

Impairment of plant and machinery

11.00

Financial charges

75.00

Gain on sale of fixed assets

(2.00)

Gain on sale of investments

(3.00)

Dividend income

(30.00)

Provision for gratuity payable (55 - 50 + 6)

11.00

Working capital changes Decrease / (increase) in current assets: Increase in stock-in-trade (97 - 68)

(29.00)

Increase in trade debts (see tutorial note)

(76.00)

Other current assets (100 - 120)

20.00

Increase / (decrease) in current liabilities: Trade and other payables ([73 - 7] - [56 - 3])

13.00

Cash generated from operations

128.00

Financial charges paid (3 + 75 - 7)

(71.00)

Income tax paid (5 + 21 + 21 - 12 - 15)

(20.00)

Gratuity paid

(6.00)

Net cash generated from operating activities

31.00

Cash flows from investing activities Capital expenditure 1 (289.00)

Proceeds from sale of property, plant and equipment (5+2) 7.00

Proceeds from sale of investments (10+3)

13.00

Purchase of long term investments (130-100+10)

(40.00)

Dividend received

30.00

Net cash used in investing activities

(279.00)

Cash flows from financing activities

Insurance of ordinary shares 2 40.00

Proceeds from long term loan (330 - 110)

220.00

Payment of dividend (2 + (440 × 5%) - 4)

(20.00)

Net cash from financing activities

240.00

Net decrease in cash and cash equivalents

(8.00)

Cash and cash equivalent at the beginning of the year

39.00

Cash and cash equivalent at the end of the year

31.00

Page 148: CAF7-Financial Accounting and Reporting II_Questionbank

Financial accounting and reporting II

© Emile Woolf International 138 The Institute of Chartered Accountants of Pakistan

W1: Capital expenditure

Rs.m

Closing balance

633.00

Add: Depreciation for the year

50.00

Add: Impairment against plant

11.00

Add: Disposal during the year

5.00

Less: Opening balance

(410.00)

289.00

W2: Issuance of ordinary shares Closing balance of share capital

494.00

Closing balance of share premium

8.00

Less: Bonus shares issued (440 × 5%)

(22.00)

Less: Opening balance of share capital

(440.00)

40.00

Tutorial note:

The original ICAP answer did not simply adjust for the movement in trade debts but added back the write off for bad debts (Rs. 6 million) and movement in the doubtful debt provision (Rs. 4 million) and then adjusted for the movement in trade debt before these write offs (Rs. 86 million).

As the trade debt contains the credit for the write off and the profit for the year contains the debit it is easier to leave the expense in and adjust for the net movement.

The following working was provided in the official answers.

WORKINGS (All amount in million rupees)

W1:

Provision for bad debts

Trade

debtors

Closing balance (133 ÷ 0.95) - 133 7.00 (133 ÷ 0.95) 140.00

Add: Bad debts written off

6.00

6.00

Less : Opening balance (57÷ 0.95) - 57 (3.00) (57÷ 0.95) (60.00)

10.00

86.00

Page 149: CAF7-Financial Accounting and Reporting II_Questionbank

Answers

© Emile Woolf International 139 The Institute of Chartered Accountants of Pakistan

CHAPTER 4 – CONSOLIDATED ACCOUNTS: STATEMENTS OF FINANCIAL

POSITION– BASIC APPROACH

4.1 HALL

Consolidated statement of financial position as at 31 December 2015

Rs.000 Assets Non-current assets

Property, plant and equipment (35,000 + 20,000) 55,000 Goodwill 3,000 ———— 58,000

Current assets (16,000 + 14,000) 30,000 ———— 88,000 ————

Equity and liabilities Capital and reserves

Share capital 10,000 Retained earnings (W5) 16,000 ———— 26,000

Non-controlling interest (W4) 4,000 Long-term liabilities

8% Debenture loans (20,000 + 9,000) 29,000 Current liabilities (20,000 + 9,000) 29,000

———— 88,000 ————

WORKINGS

(1) Group structure

Hall

Stand

75%

(2) Net assets of Stand

Reporting

date Date of

acquisition

Post acquisition

Rs.000 Rs.000 Share capital 4,000 4,000 Retained earnings 12,000 8,000 4,000

16,000 12,000

Page 150: CAF7-Financial Accounting and Reporting II_Questionbank

Financial accounting and reporting II

© Emile Woolf International 140 The Institute of Chartered Accountants of Pakistan

(3) Goodwill

Rs.000 Cost of shares 12,000

Less Net assets acquired (75% 12,000 (W2)) (9,000) ——— 3,000 ———

(4) Non-controlling interest (25% 16,000 (W2)) Rs.000

4,000

——–

(5) Retained earnings

Rs.000 Hall Inc 13,000

Stand Inc (75% 4,000 (W2)) 3,000 ——– 16,000 ——–

4.2 HASSLE

Consolidated statement of financial position as at 31 December 2015

Rs. Sundry net assets (207,500 + 226,600) 474,100 ———— 474,100 ———— Equity capital 120,000 Retained earnings (W5) 123,500 ———— 243,500 Non-controlling interests (W4) 24,000 Sundry liabilities (100,000 + 106,600) 206,600 ———— 474,100 ———— WORKINGS

(1) Group structure

Hassle

Strife

80%

Page 151: CAF7-Financial Accounting and Reporting II_Questionbank

Answers

© Emile Woolf International 141 The Institute of Chartered Accountants of Pakistan

(2) Net assets of Strife

Reporting

date Date of

acquisition

Post acquisition

Rs. Rs. Share capital 50,000 50,000 Retained earnings 70,000 50,000 20,000

120,000 100,000

(3) Goodwill Rs.

Cost 60,000

Net assets acquired (80% 100,000) (W2) (80,000) ———— (20,000) ————

(4) Non-controlling interest Rs.

20% 120,000) (W2) 24,000

(5) Retained earnings Rs.

Hassle 87,500

Strife (80% (70,000 − 50,000) (W2)) 16,000 “Negative goodwill” (W3) 20,000 ———– 123,500 ———–

4.3 HYMN

Consolidated statement of financial position as at 31 December 2015

Rs. Assets Non-current assets

Property, plant and equipment 170,000 Goodwill 29,000

Current assets 275,000 ———— 474,000 ————

Equity and liabilities Shareholders’ equity

Share capital 100,000 Retained earnings (W5) 178,200

———–

278,200 Non-controlling interest (W4) 19,800 Current liabilities 176,000 ———— 474,000 ————

Page 152: CAF7-Financial Accounting and Reporting II_Questionbank

Financial accounting and reporting II

© Emile Woolf International 142 The Institute of Chartered Accountants of Pakistan

WORKINGS

(1) Group structure

Hymn

Psalm

80%

(2) Net assets of Psalm

Reporting

date Date of

acquisition

Post acquisition

Rs. Rs. Share capital 50,000 50,000 Retained earnings 49,000 20,000 29,000

99,000 70,000

(3) Goodwill Rs.

Cost of shares 85,000 Net assets acquired

Psalm Inc (80% 70,000) (W2) (56,000) ————

(29,000) ————

(4) Non-controlling interest Rs.

20% 99,000 (W2) 19,800

(5) Retained earnings Rs.

Hymn 155,000

Psalm (80% 29,000 (W2)) 23,200 —————

178,200 —————

Page 153: CAF7-Financial Accounting and Reporting II_Questionbank

Answers

© Emile Woolf International 143 The Institute of Chartered Accountants of Pakistan

4.4 HANG

Consolidated statement of financial position as at 31 December 2015

Rs. Assets Non-current assets Property, plant and equipment (240 + 180) 420,000 Goodwill 26,600 Current assets (250 + 196) 446,000 ————– 892,600 ————– Equity and liabilities Shareholders’ equity Share capital 200,000 Share premium account 25,000 Retained earnings (W5) 198,000 ————– 423,000 Non-controlling interest (W4) 87,600 Current liabilities (225 + 157) 382,000 ————– 892,600 ————–

WORKINGS

(1) Group structure

Hang

Swing

60%

(2) Net assets of Swing Inc

31 Dec 31 Dec 2015 2014 Rs. Rs.

Ordinary shares of Rs.1 each 90,000 90,000 Share premium account 49,000 49,000 Retained earnings 80,000 50,000 ———– ———– 219,000 189,000 ———– ———–

Page 154: CAF7-Financial Accounting and Reporting II_Questionbank

Financial accounting and reporting II

© Emile Woolf International 144 The Institute of Chartered Accountants of Pakistan

Reporting

date Date of

acquisition

Post acquisition

Rs. Rs. Share capital 90,000 90,000 Share premium 49,000 49,000 Retained earnings 80,000 50,000 30,000

219,000 189,000

(3) Goodwill Rs.

Cost 140,000

Net assets acquired (60% 189,000) (W2) (113,400) ———— 26,600 ————

(4) Non-controlling interest Rs.

40% 219,000 (W2) 87,600

(5) Retained earnings Rs.

Hang 180,000

Swing (60% 30,000 (W2)) 18,000 ————– 198,000 ————–

4.5 HASH

Consolidated statement of financial position as at 31 December 2015

Rs.000 Sundry net assets (207,500 + 226,600) 434,100 Goodwill (W2) 8,800 ———— 442,900 ———— Share capital 120,000 Retained earnings (W5) 92,300 ———— 212,300 Non-controlling interests (W4) 24,000 Sundry liabilities (100,000 + 106,600) 206,600 ———— 442,900 ————

Page 155: CAF7-Financial Accounting and Reporting II_Questionbank

Answers

© Emile Woolf International 145 The Institute of Chartered Accountants of Pakistan

WORKINGS

(1) Group structure

Hash

Stash

80%

(2) Net assets of Stash

Reporting

date Date of

acquisition

Post acquisition

Rs.000 Rs.000 Share capital 50,000 50,000 Retained earnings:

At the start of the year (70,000 – 24,000) 46,000

Profit for the first 9m (24,000 ×9/12) 18,000

70,000 64,000 6,000

120,000 114,000

(3) Goodwill Rs.000

Cost 100,000

Net assets acquired (80% 114,000) (W2) (91,200) ——— 8,800 ———

(4) Non-controlling interest Rs.000

20% 120,000) (W2) 24,000

(5) Retained earnings Rs.000

Hash 87,500

Stash (80% (70,000 − 64,000) (W2)) 4,800 ———– 92,300 ———–

Page 156: CAF7-Financial Accounting and Reporting II_Questionbank

Financial accounting and reporting II

© Emile Woolf International 146 The Institute of Chartered Accountants of Pakistan

CHAPTER 5 – CONSOLIDATED ACCOUNTS: STATEMENTS OF FINANCIAL

POSITION– COMPLICATIONS

5.1 HAIL

Consolidated statement of financial position as at 31 December 2015

Rs.000 Rs.000 Assets Non-current assets Property, plant and equipment 246,000 Investments (68,000 – 65,000) 3,000 Goodwill (W3) 6,500 Current assets Cash at bank and in hand 39,900 Trade receivables 138,300 Inventories 92,400

———–

526,100

———–

Equity and liabilities

Capital and reserves Share capital 100,000 Capital reserve (W6) 18,000 Retained earnings (W5) 210,480

———–

328,480 Non-controlling interest (W4) 11,420

Current liabilities Trade payables 183,000 Proposed dividend – parent company 3,000 – non controlling interest 200

———– 3,200

———–

526,100

———–

WORKINGS

(1) Group structure

Hail

Snow

90%

Page 157: CAF7-Financial Accounting and Reporting II_Questionbank

Answers

© Emile Woolf International 147 The Institute of Chartered Accountants of Pakistan

(2) Net assets of Snow

Reporting

date Date of

acquisition

Post acquisition

Rs.000 Rs.000 Share capital 50,000 50,000 Share premium account 5,000 5,000

Revaluation reserve 20,000

Retained earnings

Per question 41,200 Proposed dividend (2,000)

39,200 10,000 29,200

114,200 65,000

(3) Goodwill

Rs.000 Cost of shares 65,000

Net assets acquired (90% 65,000) (W2) (58,500)

————

6,500

————

(4) Non-controlling interest

Rs.000

10% 114,200 (W2) 11,420

————

(5) Retained earnings

Rs.000 Hail 185,400 Proposed dividend (3,000) Dividend receivable from Snow 1,800

Snow (90% 29,200 (W2)) 26,280

————

210,480

————

(6) Capital reserve

Rs.000

Snow (90% 20,000 (W2)) 18,000

————

Page 158: CAF7-Financial Accounting and Reporting II_Questionbank

Financial accounting and reporting II

© Emile Woolf International 148 The Institute of Chartered Accountants of Pakistan

5.2 HAIRY

Consolidated statement of financial position as at 31 December 2015

Rs.000 Assets Non-current assets Property, plant and equipment 180,000

Current assets Cash at bank and in hand 15,500 Investments 3,000 Receivables 91,700 Inventory (17,000 + 11,000 – 800) 27,200

———– 317,400 ———–

Equity and liabilities Capital and reserves Share capital 100,000 Share premium account 20,000 Capital reserve 23,000 Retained earnings (W5) 102,900

———– 245,900

Non-controlling interest (W4) 16,500

Current liabilities 55,000 ———– 317,400 ———–

WORKINGS

(1) Group structure

Hairy

Spider

80%

Page 159: CAF7-Financial Accounting and Reporting II_Questionbank

Answers

© Emile Woolf International 149 The Institute of Chartered Accountants of Pakistan

(2) Net assets of Spider

Reporting

date Date of

acquisition

Post acquisition

Rs.000 Rs.000

Share capital 60,000 60,000 Share premium account 16,000 16,000 Retained earnings

Per question 7,300 Unrealised profit (800)

6,500 2,300 4,200

82,500 78,300

(3) Goodwill Rs.000

Cost of shares 55,000

Less Net assets acquired (80% 78,300 (W2)) (62,640) ———— (7,640) ————

(4) Non-controlling interest Rs.000

Share of net assets (20% 82,500 (W2)) 16,500 ————

(5) Retained earnings Rs.000

Hairy 91,900

Spider (80% 4,200 (W2)) 3,360 Negative goodwill (W4) 7,640 ———— 102,900 ————

Page 160: CAF7-Financial Accounting and Reporting II_Questionbank

Financial accounting and reporting II

© Emile Woolf International 150 The Institute of Chartered Accountants of Pakistan

5.3 HARD

Consolidated statement of financial position as at 31 December 2015

Rs.000

Assets Non-current assets

Property, plant and equipment (225 + 175 – 17.5 (W6)) 382,500 Goodwill (W3) 14,000

Current assets (271 + 157) 428,000 ———– 824,500 ———–

Equity and liabilities Shareholders’ equity

Share capital 100,000 Share premium account 15,000 Retained earnings (W5) 260,500 ———– 375,500

Non-controlling interest (W4) 76,000 Current liabilities 373,000

———– 824,500 ———–

WORKINGS

(1) Group structure

Hard

Soft

60%

(2) Net assets of Soft Inc

31 Dec 2015

31 Dec 2014

Post acquisition

Rs.000 Rs.000

Share capital 100,000 100,000 Share premium account 10,000 10,000 Retained earnings 80,000 50,000 30,000

190,000 160,000

Page 161: CAF7-Financial Accounting and Reporting II_Questionbank

Answers

© Emile Woolf International 151 The Institute of Chartered Accountants of Pakistan

(3) Goodwill Rs.000

Cost 110,000 Net assets acquired

60% 160,000 (W2) (96,000) ———— 14,000 ————

(4) Non-controlling interest Rs.000

40% 190,000 (W2) 76,000

(5) Retained earnings Rs.000

Hard 260,000 Less Adjustment re intra group transfer (17,500) ———— 242,500

Soft (60% (80,000 – 50,000 (W2)) 18,000 ———— 260,500 ————

(6) PURP on non current assets

IS Rs.000 Cost 50,000 Accumulated depreciation (12,500) ———— 37,500 ———— SHOULD BE Cost 100,000 Accumulated depreciation (80,000) ———— 20,000 ———— Dr Retained earnings 17,500 Cr Non current assets 17,500

Page 162: CAF7-Financial Accounting and Reporting II_Questionbank

Financial accounting and reporting II

© Emile Woolf International 152 The Institute of Chartered Accountants of Pakistan

5.4 HALE

(a) Consolidated statement of financial position as at 31 December 2015

Rs.000 Assets Non-current assets Property, plant and equipment (152,000 + 129,600 + 28,000 (W2)) 309,600 Goodwill (W3) 61,400 Current assets Bank (41,000 + 8,000) 49,000 Receivables (104,000 + 84,000) 188,000 Inventory (112,000 + 74,400 – 3,200 (W6)) 183,200 ————– 791,200 ————– Equity and liabilities Capital and reserves Share capital 100,000 Retained earnings (W5) 555,200 ————– 655,200 Non-controlling interest (W3) 60,000 Current liabilities (52,000 + 24,000) 76,000 ————– 791,200 ————–

WORKINGS

(1) Group structure

Hale

Sowen

= 80% ord ordords ords

128

160

(2) Net assets of Sowen

Reporting

date Date of

acquisition

Post acquisition

Rs.000 Rs.000 Share capital 160,000 160,000 Fair value adjustment on non-current assets 28,000 28,000

Retained earnings 112,000 (11,000) 123,000

300,000 177,000

Page 163: CAF7-Financial Accounting and Reporting II_Questionbank

Answers

© Emile Woolf International 153 The Institute of Chartered Accountants of Pakistan

(3) Goodwill

Rs.000 Cost of shares 203,000

Less Net assets acquired (80% 177,000 (W2)) (141,600) ————– 61,400 ————–

(4) Non-controlling interest

Share of net assets (20% 300,000 (W2)) Rs.000 60,000 ————

(5) Retained earnings

Rs.000 Hale 460,000 PURP (W6) (3,200)

Sowen (80% 123,000 (W2)) 98,400 ————– 555,200 ————–

(6) Unrealised profits

% Rs.000 SP 125 16,000 Cost (100) (12,800) —— ——— GP 25 3,200 —— ———

5.5 HELLO

Consolidated statement of financial position as at 31 December 2015

Rs. Assets Non-current assets

Property, plant and equipment (225 + 175 + 10 – 2) 408,000 Goodwill (W3) 8,000

Current assets (271 + 157) 428,000 ———– 844,000 ———–

Equity and liabilities Shareholders’ equity

Called up share capital 100,000 Retained earnings (W5) 291,800 ———– 391,800

Non-controlling interest (W4) 79,200 Current liabilities 373,000

———– 844,000 ———–

Page 164: CAF7-Financial Accounting and Reporting II_Questionbank

Financial accounting and reporting II

© Emile Woolf International 154 The Institute of Chartered Accountants of Pakistan

WORKINGS

(1) Group structure

Hello

Solong

60%

(2) Net assets of Solong Inc

Reporting

date Date of

acquisition

Post acquisition

Rs. Rs. Share capital 100,000 100,000 Retained earnings

Per the question 90,000 Less: Fair value adjustment for depreciation (2/10 × 10,000) (2,000)

88.000 60,000 Fair value adjustment 10,000 10,000

198,000 170,000

(3) Goodwill Rs. Rs.

Cost 110,000 Net assets acquired

60% 170,000 (W2) (102,000) ———— 8,000 ————

(4) Non-controlling interest Rs.

40% 198,000 (W2) 79,200

(5) Retained earnings Rs.

Hello 275,000

Solong (60% (88,000 – 60,000 (W2)) 16,800 ———– 291,800 ———–

Page 165: CAF7-Financial Accounting and Reporting II_Questionbank

Answers

© Emile Woolf International 155 The Institute of Chartered Accountants of Pakistan

5.6 HASAN LIMITED

Hasan Limited

Consolidated statement of financial position as at 31 March 2015

Rs.000 Rs.000

Assets

Non-current assets

Property, plant and equipment (W1) 4,020

Goodwill (W4) 480

Software (W1) 1,440

Investments (65 + 210) 275 –––––––––––––

6,215

Current assets

Inventories (W2) 1,274

Trade receivables (524 + 328) 852

Cash and bank (20 + 55 cash in transit) 75 –––––––––––––

2,201 –––––––––––––

Total assets 8,416 –––––––––––––

Equity and liabilities

Capital and reserves

Equity capital 2,000

Reserves

Share premium 2,000

Retained earnings (W3) 2,420 –––––––––––––

4,420 –––––––––––––

6,420

Non-controlling interest (W5) 350

Non-current liabilities

Government grants (230 + 40) 270

Current liabilities

Trade payables (475 + 472) 947

Operating overdraft 27

Income tax liability (228 + 174) 402 –––––––––––––

1,376 –––––––––––––

Total equity and liabilities 8,416 –––––––––––––

Page 166: CAF7-Financial Accounting and Reporting II_Questionbank

Financial accounting and reporting II

© Emile Woolf International 156 The Institute of Chartered Accountants of Pakistan

Workings

(W1) Property, plant and equipment

Rs.000

Balance from question – Hasan Limited 2,120

Balance from question – Shakeel Limited 1,990

Fair value adjustment on acquisition (see below) (120)

Over-depreciation re fair value adjustment year to 31 March 2015 30 –––––––––––––

4,020 –––––––––––––

A fair value of the leasehold based on the present value of the future rentals (receivable in advance) would be the next (non-discounted) payment of the rental plus the final three years as an annuity at 10%:

Rs.000

PV of rental receipts: Rs.80,000 + (Rs.80,000 2.50) 280

Carrying value on acquisition is (400) –––––––––––––

Fair value reduction of leasehold (120) –––––––––––––

The depreciation of the leasehold in Shakeel Limited’s accounts would be Rs.100,000 per annum. However in the consolidated accounts it should be Rs.70,000 (Rs.280,000/4). This would require a reduction in depreciation of Rs.30,000 in the consolidated accounts for the next four years.

Software:

Shakeel Limited’s accounts

Consolidated figures

Difference

Rs.000 Rs.000

Capitalised amount 2,400 2,400

Depreciation to 31 March 2014 (300) 8 year life (480) 5 year life ––––– ––––– Value at date of acquisition 2,100

1,920

180 fair value adjustment

Depreciation to 31 March 2015 (300)

(480)

180 additional amortisation

––––– ––––– Carrying value 31 March 2015 1,800

1,440

––––– –––––

(W2) Inventories

Rs.000

Amounts given in the question (719 + 560) 1,279

Unrealised profit in inventories (25 25/125) (5) –––––––––––––

1,274 –––––––––––––

Page 167: CAF7-Financial Accounting and Reporting II_Questionbank

Answers

© Emile Woolf International 157 The Institute of Chartered Accountants of Pakistan

(W3) Retained earnings

Rs.000

Retained profits of Shakeel Limited, 31 March 2015 1,955

Adjustments:

Excess charge for leasehold depreciation 30

Insufficient charge for Software amortisation (180)

Unrealised profit in inventory (W2) (5) –––––––––––––

Adjusted retained profits at 31 March 2015 1,800

Retained earnings of Shakeel Limited at 1 April 2014 2,200 –––––––––––––

Shakeel Limited: loss for the year (post-acquisition loss) (400) –––––––––––––

Rs.000

Parent company share of post-acquisition loss (90%) (360)

Hasan Limited reserves at 31 March 2015 2,900

Goodwill impairment (120) –––––––––––––

Consolidated retained profits at 31 March 2015 2,420 –––––––––––––

(W4) Goodwill

Rs.000

At acquisition date

Shares of Shakeel Limited 1,500

Share premium of Shakeel Limited 500

Retained earnings of Shakeel Limited 2,200

Fair value adjustments:

Leasehold (W1) (120)

Software (W1) (180) –––––––––––––

3,900 –––––––––––––

Acquired by Hasan Limited (90%) 3,510

Cost of investment 4,110 –––––––––––––

Goodwill at acquisition 600

Impairment 120 –––––––––––––

Goodwill at 31 March 2015 480 –––––––––––––

Page 168: CAF7-Financial Accounting and Reporting II_Questionbank

Financial accounting and reporting II

© Emile Woolf International 158 The Institute of Chartered Accountants of Pakistan

(W5) Non-controlling interests

Rs.000

Share capital of Shakeel Limited 1,500

Share premium of Shakeel Limited 500

Adjusted retained earnings of Shakeel Limited, 31 March 2015 (W3)

1,800

Fair value adjustments:

Leasehold (120)

Software (180) –––––––––––––

Total net assets at 31 March 2015 3,500 –––––––––––––

Non-controlling interests (10%) 350 –––––––––––––

(W6) Elimination of current accounts:

Rs.000

Shakeel Limited’s current account with Hasan Limited per question

75

Deduct cash in transit regarding this balance (15) –––––––––––––

Adjusted figure to cancel 60 –––––––––––––

(W7) Elimination of intra-group loan:

Rs.000

Investment in Hasan Limited’s books 200

Deduct repayment in transit (40) –––––––––––––

Non-current liability in Shakeel Limited’s books 160 –––––––––––––

Page 169: CAF7-Financial Accounting and Reporting II_Questionbank

Answers

© Emile Woolf International 159 The Institute of Chartered Accountants of Pakistan

CHAPTER 6 – CONSOLIDATED ACCOUNTS: STATEMENTS OF

COMPREHENSIVE INCOME

6.1 HARRY

Consolidated statement of profit or loss for the year ended 31 December 2015

Rs.000

Revenue 1,410 Cost of sales (733) ——– Gross profit 677 Distribution costs (90) Administrative expenses (100) ——– Operating profit 487 Investment income 9 Finance costs (22) ——– Profit before tax 474 Income tax expense (165) ——– Profit after tax 309 Non-controlling interest (W3) (15) ——– Profit 294 ——–

Movement on consolidated retained earnings for the year ended 31 December 2015

Retained earnings at 1 January 2014 (W4) 127 Retained earnings for the year 294 Dividends (50) ——– Retained earnings at 31 December 2015 (W5) 371 ——–

WORKINGS

(1) Group structure

Harry

Sally

75%

Page 170: CAF7-Financial Accounting and Reporting II_Questionbank

Financial accounting and reporting II

© Emile Woolf International 160 The Institute of Chartered Accountants of Pakistan

(2) Consolidated statement of profit or loss

Harry Sally Adj Consol

Rs.000 Rs.000 Rs.000 Rs.000

Revenue 1,120 390 (100) 1,410 C of S – per Q (610) (220) 100 – PURP (3) – – (733) Distribution costs (50) (40) (90) Administrative expenses (55) (45) (100) Investment income (20 – 15) 5 4 9 Interest payable (18) (4) (22) Tax (140) (25) (165) —– PAT 60 —–

(3) Non-controlling interest

Rs.000

25% 60,000 (W1) or as per PAT in question 15 ——

(4) Reserves brought forward

Rs.000

Harry 100

Sally (75% (45 – 9)) 27 —— 127 ——

(5) Reserves carried forward (proof)

Rs.000

Harry 317 PURP (3)

Sally (75% (85 – 9)) 57 —— 371 ——

(6) Inter-company dividend

Rs.000

Payable by Sally 20 —–

Receivable by Harry (75% 20) 15 —–

Page 171: CAF7-Financial Accounting and Reporting II_Questionbank

Answers

© Emile Woolf International 161 The Institute of Chartered Accountants of Pakistan

6.2 HORNY

Consolidated statement of profit or loss for the year ended 31 December 2015 Rs.000

Revenue 362,000 Cost of sales (169,050) ———– Gross profit 192,950 Operating costs (93,817) ———– Operating profit 99,133 Investment income 13,100 Negative goodwill 3,800 ———– Profit before tax 116,033 Income tax (48,400) ———– Profit after tax 67,633 Non-controlling interest (W3) (2,996) ———– Profit 64,637 ———–

Movement on consolidated retained earnings for the year ended 31 December 2015

Rs.000

Retained earnings at 1 January 2015 80,200 Retained profit for the year 64,637 Dividend (20,000) ———– Retained earnings at 31 December 2015 124,837 ———–

WORKINGS

(1) Group structure

Horny

Smooth

75% (acq 31 August 2005)

Page 172: CAF7-Financial Accounting and Reporting II_Questionbank

Financial accounting and reporting II

© Emile Woolf International 162 The Institute of Chartered Accountants of Pakistan

(2) Consolidation schedule

Horny Smooth Adj Consol

4

12

Rs.000 Rs.000 Rs.000 Rs.000

Revenue 304,900 65,100 (8,000) 362,000 Cost of sales (144,200) (32,850) 8,000 (169,050) Operating costs (76,450) (17,367) (93,817) Investment income of H 10,500 of S (all of it) 2,600 13,100 Tax (42,900) (5,500) (48,400) ——— PAT 11,983

(3) Non-controlling interest @ 25% ——– = 2,996 ——–

(4) Consolidated retained earnings carried forward - proof Rs.000

Horny 112,050 Simpson (11,983 – 2,996) 8,987 Negative goodwill 3,800 ——— 124,837 ———

Page 173: CAF7-Financial Accounting and Reporting II_Questionbank

Answers

© Emile Woolf International 163 The Institute of Chartered Accountants of Pakistan

6.3 HERON

Consolidated statement of financial position as at 30 June 2015

Rs.000 Assets Non-current assets Property, plant and equipment (31,000 + 15,000) 46,000

Current assets (23,000 + 11,000) 34,000

——— 80,000 ════

Equity and liabilities Shareholders’ equity Share capital 10,000 Share premium account 5,000

Retained earnings (20,000 + (3

2 18,500)) 32,333

——— 47,333

Non-controlling interest (13 20,000) 6,667

Non-current liabilities 15,000 Current liabilities (5,000 + 6,000) 11,000 ——— 80,000 ════ Consolidated statement of profit or loss for the year ended 30 June 2015

Rs.000

Revenue (30,000 + 25,000) 55,000 Cost of sales (9,000 + 10,000) (19,000) ——— Gross profit 36,000 Distribution costs (3,000 + 1,200) (4,200) Administrative expenses (1,000 + 2,800) (3,800) Finance costs (2,000) ——— Profit before tax 26,000 Income tax expense (3,000 + 3,000) (6,000) ——— Profit for the period 20,000

Non-controlling interest (13 8,000) (2,667)

——— Profit for the financial year attributable to the members of Heron Inc 17,333 ——— Consolidated statement of changes in equity for the year ended 30 June 2015 (extract)

Retained earnings brought forward (8,000 + (23 10,500)) 15,000

Profit for the financial year attributable to the members of Heron Inc 17,333 ——— Retained earnings carried forward 32,333 ———

Page 174: CAF7-Financial Accounting and Reporting II_Questionbank

Financial accounting and reporting II

© Emile Woolf International 164 The Institute of Chartered Accountants of Pakistan

6.4 HANKS

Consolidated statement of financial position as at 31 December 2015

Rs.000 Rs.000 Assets Non-current assets

Property, plant and equipment (32,000 + 25,000 + 20,000 + 6,000) 83,000 Goodwill 4,500

———– 87,500

Current assets Cash at bank and in hand (9,500 + 2,000 + 4,000) 15,500 Receivables (20,000 + 8,000 + 17,000) 45,000 Inventory (30,000 + 18,000 + 18,000 – 2,100) 63,900

———– 124,400

———– Total assets 211,900 ═════ Equity and liabilities Share capital 40,000 Share premium account 6,500 Retained earnings (W5) 88,300 ———–

134,800

Non-controlling interest (W4) 28,100

Current liabilities Trade payables (23,500 + 6,000 + 17,000) 46,500

Proposed dividends – to minority shareholders (2,500 – 2,000) 500 – to Hanks’s shareholders 2,000 ———– 49,000 ———–

Total equity and liabilities 211,900 ═════

Consolidated statement of profit or loss for the year ended 31 December 2015

Rs.000

Revenue (W6) 310,000 Cost of sales (W6) (159,100) ———– Gross profit 150,900 Distribution costs (W6) (51,000) Administrative expenses (W6) (29,500) ———– Profit before taxation 70,400 Tax (W6) (24,000) ———– Profit after taxation 46,400 Non-controlling interest (W6) (9,200) ———– Profit 37,200 ═════

Page 175: CAF7-Financial Accounting and Reporting II_Questionbank

Answers

© Emile Woolf International 165 The Institute of Chartered Accountants of Pakistan

Statement of movements on reserves for the year ended 31 December 2015

Share Share premium Retained Capital account earnings Total Rs.000 Rs.000 Rs.000

At 1 January 2015 40,000 6,500 53,100 (W7) 99,600 Profit for the year – 37,200 37,200 Dividends (proposed) (2,000) (2,000) ——–— –––––– ––––––– ––––––– At 31 December 2015 40,000 6,500 88,300 134,800 ═════ ═════ ═════ ═════

WORKINGS

(1) Group structure

Hanks

Streep Scott

80% 60%

(2) Net assets

Streep

Reporting

date Date of

acquisition

Post acquisition

Rs.000 Rs.000 Share capital 10,000 10,000 Retained earnings

Per question 37,000 Proposed dividend (2,500)

34,500 7,500 27,000

44,500 17,500

Scott

Reporting

date Date of

acquisition

Post acquisition

Rs.000 Rs.000 Share capital 15,000 15,000 Retained earnings 27,000 3,000 24,000 Revaluation reserve 6,000 6,000

48,000 24,000

Page 176: CAF7-Financial Accounting and Reporting II_Questionbank

Financial accounting and reporting II

© Emile Woolf International 166 The Institute of Chartered Accountants of Pakistan

(3) Goodwill on Streep

Rs.000 Cost of shares 20,500

Net assets acquired (80% 17,500) (W2) (14,000) ——— 6,500 ——— Of which: Written off by start of the year (6,500 – 5,000) 1,500 Written off by end of the year (6,500 – 4,500) 2,000 ——— Recognised as impairment during the year (balancing figure) 500 ——— Goodwill on Scott Rs.000 Cost of shares 13,000

Net assets acquired (60% 24,000 (W2)) (14,400) ——— (1,400) ———

(4) Non-controlling interest

Rs.000

Streep (20% 44,500 (W2)) 8,900

Scott (40% 48,000 (W2)) 19,200 ——— 28,100 ———

(5) Consolidated retained earnings c/f

Rs.000

Hanks 55,000 Dividend receivable from Streep (80% of 2,500) 2,000 Proposed dividend (2,000)

Streep (80% 27,000 (W2)) 21,600

Scott (60% 24,000 (W2)) 14,400

PURP ((5,200 + 3,900) 30130

) (2,100)

Goodwill impairment – Streep (2,000) Negative goodwill – Scott 1,400 ——— 88,300 ———

Page 177: CAF7-Financial Accounting and Reporting II_Questionbank

Answers

© Emile Woolf International 167 The Institute of Chartered Accountants of Pakistan

(6) Consolidation schedule

Hanks Streep Scott Adj Consol Rs.000 Rs.000 Rs.000 Rs.000 Rs.000

Sales revenue 125,000 117,000 82,000 (14,000) 310,000 C of S – per Q (65,000) (64,000) (42,000) 14,000 – PURP (W5) (2,100) (159,100) Distrib (21,000) (14,000) (16,000) (51,000) Admin (14,000) (8,000) (7,000) (500) (29,500) Tax (10,000) (9,000) (5,000) (24,000) ——— ——— PAT 22,000 12,000 Non-controlling interest in profit after tax @20% @40% ——– ——– 4,400 + 4,800 = 9,200 ——– ——– ——–

(7) Consolidated retained earnings b/f

Rs.000 Hanks 40,000 Share of post acquisition profits of Streep (80% (15,000 – 7,500)) 6,000 Share of post acquisition profits of Scott (60% (15,000 – 3,000)) 7,200 Goodwill impairment - Streep (1,500) Negative goodwill credited 1,400 ——— 53,100 ———

Page 178: CAF7-Financial Accounting and Reporting II_Questionbank

Financial accounting and reporting II

© Emile Woolf International 168 The Institute of Chartered Accountants of Pakistan

CHAPTER 7 – TANGIBLE NON-CURRENT ASSETS (IAS 16: PROPERTY,

PLANT AND EQUIPMENT AND IAS 23: BORROWING COSTS)

7.1 ROONEY

(a) Borrowing costs

IAS 23 should be applied in accounting for borrowing costs.

Borrowing costs are recognised as an expense in the period in which they are incurred unless they are capitalised in accordance with IAS 23 which says that borrowing costs that are directly attributable to the acquisition, construction or production of a qualifying asset can be capitalised as part of the cost of that asset.

A qualifying asset is an asset that necessarily takes a substantial period of time to get ready for its intended use or sale.

Borrowing costs that are directly attributable to acquisition, construction or production are taken to mean those borrowing costs that would have been avoided if the expenditure on the qualifying asset had not been made.

When an enterprise borrows specifically for the purpose of funding an asset, the identification of the borrowing costs presents no problem as the amount capitalised is the actual borrowing costs net of any income earned on the temporary investment of those borrowings.

If funds are borrowed, generally, the amount of borrowing costs eligible for capitalisation is determined by applying a capitalisation rate to the expenditures on that asset calculated as the weighted average of the borrowing costs applicable to general borrowings.

IAS 23 also contains rules on commencement of capitalisation, suspension of capitalisation and cessation of capitalisation.

Amount capitalised Rs.000

Cost of manufacture 28,000 Interest capitalised (Rs.20m × 5% × 2 years) 2,000 ––––––– 30,000 –––––––

(b) Accounting

Rule

IAS 16 requires that each part of an item (that has a cost that is significant in relation to the total cost) is depreciated separately. Therefore the cost recognised at initial recognition must be allocated to each part accordingly.

Page 179: CAF7-Financial Accounting and Reporting II_Questionbank

Answers

© Emile Woolf International 169 The Institute of Chartered Accountants of Pakistan

Accounting

(i) 31st March 2016

Carrying Depreciation Carrying value value 1.4.2015 31.3.2016

Rs.000 Rs.000 Rs.000 Hydraulic system 9,000 3,000 6,000 “Frame” 21,000 2,625 18,375 –––––– –––––– –––––– 30,000 5,625 24,375 –––––– –––––– –––––– Revaluation loss (to profit and loss)

(3,375)

–––––– Fair value. 21,000 ––––––

The carrying value of the assets should be written down by a factor of 21,000/24,375. This gives a carrying value for the hydraulic system (in Rs.000) of 5,169 and for the ‘frame’ 15,831.

The hydraulic plant should be depreciated over two more years and the

‘frame’ over 7 more years. (ii) 31st March 2017

Carrying Depreciation Carrying value charge value 1.4.2016 31.3.2017

Rs.000 Rs.000 Rs.000 Hydraulic system 5,169 2,585 2,584 “Frame” 15,831 2,262 13,569 ––––––– ––––––– ––––––– 21,000 4,847 16,153 Revalued amount 19,600 ––––––– Total gain 3,447 ––––––– To statement of profit or loss

3,375

Other comprehensive income

72

––––––– Fair value 19,600 –––––––

The total revaluation gain is 3,447. Of this total amount, 3,375 reverses the loss in the previous year and is therefore reported in profit and loss for the year. The remaining 72 is reported as other comprehensive income. (Tutorial note: Deferred tax is ignored by this question.)

Page 180: CAF7-Financial Accounting and Reporting II_Questionbank

Financial accounting and reporting II

© Emile Woolf International 170 The Institute of Chartered Accountants of Pakistan

7.2 EHTISHAM

IAS 16 permits assets to be carried at cost or revaluation. Where the latter is chosen, the asset must be stated at its fair value.

The original depreciation was Rs. 40,000 (Rs. 1,000,000/25 years) per annum.

On 31st March 2014 the asset is two years old. Its carrying value before revaluation was therefore Rs.1million less accumulated depreciation of Rs.80,000 (2/25 × Rs. 1 million).

Rs.

Cost/valuation 1,000,000

Accumulated depreciation (80,000)

Net book value 920,000

In order to effect the revaluation, the cost is uplifted to fair value of Rs.1.15m, the accumulated depreciation is eliminated, and the uplift to the net book value is credited to a revaluation surplus account.

Debit Credit

Cost/valuation 150,000

Accumulated depreciation 80,000

Revaluation surplus 230,000

The impact of the journal is as follows:

Before Adjustment After

Cost/valuation 1,000,000 150,000 1,150,000

Accumulated depreciation (80,000) 80,000 nil

Net book value 920,000 1,150,000

The asset is depreciated over its remaining useful economic life of 23 years giving a charge of Rs. 50,000 (Rs. 1,150,000/23 years) per annum in the year to 31st March 2015.

Debit Credit

Statement of profit or loss 50,000

Accumulated depreciation 50,000

This results in a carrying value as at 31st March 2015 of:

Rs.

Cost/valuation 1,150,000

Accumulated depreciation (50,000)

Net book value 1,100,000

Transfer from revaluation surplus to retained earnings

As a result of the revaluation, the annual depreciation has increased from Rs.40,000 to Rs.50,000. This extra depreciation of Rs.10,000 is transferred from the revaluation reserve to accumulated profits each year.

Debit Credit

Revaluation surplus 10,000

Accumulated profits 10,000

Page 181: CAF7-Financial Accounting and Reporting II_Questionbank

Answers

© Emile Woolf International 171 The Institute of Chartered Accountants of Pakistan

By the 31st March 2015, the balance remaining on the revaluation reserve will be Rs.220,000.

Rs.

Surplus recognised at 31 March 2014 230,000

Transfer to accumulated profits (10,000)

Net book value 220,000

The fall in property values at the year-end. The asset must be revalued downwards to Rs.0.8million, a write-down of Rs.300,000.

Rs.220,000 of this is charged against the revaluation reserve relating to this asset, and the remaining Rs.80,000 must be charged against profits.

The reduction of the carrying amount of the asset is achieved by removing the accumulated depreciation and adjusting the asset account by the balance.

Debit Credit

Revaluation surplus 220,000

Statement of profit or loss 80,000

Asset at valuation 350,000

Accumulated depreciation 50,000

The impact of the journal is as follows:

Before Adjustment After

Cost/valuation 1,150,000 350,000 800,000

Accumulated depreciation (50,000) 50,000 nil

Net book value 1,100,000 800,000

This balance is depreciated over the remaining useful life of the asset (22 years).

Page 182: CAF7-Financial Accounting and Reporting II_Questionbank

Financial accounting and reporting II

© Emile Woolf International 172 The Institute of Chartered Accountants of Pakistan

7.3 CARLY

Financial statements for the year ended 31 December 2015 (extract)

Property, plant and equipment

Land and buildings

Plant and machinery

Computer equipment

Total

Rs. Rs. Rs. Rs.

Cost/valuation At 1 January 2015 1,500,000 340,500 617,800 2,458,300 Revaluation 250,000 - - 250,000 Additions (W2) - 17,550 - 17,550 Disposals - (80,000) - (80,000) –––––––––– –––––––– –––––––– –––––––––– At 31 December 2015 1,750,000 278,050 617,800 2,645,850 –––––––––– –––––––– –––––––– –––––––––– Accumulated depreciation At 1 January 2015 600,000 125,900 505,800 1,231,700 Charge for the year (W1) 20,000 51,191 44,800 115,991 Revaluation (620,000) - - (620,000) Disposals - (57,000) - (57,000) –––––––––– –––––––– –––––––– –––––––––– At 31 December 2015 nil 120,091 550,600 670,691 –––––––––– –––––––– –––––––– –––––––––– Carrying amount At 31 December 2014 900,000 214,600 112,000 1,226,600 –––––––––– –––––––– –––––––– –––––––––– At 31 December 2015 1,750,000 157,959 67,200 1,975,159 –––––––––– –––––––– –––––––– ––––––––––

Workings

(1) Depreciation charges

Buildings = (1,500,000 – 500,000) 2% = 20,000.

Plant and machinery: Rs.

New machine (17,550 25% 9/12) 3,291

Existing plant (((340,500 – 80,000) – (125,900 – 57,000)) 25%) 47,900 ––––––– 51,191 –––––––

Computer equipment = 112,000 40% = Rs.44,800

(2) Cost of new machine Rs.

Purchase price (20,000 – 3,000 – 1,000) 16,000 Delivery costs 500 Installation costs 750 Interest on loan taken out to finance the purchase 300 ––––––– 17,550 –––––––

Page 183: CAF7-Financial Accounting and Reporting II_Questionbank

Answers

© Emile Woolf International 173 The Institute of Chartered Accountants of Pakistan

7.4 ADJUSTMENTS LIMITED

(a) Lathe

The lathe was purchased in 2009 and was originally being written off over an estimated useful life of twelve years. As at 1 January 2015 six of the years have elapsed with a further six years remaining. It was decided that the machine will now only be usable for a further four years.

IAS 16 Property, plant and equipment requires that where the original estimate of useful life is revised, adjustments should be made in current and future periods (not in prior periods). The unamortised cost of the asset should be charged to revenue over the remaining useful life of the asset. The net book value of Rs.75,000 should therefore be charged over the remaining four years of useful life, giving an annual depreciation charge of Rs.18,750.

The revision is not a change in accounting policy, or a fundamental error but a change in accounting estimate. It is therefore not appropriate to deal with any excess depreciation by adjusting opening retained earnings.

(b) Grinder

The grinder was purchased in 2012 and was originally being depreciated on a straight line basis. It has now been decided to depreciate this on the sum of digits basis.

IAS 16 requires that depreciation methods be reviewed periodically and if there is a significant change in the expected pattern of economic benefits, the method should be changed. Depreciation adjustments should be made in current and future periods. This change might be appropriate if, for instance, usage of the machine is greater in the early years of an asset’s life when it is still new and consequently it is appropriate to have a higher depreciation charge.

If the change is implemented, the unamortised cost (the net book value) of the asset should be written off over the remaining useful life commencing with the period in which the change is made. The depreciation charge for the remaining life of the asset will therefore be as follows.

Year No of digits Depreciation Rs.

2015 7 7/28 Rs.70,000 17,500

2016 6 6/28 Rs.70,000 15,000 2017 5 12,500 2018 4 10,000 2019 3 7,500 2020 2 5,000 2021 1 2,500 —— ———–

1/2 7 (7 + 1) 28 Rs. 70,000 —— ———–

Disclosure will need to be made in the accounts of the details of the change, including the effect on the charge in the year.

Page 184: CAF7-Financial Accounting and Reporting II_Questionbank

Financial accounting and reporting II

© Emile Woolf International 174 The Institute of Chartered Accountants of Pakistan

(c) Leasehold land

IAS 16’s allowed alternative treatment in respect of measurement of property plant and equipment (subsequent to initial recognition), is that of revaluation. Revaluation is made at fair value.

Where any item of property plant or equipment is revalued, the entire class to which the asset belongs should be revalued. Revaluations must be kept up to date. Where there are volatile movements in fair value, the revaluation should be performed annually. Where there are no such movements, revaluations every three to five years may be appropriate.

Accumulated depreciation at the date of revaluation is either

(i) restated proportionately with the change in the gross carrying amount so that the carrying amount after the revaluation equals the revalued amount (e.g. where revaluations are made to depreciated replacement cost using indices)

(ii) eliminated against the gross carrying amount of the assets and the net amount restated to the revalued amount of the asset (e.g. where buildings are revalued to their market value).

IAS 16 requires that the subsequent charge for depreciation should be based on the revalued amount. The annual depreciation will therefore be Rs.62,500, i.e. Rs.1,500,000 divided by the 24 years of remaining life.

There will then be a difference between the revalued depreciation charge and the historical depreciation charge.

The resulting excess depreciation may be dealt with by a movement in reserves, i.e. by transferring from the revaluation reserve to retained earnings a figure equal to the depreciation charged on the revaluation surplus each year.

Page 185: CAF7-Financial Accounting and Reporting II_Questionbank

Answers

© Emile Woolf International 175 The Institute of Chartered Accountants of Pakistan

7.5 FAM

Accounting policies

(a) Property, plant and equipment is stated at historical cost less depreciation, or at valuation.

(b) Depreciation is provided on all assets, except land, and is calculated to write down the cost or valuation over the estimated useful life of the asset.

The principal rates are as follows.

Buildings 2% pa straight line Plant and machinery 20% pa straight line Fixtures and fittings 25% pa reducing balance

Fixed asset movements Land Plant Fixtures, Payments on

and and fittings, account and buildings machinery tools and assets in the Total equipment course of construction

Cost/valuation Rs.000 Rs.000 Rs.000 Rs.000 Rs.000

Cost at 1 January 2015 900 1,613 390 91 2,994 Revaluation adjustment 600 – – – 600 Additions – 154 40 73 (W1) 267 Reclassifications 100 – – (100) – Disposals – (277) (41) – (318) ——— ——— —— —— ——— Cost at 31 December 2015 100 1,490 389 64 2,043 2015 valuation 1,500 1,500 ——— ——— —— —— ——— Depreciation At 1 January 2015 80 458 140 – 678 Revaluation adjustment (80) – – – (80) Provisions for year (W2) 17 298 70 – 385 Disposals – (195) (31) – (226) —— —— —— —— —— At 31 December 2015 17 561 179 – 757 —— —— —— —— —— Net book value At 31 December 2015 1,583 929 210 64 2,786 ——— ——— —— —— ——— At 31 December 2014 820 1,155 250 91 2,316 ——— ——— —— —— ———

Land and buildings have been revalued during the year by Messrs Jackson & Co on the basis of an existing use value on the open market.

Page 186: CAF7-Financial Accounting and Reporting II_Questionbank

Financial accounting and reporting II

© Emile Woolf International 176 The Institute of Chartered Accountants of Pakistan

The corresponding historical cost information is as follows.

Land and buildings Rs.000 Cost Brought forward 900 Reclassification 100 ——— Carried forward 1,000 ——— Depreciation Brought forward 80 Provided in year 10 ——— Carried forward 90 ——— Net book value 910 ———

WORKINGS

Rs.000 (1) Additions to assets under construction 53 Deposit on computer 20 —— 73 —— Rs.000

(2) Depreciation on buildings 60040

+ (100 2%) 17

2% straight line depreciation is equivalent to a 50 year life. The buildings are ten years old at valuation and therefore have 40 years remaining.

Depreciation on plant (1,613 + 154 – 277) 20% 298

Depreciation on fixtures (390 + 40 – 41 – 140 + 31) 25% 70

Page 187: CAF7-Financial Accounting and Reporting II_Questionbank

Answers

© Emile Woolf International 177 The Institute of Chartered Accountants of Pakistan

7.6 IMRAN LIMITED

(a) Specific borrowings

Rs.

Borrowing costs incurred:

13% bank loan outstanding for 10 months (Rs. 32 million x 306/365 x 13%) 3,487,562 11% bank loan outstanding for 5 months (Rs. 10 million x 153/365 x 11%) 461,096

Borrowing costs 3,948,658 Less: Interest that relates to suspension

13% bank loan: (Rs. 32 million x 61/365 x 13%) 695,233 11% bank loan (Rs. 10 million x 61/365 x 11%) 183,836

(879,068)

3,069,590

Less: Investment income on temporary investment of the borrowings (500,000)

2,569,590

(b) General borrowings

Phase 1 Phase 2 Phase 3

Building cost capitalised 20,000,000 18,000,000 16,000,000 Financed out of rights issue (15,000,000) Financed from borrowing 5,000,000 18,000,000 16,000,000 Period to the year end March 1 to December 31 306 April 1 to December 31 275 October 1 to December 31 92 Period of suspension (61) (61) Number of days for which

borrowing should be capitalised 245 214 92 Weighted average borrowing

rate (W3) 12.73% 12.73% 12.73% Fraction of the year for which

the rate should be applied to costs incurred 245/365 214/365 92/365

Capitalised borrowing 427,240 1,343,451 513,385 Total 2,284,076

Page 188: CAF7-Financial Accounting and Reporting II_Questionbank

Financial accounting and reporting II

© Emile Woolf International 178 The Institute of Chartered Accountants of Pakistan

Workings W1: Average borrowings Rs.m

13% bank loan outstanding for 10 months (Rs. 32 million x 306/365 days) 26,827,397

11% bank loan outstanding for 5 months (Rs. 10 million x 153/365 days) 4,191,781

Average outstanding for the year 31,019,178

W2: Borrowing costs incurred (or from part a) Rs.m

13% bank loan outstanding for 10 months (Rs. 32 million x 306/365 x 13%) 3,487,562 11% bank loan outstanding for 5 months (Rs. 10 million x 153/365 x 11%) 461,096

Borrowing costs 3,948,658

W3: Weighted average rate

Borrowing costs/ Average outstanding for the year = 3,948,658 (W2)/31,019,178 (W1) = 12.73%

Page 189: CAF7-Financial Accounting and Reporting II_Questionbank

Answers

© Emile Woolf International 179 The Institute of Chartered Accountants of Pakistan

7.7 HUMAYUN CHEMICALS LIMITED

(a) If review is performed on June 30, 2015 Cost of machine Rs. 10,000,000

Depreciation charged @ 20% for the year ended June 30, 2013 and June 30, 2014

(Rs. 10,000,000 Rs. 3,000,000) x 20% x 2 Rs. 2,800,000

WDV as at June 30, 2014 Rs. 7,200,000 Residual value (10% of the cost of machine) Rs. 1,000,000

Depreciable amount - on July 1, 2014 Rs. 6,200,000

Remaining useful lives 6 years

Depreciation charge for the year ended June 30, 2015 Rs. 1,033,333

If review is performed on June 30, 2014

Cost of machine Rs. 10,000,000

Depreciation for the year ended June 30, 2013 (Rs. 10,000,000 - Rs. 3,000,000) x 20% Rs. 1,400,000

WDV as at June 30, 2013 Rs. 8,600,000 Residual value (10% of the cost of machine) Rs. 1,000,000

Depreciable amount - on July 1, 2013 Rs. 7,600,000

Remaining useful lives 6 years

Depreciation charge for the year ended June 30, 2015 Rs. 1,266,667

Depreciation charged in the financial statement for the year ended June 30, 2014 Rs. 1,400,000

Effect of change in estimate to be incorporated (Reversal)

[Rs. 1,400,000 1,266,667) (Rs. 133,333)

(b) According to IAS-16, the following factors should be considered when estimating the useful life of a depreciable asset:

(i) Expected usage

(ii) Expected physical wear and tear

(iii) Obsolescence

(iv) Legal or other limits on the use of the assets.

Once the useful life of a depreciable asset is determined, it shall be reviewed at least at each financial year-end.

If expectations vary from the previous estimates, then change should be adjusted for current and future periods in accordance with the requirements of IAS 8.

Page 190: CAF7-Financial Accounting and Reporting II_Questionbank

Financial accounting and reporting II

© Emile Woolf International 180 The Institute of Chartered Accountants of Pakistan

7.8 FARADAY PHARMACEUTICAL LIMITED

Date Particulars

Debit Credit

Rs.000 Rs.000

01.07.2011 Building 200,000

Bank 200,000

(Record purchase of plant)

30.06.2012 Depreciation 10,000

Accumulated depreciation – Building 10,000

(Record depreciation for the year 2012)

Working: Rs. 200,000 ÷ 20 = Rs. 10,000

01.07.2012 Accumulated depreciation – Building 10,000

Building 10,000

(Reversal of prior year depreciation)

01.07.2012 Building 40,000

Surplus on revaluation of fixed assets 40,000

(Increase in value through revaluation)

Working: Rs. 230,000 – Rs. 190,000 = Rs. 40,000

30.06.2013 Depreciation 12,105

Accumulated depreciation – Building 12,105

(Record depreciation for the year 2013)

Working: Rs. 230,000 ÷ 19 = Rs. 12,105

30.06.2013 Surplus on revaluation of fixed assets 2,105

Retained earnings/Profit & loss account 2,105

(transfer of surplus through retained earning to the extent of excess depreciation)

Working: Rs. 40,000 ÷ 19 = Rs. 2,105 01.07.2013 Accumulated depreciation – Building 12,105

Building 12,105

(Reversal of prior year depreciation)

01.07.2013 Surplus on revaluation of fixed assets 37,895

Revaluation expense 10,000

Building 47,895

(Decrease in value through revaluation)

Working:

Reversal of Surplus balance (Rs. 40,000 – Rs. 2,105) Rs. 37,895. Balancing figure of Rs. 10,000 charged to Profit and Loss

Building value decline: (Rs. 230,000 – Rs. 12,105) – Rs. 170,000 =Rs. 47,895

30.06.2014 Depreciation 9,444

Accumulated depreciation – Building 9,444

(Record depreciation for the year 2014)

Working: Rs. 170,000 ÷ 18 = Rs. 9,444

01.07.2014 Accumulated depreciation – Building 9,444

Building 9,444

(Reversal of prior year depreciation)

Page 191: CAF7-Financial Accounting and Reporting II_Questionbank

Answers

© Emile Woolf International 181 The Institute of Chartered Accountants of Pakistan

Date Particulars

Debit Credit

Rs.000 Rs.000

01.07.2014 Building 19,444

Revaluation income 9,444

Surplus on revaluation of fixed assets (balancing) 10,000

(Reversal of prior year impairment)

Working:

Revaluation income = Rs. 10,000 – [ Rs. 10,000 – Rs. 9,444] = Rs. 9,444

Building: [Rs. 170,000 – Rs. 9,444] – Rs. 180,000 =Rs. 19,444

30.06.2015 Depreciation 10,588

Accumulated depreciation – Building 10,588

(Record depreciation for the year 2015)

Working: Rs. 180,000 ÷ 17 = Rs. 10,588

30.06.2015 Surplus on revaluation of fixed assets 588

Retained earnings 588

(Reverse the excess depreciation)

Working: Rs. 10,000 ÷ 17 = Rs. 588

Page 192: CAF7-Financial Accounting and Reporting II_Questionbank

Financial accounting and reporting II

© Emile Woolf International 182 The Institute of Chartered Accountants of Pakistan

7.9 SPIN INDUSTRIES LIMITED

Rupees Commitment fee 125,000 Actual borrowing costs of specific loan (W1) 2,050,000 General borrowing costs (W1) 1,175,283 Less: Investment income (W2) (137,500)

Interest costs to be capitalised 3,212,783

W1 Outstanding

amount (Rs.) Months outstanding

Outstanding month up to completion

Rate of interest

Borrowing cost (Rs.)

Specific loan Utilised till first repayment 25,000,000 1-Sep-14 31-Jan-15 5 12% 1,250,000 Utilised after the first

repayment 20,000,000 1-Feb-15 31-May-15 4 12% 800,000

2,050,000

General Borrowings (W4) Utilised after specific loan

exhausted on 2nd

payment to contractor (W3) 8,125,000 1-Dec-14 31-May-15 6 12.08% 490,750

Principal payment of specific loan 5,000,000 1-Feb-15 31-May-15 4

12.08% 201,333

3rd payment to contractor 12,000,000 1-Feb-15 31-May-15 4 12.08% 483,200

4rd payment to contractor 9,000,000 1-Jun-15 31-May-15 0 12.08% -

1,175,283

W2: Investment income Surplus fund available from 1-Sep-14 to 30-Nov-14 (Rs. 25m – Rs.

0.125m – Rs. 8m – Rs. 10m) × 8% × 3/12 Rs.137,500

W3: Specific loan utilization Commitment fee 125,000 Payment for obtaining permit 8,000,000 1st payment to contractor 10,000,000 2nd payment to contractor (balancing) 6,875,000

25,000,000

2nd payment to contractor (total) 15,000,000 Less: paid out of specific loan (as worked out above) 6,875,000

Paid from general borrowing 8,125,000

W4: Weighted average rate of borrowing

Weighted average amount of loan (Rs.)

Interest

(Rs.) From Bank A 25,000,000 Rs. 25,000,000 × 13% × 9/12 2,437,500 From Bank B 20,000,000 3,000,000

45,000,000 5,437,500

Weighted average rate of borrowing (Rs. 5,437,500 / 45,000,000) 12.08%

Page 193: CAF7-Financial Accounting and Reporting II_Questionbank

Answers

© Emile Woolf International 183 The Institute of Chartered Accountants of Pakistan

7.10 SCIENTIFIC PHARMA LIMITED

Scientific Pharma Limited

Journal entries for the year ended June 30, 2015

Debit Credit

Rs.000 Rs.000

30.06.2015 Repair and maintenance expenses 1,500

Account payable / Bank

1,500

(Repair cost of major break down of the plant)

30.06.2015 Depreciation expense (45,000-2,000)/10.5 years 4,095

Accumulated depreciation

4,095

(Depreciation expense for the year)

30.06.2015 Revaluation surplus (10,380/10.5)

989

Retained earnings

989

(Incremental depreciation credited to retained earnings)

30.06.2015 Impairment loss W1 5,296

Property, plant and equipment

5,296

(Impairment of plant due to break down)

30.06.2015 Revaluation surplus

5,296

Impairment loss W1

5,296

(Impairment loss adjusted against revaluation)

W1: Impairment loss

Recoverable amount 19,227

WDV of the plant on impairment date W2 (24,523)

Impairment loss as on 30.06.2015 (5,296)

W2: WDV of the plant on impairment date

Rs.000

FOB price (US$ 800,000 at Rs. 52)

41,600

Other charges including installation cost

7,000

48,600

Accumulated depreciation (1-1-2006 to 30-6-2010) {(48,600-2,000)/15*4.5} (13,980)

WDV as on 30-6-2010

34,620

Revaluation surplus (45,000-34,620)

10,380

Revalued amount as of July 1, 2010

45,000

Accumulated depreciation (1-7-2010 to 30-6-2015) {(45,000-2,000)/10.5*5) (20,476)

WDV as on 30-6-2015

24,523

W3: Revaluation surplus on impairment date

Revaluation surplus W2 10,380

Transferred to retained earnings (01.07.2010 to 30.06.2015) (10,380/10.5*5) (4,943)

Revaluation surplus balance on impairment date

5,437

Since impairment loss is less than the revaluation surplus on impairment date, the full amount of impairment would be adjusted against the revaluation surplus.

Page 194: CAF7-Financial Accounting and Reporting II_Questionbank

Financial accounting and reporting II

© Emile Woolf International 184 The Institute of Chartered Accountants of Pakistan

7.11 QURESHI STEEL LIMITED

Capital work in progress – Factory building Rs.000

Progress invoices received from the contractor (30,000+20,000+10,000+15,000) 75,000.00

(Rain damages paid would be chargeable to profit and loss account/ insurance claim)

Borrowing costs to be capitalised:

Loan processing charges 500.00

Interest on bank loan W1 1,841.67 Interest on running finance W2 2,730.00

Interest income from surplus loan amount W4 (395.00)

Capital work in progress – June 30, 2015 79,676.67

W1: Interest on bank loan:

Rs.000

Interest amount Outstanding loan amount

Interest at 13% From To Months

01-14-2014 31-05-2015 6 25,000 1625.00

01-06-2015 30-06 -2015 1 20,000 216.67

1,841.67

W2: Interest on running finance

Rs.000

Payments date

Description Invoice amount

Payments net of

deductions

Payments from Months

outstanding up to

30-6-10

Interest at 15%

per annum

(W3)

Right issue

Bank loan

Running finance

01-07-14 Advanced payment 10,000 10,000 10,000 12.00 1,500

15-10-14 1st progress

bill 30,000 25,500 15,000 10,500 8.50 1,116

15-01-15 2nd

progress bill 20,000 17,000 17,000 - - -

15-04-15 3rd

progress bill 10,000 8,500 7,500 1,000 2.50 31

31-05-15 Loan interest 1,625 1,625 1.00 20

31-05-15 Loan instalment 5,000 5,000 1.00 63

15,000 *24,500 29,125 2,730

*Loan amount of Rs. 25,000,000 less processing charges of Rs. 500,000

W3: Average rate of interest for running finance facility (9,000/60,000) 15%

W4: Interest income from surplus loan amounts: Rs.000

Interest income Surplus loan

amounts

Interest income at 8% From To Months

01-14-14 15-01-15 1.5 24,500 (245)

16-01-15 15-04-15 3.0 7,500 (150)

(395)

Page 195: CAF7-Financial Accounting and Reporting II_Questionbank

Answers

© Emile Woolf International 185 The Institute of Chartered Accountants of Pakistan

7.12 GRANITE CORPORATION

Borrowing costs to be capitalised Workings 2015 2014 Commitment fee @ 1% - 700,000 Borrowing costs on specific loan 1 6,987,500 3,033,333 Borrowing costs on running finance 3 1,381,625 -

Less: Investment income 2

(2,099,001)

(1,381,334)

6,720,124 2,351,999

W1 : Actual borrowing costs on specific loan

Ou

tsta

nd

ing

am

ou

nt

(R

s.)

Ou

tsta

nd

ing

mo

nth

Su

sp

en

sio

n

Net

ou

tsta

nd

ing

mo

nth

s

Bo

rro

win

g c

os

t

to b

e c

ap

itali

sed

(R

s.)

@ 1

3%

From commencement on to June 30 70,000,000 4 0 3,033,333

Amount to be capitalised as on 30-Jun-2014 3,033,333

From June 30 to first principal repayment 70,000,000 2 0 2 1,516,667

After the 1st principal repayment 65,000,000 6 1 5 3,520,833

After the 2nd principal repayment to completion 60,000,000 3 0 3 1,950,000

Amount to be capitalised as on 30-Jun-2015 6,987,500

W2 : Investment income (All amounts in Rupees)

Available Funds

O/s amount up to completion

Used to reduce running finance (14%)

Invested in saving account @ 8%

Total Income

Amount Income Amount Income

Rs.(70m - 25m - 0.7m) 44,300,000 4 10,000,000 466,667 34,300,000 914,667 1,381,334

Investment income – 2014 1,381,334

Rs. (70m - 25m - 0.7m) 44,300,000 2 10,000,000 233,333 34,300,000 457,333 690,666

Rs.(44.3 - 5m - 4.55m) 34,750,000 5 10,000,000 583,335 24,750,000 825,000 1,408,335

Investment income – 2015 2,099,001

W3 : Interest on running finance

Description Amount

2015

Bo

rro

win

g c

os

t

to b

e c

ap

itali

sed

(Rs.)

@ 1

4%

No

. o

f m

on

ths

ou

tsta

nd

ing

Su

sp

en

sio

n

Net

ou

tsta

nd

ing

mo

nth

s

2nd payment to contractor (Rs. 65m - 34.75m) 30,250,000 4 1 3 1,058,750

Payment of 2nd instalment

Principal 5,000,000 3 0 3 175,000

Interest (Rs. 65m x 13% x 6/12) 4,225,000 3 0 3 147,875

3rd payment to contractor 10,000,000 0 0 0 -

49,475,000 1,381,625

Page 196: CAF7-Financial Accounting and Reporting II_Questionbank

Financial accounting and reporting II

© Emile Woolf International 186 The Institute of Chartered Accountants of Pakistan

CHAPTER 8 – IAS 38: INTANGIBLE ASSETS

8.1 FAZAL

In accordance with IAS 38, expenditure on intangible assets must be expensed unless it meets the recognition criteria for capitalisation. These criteria require the demonstration that future benefits will arise from the incurred costs. It would be difficult to prove that this is the case in relation to training costs and IAS 38 specifically states that training costs should always be expensed as they are incurred and not treated as an intangible asset.

Hence the treatment adopted by Fazal is not correct and the costs being carried forward must be expensed to the year’s profits.

8.2 HENRY

Property, plant and equipment

Plant and machinery

Cost Rs. On 1 January 2015 X Additions 30,000 ––––––– On 31 December 2015 X ––––––– Accumulated depreciation On 1 January 2015 X

Charge for the year (30,000 9/12 ÷ 5) 4,500 ––––––– On 31 December 2015 X ––––––– Carrying amount On 31 December 2014 X ––––––– On 31 December 2015 25,500 –––––––

Intangible assets

Internally generated research and development expenditure

Cost Rs. On 1 January 2015 412,500 Additions 45,000 –––––––– On 31 December 2015 457,500 –––––––– Accumulated amortisation On 1 January 2015 - Charge for the year (W) 68,750 –––––––– On 31 December 2015 68,750 –––––––– Carrying amount On 31 December 2014 412,500 –––––––– On 31 December 2015 388,750 ––––––––

Page 197: CAF7-Financial Accounting and Reporting II_Questionbank

Answers

© Emile Woolf International 187 The Institute of Chartered Accountants of Pakistan

Working

Amortisation charge (Project A) Rs.

Total savings (100,000 + 300,000 + 200,000) 600,000

2015 amortisation charge (100,000/600,000 412,500) 68,750

Tutorial notes

The costs in respect of Project B cannot be capitalised as there are uncertainties surrounding the successful outcome of the project – but the machine bought may be capitalised in accordance with IAS16.

The 2015 costs in respect of Project C can be capitalised as the uncertainties have now been resolved. However, the 2014 costs cannot be reinstated.

8.3 TOBY

Intangible assets

Goodwill Patents Brands Total

Rs. Rs. Rs. Rs. Cost On 1 January 2015 - - - - Additions (W1) 10,000 20,000 50,000 80,000 ––––––– ––––––– ––––––– ––––––– On 31 December 2015 10,000 20,000 50,000 80,000 ––––––– ––––––– ––––––– ––––––– Accumulated amortisation/impairment On 1 January 2015 - - - - Written off/amortised during the year (W1 and W2) 3,000 2,500 7,500 13,000 ––––––– ––––––– ––––––– ––––––– On 31 December 2015 3,000 2,500 7,500 13,000 ––––––– ––––––– ––––––– ––––––– Carrying amount On 31 December Year 0 - - - - ––––––– ––––––– ––––––– ––––––– On 31 December 2015 7,000 17,500 42,500 67,000 ––––––– ––––––– ––––––– –––––––

Workings

(1) Goodwill on acquisition of George

Rs.

Cost of acquisition 105,000 Minus fair value of net assets acquired (100,000 – 5,000) (95,000) –––––––– Goodwill 10,000 Recoverable value (7,000) –––––––– Impairment write off 3,000 ––––––––

(2) Amortisation of patent

20,000 ÷ 8 = Rs.2,500

(3) Amortisation of brand

50,000 ÷ 5 × 9/12 = Rs.7,500

Tutorial note

IAS38 Intangible assets prohibits the recognition of internally generated brands (3) or internally-generated goodwill (4).

Page 198: CAF7-Financial Accounting and Reporting II_Questionbank

Financial accounting and reporting II

© Emile Woolf International 188 The Institute of Chartered Accountants of Pakistan

8.4 BROOKLYN

1 Development expenditure

IAS 38 on intangibles requires that research and development be considered separately:

research – which must be expensed as incurred

development – which must be capitalised where certain criteria are met.

It must first be clarified how much of the Rs.3 million incurred to date (10 months at Rs.300,000) is simply research and how much is development. The development element will only be capitalised where the IAS 38 criteria are met. The criteria are listed below together with the extent to which they appear to be met.

The project must be believed to be technically feasible. This appears to be so as the feasibility has been acknowledged.

There must be an intention to complete and use/sell the intangible. Completion is scheduled for June 2016

The entity must be able to use or sell the intangible. Interest has been expressed in purchasing the knoWhow on completion

It must be considered that the asset will generate probable future benefits. Confirmation is required from Brooklyn as to the extent of interest shown by the pharmaceutical companies and whether this is of a sufficient level to generate orders and to cover the deferred costs.

Availability of adequate financial and technical resources must exist to complete the project. The financial position of Brooklyn must be investigated. A grant is being obtained to fund further work and the terms of the grant, together with any conditions, must be discussed further.

Able to identify and measure the expenditure incurred. A separate nominal ledger account has been set up to track the expenditure.

If all of the above criteria are met, then the development element of the Rs.3m incurred to date must be capitalised as an intangible asset. Amortisation will not begin until commercial production commences.

2 Provision

Although the claim was made after the reporting period, IAS 10 considers this to be an adjusting event after the reporting period. The employment of the individual dates back to 20X2 and so the lawsuit constitutes a current obligation for the payment of damages as a result of this past event (the employment).

The amount and the timing are not precisely known but the likelihood of payment of damages by Brooklyn is probable and so a provision should be made for the estimated amount of the liability, as advised by the lawyer. Disclosure, rather than provision, would only be appropriate if the expected settlement was possible or remote, and the lawyer’s view is that a payment is more likely than not.

It is not appropriate to calculate an expected value where there is only one event, instead a provision should be made for the most likely outcome. The lawyer has various views on the possible payout, but the most likely payout is Rs.500,000 as this has a 50% probability. As settlement of the provision is not anticipated until 2018, the provision should be discounted back at 8% to give a liability of Rs.476,280.

Page 199: CAF7-Financial Accounting and Reporting II_Questionbank

Answers

© Emile Woolf International 189 The Institute of Chartered Accountants of Pakistan

Provided that the payment from the insurance company is virtually certain, this should be shown as an asset, also at its discounted value of Rs.47,628, being 10% of the provision.

In both cases the discounting should be unwound over the coming three years through profit or loss.

3 Revaluation

IAS 16 on Property, Plant and Equipment does not impose a frequency for updating revaluations. It simply requires a revaluation where it is believed that the fair value of the asset has materially changed. Hence, if in the past there have been material differences between the carrying amount and fair value at the 5 yearly review then Brooklyn should consider having more frequent valuations following on from this year’s valuation.

Revaluations should be regular and not timed simply when property prices are at a peak. It is not acceptable for Brooklyn to defer its next revaluation while values are low. If property prices do fall in 2016, then it may be necessary to perform an impairment test in accordance with IAS 36 Impairment of assets.

If it is believed that an asset value has moved materially, then all assets in that class must be revalued. Hence it is not sufficient for Brooklyn to just revalue the London property.

IAS 16 does not require the valuation to be performed by an external party, and so the use of the property manager to conduct the valuations is acceptable. Notes to the financial statements will disclose that he is not independent of the company.

8.5 ZOUQ INC

(a) (i) The depreciable amount of an intangible asset with a finite useful life shall be allocated on a systematic basis over its useful life.

(ii) Amortization shall begin when the asset is available for use

(iii) Amortization shall cease at the earlier of the date that the asset is

classified as held for sale and the date that the asset is derecognised.

(iv) The amortization method used shall reflect the pattern in which the asset's future economic benefits are expected to be consumed by the entity.

(v) The amortization charge for each period shall be recognised in

statement of profit or loss. (b) Goodwill Account Rupees Rupees

01.01.2014 Goodwill recognised (W1) 270,000,000 31.12.2014

Impairment of goodwill 50,000,000

31.12.2014 Balance b/d 220,000,000

270,000,000 270,000,000

01.01.2015 Balance b/d 220,000,000

31.12.2015 Balance b/d 220,000,000

220,000,000 220,000,000

Page 200: CAF7-Financial Accounting and Reporting II_Questionbank

Financial accounting and reporting II

© Emile Woolf International 190 The Institute of Chartered Accountants of Pakistan

Brand Account Rupees Rupees

01.01.2014 Brand

recognised 100,000,000 31.12.2014 Amortization 10,000,000

31.12.2014 Balance c/d 90,000,000

100,000,000 100,000,000

01.01.2015 Balance b/d 90,000,000 31.12.2015 Amortization 10,000,000

31.12.2015 Impairment of Brand 13,500,000

- 31.12.2015 Balance c/d 68,000,000

90,000,000 90,000,000

W1: Value of goodwill Rupees Purchase price (50,000,000 x Rs. 30 x 90%) 1,350,000,000

Less: Fair value of net identifiable assets and liabilities (Rs. 1,100,000,000 x 90%) (990,000,000)

Less: Value of brand (Rs. 100,000,000 x 90%) (90,000,000)

Goodwill recognised 270,000,000

8.6 STAR-BRIGHT PHARMACEUTICAL LIMITED

Star-Bright Pharmaceutical Limited Statement of financial position As at December 31, 2015

2015

2014 Restated

Rs. in million Non-current assets

Intangible asset – brand [Note 8] 274 285 Shareholders’ equity Retained earnings (W5 and 6) 2,071 1,879

Star-Bright Pharmaceutical Limited

Statement of Financial Position

As at December 31, 2015

8- Intangible assets – Brand Cost

At beginning of the year (2015: 382+24+54+38, 2014: 382+ 24+54) 498 460 Capitalised during the year 43 38 541 498 Amortization

At beginning of the year (W1 and 2) (213) (163) During the year (W3 and 4)

*3 (54)

*4 (50)

(267) (213)

274 285

W1 : 382 x 50% + 24 x 30% + 54 x 20% + 38 x 10% = 213

W2 : 382 x 40% + 24 x 20% + 54 x 10% = 163

W3 : 541 x 10% = 54

W4 : 498 x 10% = 50

W5 : 1,950 + 24 + 54 + 38 + 43 – [267 – (382 x 60%)] = 2,071

W6 : 1,785 + 24 + 54 + 38 – [213 – (382 x 50%)] = 1,879

Page 201: CAF7-Financial Accounting and Reporting II_Questionbank

Answers

© Emile Woolf International 191 The Institute of Chartered Accountants of Pakistan

8.7 RAISIN INTERNATIONAL

(a) Following are the criteria that should be used while recognizing intangible assets from research and development work.

(i) No intangible asset arising from research shall be recognised.

(ii) An intangible arising from development shall be recognised if, and only if , an entity can demonstrate all of the following:

the technical feasibility of completing the intangible asset so that it will be available for use or sale.

its intention to complete the intangible asset and use or sell it.

its ability to use or sell the intangible asset.

how the intangible asset will generate probable future economic benefits. Among other things, the entity can demonstrate the existence of a market for the output of the intangible asset or the intangible asset itself or, if it is to be used internally, the usefulness of the intangible asset.

the availability of adequate technical, financial and other resources to complete the development and to use or sell the intangible asset.

its ability to measure reliably the expenditure attributable to the intangible asset during its development.

(b) (i) Since the product met all the criteria for the development of the product, it should be recognised as an intangible in the statement of financial position (SOFP) of the company. However, RI should capitalise only the development work (i.e. Rs. 9 million) as intangible asset. IAS-38 does not allow capitalization of cost relating to the research work, training of staff and cost of trial run.

Since the product has a useful life of 7 years, the amortization expense amounting to Rs. 0.32 million (Rs. 9 million × 3/12 ÷ 7 years) should be recorded in the statement of profit or loss.

(ii) This purchasing of right to manufacture should be recognised as an intangible in the SOFP because:

it is for an established product which would generate future economic benefits.

cost of the patent can be measured reliably.

Since there is a finite life, the patent must be amortised over its useful life. The useful life will be shorter of its actual life (i.e. 10 years) and its legal life (i.e. 5 years. The amortization to be recorded in SOCI is Rs. 2.83 million (Rs. 17 million × 10/12 ÷ 5).

(iii) The acquired brand should be recognised as an intangible in the SOFP because acquisition price is a reliable measure of its value. The amortization to be recorded in SOCI is Rs. 0.12 million (Rs. 2 million ÷ 10 years x 7/12).

(iv) The carrying value of the intangible asset should be increased to Rs. 10 million in the SOFP. Since there is an indefinite useful life of the intangible assets, it should not be amortised. Instead, RI should test the intangible asset for impairment by comparing its recoverable amount with its carrying amount.

Page 202: CAF7-Financial Accounting and Reporting II_Questionbank

Financial accounting and reporting II

© Emile Woolf International 192 The Institute of Chartered Accountants of Pakistan

CHAPTER 9 – IAS 17: LEASES

9.1 DAWOOD

The lease has been correctly classified as a finance lease as it is being leased for its entire useful economic life which indicates that the risks and rewards of ownership have been transferred to Dawood.

The leased asset should be capitalised as a non-current asset and depreciated over the 5 year lease period/useful life. By 31st March 2015, the net book value of the asset will be Rs.225,000, being the cost of Rs.250,000 less 6 months depreciation.

A finance lease creditor should be established initially for Rs.250,000. During the year, finance costs will be added and the first payment of Rs.29,500 will be deducted.

The finance costs on the lease of Rs.45,000 (Being total payments of (10 × Rs.29,500) – cash price Rs.250,000) will be spread over the lease term using the sum-of-digits method.

Sum of digits = 2

109

2

)1(

nn= 45

(Note: n = number of periods of borrowing, and as the payments are in ADVANCE not arrears, Dawood is only financing the asset of 4 1/2 years (9 six-monthly periods)).

Therefore, the finance cost relating to the first 6 months through to 31st March 2015

is Rs.9,000 (9/45 Rs.45,000).

This will result in a movement on the finance lease creditor as follows:

6 months to Brought forward Payment Finance cost Carried forward

31 March 13 250,000 (29,500) 9,000 229,500

30 Sept 13 229,500 (29,500) 8,000 208,000

31 March 14 208,000 (29,500) 7,000 185,500

The year end liability of Rs.229,500 will be split between current liabilities Rs.51,000 (29,500 + (29,500–8,000)), and the balance of Rs.178,500 as non-current liabilities.

9.2 FINLEY

Financial statements for the year ended 31 December 2015 (extracts)

Statement of financial position

Non-current assets Rs. Property, plant and equipment (36,000 – 9,000) 27,000

Current liabilities Finance lease obligations (W1) 10,000

Non-current liabilities Finance lease obligations (W1) 17,950

Statement of profit or loss

Depreciation on leased assets ((36,000 ÷ 4) 9,000

Finance lease charges (W1) 1,950

Page 203: CAF7-Financial Accounting and Reporting II_Questionbank

Answers

© Emile Woolf International 193 The Institute of Chartered Accountants of Pakistan

Workings

(1) Finance lease obligations (boat)

Year ended

Opening balance

Lease payment

Capital outstanding

Interest at 7.5%

Closing balance

Rs. Rs. Rs. Rs. Rs.

31 December 2015 36,000 (10,000) 26,000 1,950 27,950

31 December Year 5 27,950 (10,000) 17,950 1,346 19,296

Rs.

Current (balancing figure) 10,000

Non-current 17,950 –––––––

27,950 –––––––

9.3 FABIAN

Financial statements for the year ended 31 December 2015 (extracts)

Statement of financial position

Non-current assets Rs.

Property, plant and equipment (126,760 – 31,690) 95,070

Current assets

Trade and other receivables (W1) 6,250

Current liabilities

Finance lease obligations (W2) 30,056

Non-current liabilities

Finance lease obligations (W2) 69,380

Statement of profit or loss

Operating expenses

Operating lease rentals (W1) 5,450

Depreciation on leased assets (126,760 ÷ 4) 31,690

Finance costs

Finance lease charges (W2) 12,676

Tutorial note

The notes to the financial statements would disclose the fact that included in trade and other receivables is Rs.3,750 (W1) due in more than one year.

Page 204: CAF7-Financial Accounting and Reporting II_Questionbank

Financial accounting and reporting II

© Emile Woolf International 194 The Institute of Chartered Accountants of Pakistan

Workings

(1) Operating lease (car)

Rs.

statement of profit or loss charge = ((7,500 + (36 700)) 6/36) =

5,450

––––––– Cash paid in 2015 (7,500 + (700 6)) 11,700

Minus charged to statement of profit or loss in 2015 (5,450) ––––––– Prepayment at end of 2015 6,250 ––––––– Prepayment at end of 2015 6,250

Cash paid in 2016 (12 700) 8,400

Minus charged to statement of profit or loss in 2016 (5,450 2) (10,900) ––––––– Prepayment at end of 2016 3,750 –––––––

(2) Finance lease obligations (machine)

Date

Opening balance

Interest (10%)

Lease payment

Closing balance

Rs. Rs. Rs. Rs. 2015 126,760 12,676 (40,000) 99,436 2016 99,436 9,944 (40,000) 69,380

Rs.

Current (balancing figure) 30,056 Non-current 69,380 ––––––– 99,436 –––––––

9.4 XYZ INC

(a) Extracts from the financial statements of XYZ Inc at 31 December 2015

Statement of financial position

(i) Tangible fixed assets held under finance leases

Plant and machinery Rs.000 Cost At 1 January 2015 x Additions 4,400 ——— At 31 December 2015 4,400 ——— Accumulated depreciation At 1 January 2015 x Charge for the year 629 ——— At 31 December 2015 629 ——— Net book value At 31 December 2015 3,771 ——— At 1 January 2015 x ———

Page 205: CAF7-Financial Accounting and Reporting II_Questionbank

Answers

© Emile Woolf International 195 The Institute of Chartered Accountants of Pakistan

(ii) Finance lease payables

Amounts payable: Rs.000

Within one to five years (600 8 – 284) 4,516 Less future finance charges 996 ——— 3,520 ——— Accruals Rs.000 Finance leases (667 + 284) 951

Statement of profit or loss

Profit is stated after charging Rs.000

Finance charges 604 (W2) Depreciation 4,400 ÷ 7 629

(b) Table

Period ended Amount Repaid Capital due 7.68% Amount due borrowed for period interest at period end Rs.000 Rs.000 Rs.000 Rs.000 Rs.000

30 June 2015 4,400 (600) 3,800 292 4,092 31 December 2015 4,092 (600) 3,492 268 3,760 30 June 2016 3,760 (600) 3,160 243 3,403 31 December 2016 3,403 (600) 2,803 215 3,018 30 June 2017 3,018 (600) 2,418 186 2,604 31 December 2017 2,604 (600) 2,004 154 2,158 30 June 2018 2,158 (600) 1,558 119 1,677 31 December 2018 1,677 (600) 1,077 83 1,160 30 June 2019 1,160 (600) 560 40 600 31 December 2019 600 (600) – – –

Comparison

Period Sum of digits (W2) Actuarial (as above) Rs.000 Rs.000

1 320 292 2 284 268 3 249 243 4 213 215 5 178 186 6 142 154 7 107 119 8 71 83 9 36 40 10 – – ——— ——— 1,600 1,600 ——— ———

Page 206: CAF7-Financial Accounting and Reporting II_Questionbank

Financial accounting and reporting II

© Emile Woolf International 196 The Institute of Chartered Accountants of Pakistan

WORKINGS

(1) Calculation of finance charge

Rs.000

Minimum lease payments 5 600 2 6,000 Fair value of asset (4,400) ——— Finance charge 1,600 ———

(2) Allocation of finance charge

Period ended Digits Finance charge Rs.000

30 June 2015 9 9/45 1,600 320

31 December 2015 8 8/45 1,600 284 ——— 604

30 June 2016 7 7/45 1,600 249

31 December 2016 6 6/45 1,600 213

30 June 2017 5 5/45 1,600 178

31 December 2017 4 4/45 1,600 142

30 June 2018 3 3/45 1,600 107

31 December 2018 2 2/45 1,600 71

30 June 2019 1 1/45 1,600 36 31 December 2019 – —— ———

n(n + 1)

2 =

9(9 + 1)2

45 1,600

—— ———

(3) Lease obligation

Period ended Amount Repaid Capital Interest Amount borrowed due for due at period period end Rs.000 Rs.000 Rs.000 Rs.000 Rs.000

30 June 2015 4,400 (600) 3,800 320 4,120 31 December 2015 4,120 (600) 3,520 284 3,804 30 June 2016 3,804 (600) 3,204 249 3,453 31 December 2016 3,453 (600) 2,853 213 3,066

Page 207: CAF7-Financial Accounting and Reporting II_Questionbank

Answers

© Emile Woolf International 197 The Institute of Chartered Accountants of Pakistan

9.5 SNOW INC

Extracts from the financial statements of Snow Inc for year ended 31 December 2015

Statement of profit or loss

Profit is stated after charging

Rs.000

Finance charges (1,714 + 1,429 + 9,614) (W1 and 2) 12,757

Depreciation 41,667

Statement of financial position

Tangible fixed assets held under finance leases

Plant and machinery

Cost Rs.000 At 1 January 2015 – Additions (35,000 + 150,000) 185,000 ———— At 31 December 2015 185,000 ———— Accumulated depreciation At 1 January 2015 –

Charge for year

35,000

3 +

15,0005

41,667

——— At 31 December 2015 41,667 ——— Net book value At 31 December 2015 143,333 ———— At 1 January 2015 – ——— Finance lease payables

Amounts payable: Rs.000

Within one to five years 166,000 (6,500 4 + 35,000 4) Less future finance charges 18,243 (2,857 + 15,386 *) ———— 147,757 ————

* 35,000 5 = 175,000 – 150,000 – 9,614 = 15,386 Accruals Rs.000

Finance leases 46,000 (11,000 + 35,000)

Page 208: CAF7-Financial Accounting and Reporting II_Questionbank

Financial accounting and reporting II

© Emile Woolf International 198 The Institute of Chartered Accountants of Pakistan

WORKINGS

(1) Snowplough (a) Calculation of finance charge Rs.000 Deposit 2,000

MLP (6 6,500) 39,000 Fair value of asset (35,000) ——— Finance charge 6,000 ——— (b) Allocation of finance charge Period Digits Finance ended charge Rs.000

30.06.2015 6 6

21 6,000 1,714

31.12.2015 5 5

21 6,000 1,429

30.06.2016 4 4

21 6,000 1,143

31.12.2016 3 3

21 6,000 857

30.06.2017 2 2

21 6,000 571

31.12.2017 1 1

21 6,000 286

—— ——— 21 6,000 —— ———

n (n + 1)

2 =

6 (7)2

= 21

(c) Period Capital Interest Amount Repayment Capital ended O/S at start O/S at end O/S at end Rs.000 Rs.000 Rs.000 Rs.000 Rs.000

30.6.15 33,000 1,714 34,714 (6,500) 28,214 31.12.15 28,214 1,429 29,643 (6,500) 23,143 30.6.16 23,143 1,143 24,286 (6,500) 17,786 31.12.16 17,786 857 18,643 (6,500) 12,143 (2) Snow machine Period Amount Repayment Capital Interest Amount ended O/s at start O/S at start at 8.36% O/S at end Rs.000 Rs.000 Rs.000 Rs.000 Rs.000 31.12.15 150,000 (35,000) 115,000 9,614 124,614 31.12.16 124,614 (35,000) 89,614 7,492 97,106

Page 209: CAF7-Financial Accounting and Reporting II_Questionbank

Answers

© Emile Woolf International 199 The Institute of Chartered Accountants of Pakistan

9.6 MIRACLE TEXTILE LIMITED

Miracle Textile Limited Statement of financial position (Extracts)

As at 30 June 2015

Note 2015 2014

ASSETS

Rs. Rs.

Non-current assets Property, plant and equipment 4 16,000,000 18,000,000

LIABILITIES

Non-current liabilities Obligation under finance lease 9 6,505,219 10,633,074

Current liabilities

Current portion of obligation under finance lease 9 4,127,856 3,566,925

Miracle Textile Limited

Notes to the financial statements (Extracts) As at 30 June 2015

4- Property, plant and equipment

2015 2014

Rs.000 Rs.000

Leased assets Cost

Opening balance 20,000,000 - Addition during the year - 20,000,000

20,000,000 20,000,000 Accumulated depreciation

Opening balance (2,000,000) - Depreciation for the year (2,000,000) (2,000,000)

(4,000,000) (2,000,000)

Balance as at 30 June 16,000,000 18,000,000

9- Obligations under finance lease (W1)

30-Jun-2015

30-Jun-2014

Minimum lease

payment

Financial charges

for future periods

Principal outstanding

Minimum lease

payment

Financial charges

for future periods

Principal outstanding

Rs. Rs. Rs. Rs. Rs. Rs. Not later

than one year 5,800,000 1,672,144 4,127,856

5,800,000 2,233,075 3,566,925

Later than one year but not later than five years 7,800,000 1,294,781 6,505,219

13,600,000 2,966,926 10,633,074

Later than five years - - -

- - -

13,600,000 2,966,926 10,633,074

19,400,000 5,200,000 14,200,000

Page 210: CAF7-Financial Accounting and Reporting II_Questionbank

Financial accounting and reporting II

© Emile Woolf International 200 The Institute of Chartered Accountants of Pakistan

9.1 The Company has entered into a finance lease agreement with a bank in respect of a machine. The finance lease liability bears interest at the rate of 15.725879% per annum. The company has the option to purchase the machine by paying an amount of Rs. 2 million at the end of the lease term. The lease rentals are payable in annual instalments ending in June 2015. There are no financial restrictions in the lease agreement.

W1: Lease Schedule

Payment date

Opening principal

Instalment Principal

repayment Interest @

15.725879% Closing principal

01.07.2013 20,000,000 5,800,000 5,800,000 - 14,200,000 01.07.2014 14,200,000 5,800,000 3,566,925 2,233,075 10,633,075 01.07.2015 10,633,075 5,800,000 4,127,856 1,672,144 6,505,219 01.07.2016 6,505,219 5,800,000 4,776,997 1,023,003 1,728,222 30.06.2017 1,728,222 2,000,000 1,728,222 271,778 -

20,000,000 5,200,000

9.7 SHOAIB LEASING LIMITED

Entries in the books of Lessor

Date Particulars Dr. Cr.

01.07.2015 Lease payments receivable (W1) 2,680,000 Machine 2,100,000 Unearned finance income (W1) 580,000

30.06.2016 Bank 860,000 Lease payments receivable 860,000

30.06.2016 Unearned finance lease 272,941 Finance income (W2) 272,941

30.06.2017 Bank 860,000 Lease payments receivable 860,000

30.06.2017 Unearned finance lease 196,640 Finance income (W2) 196,640

30.06.2018 Bank 960,000

Lease payments receivable 960,000

30.06.2018 Unearned finance lease 110,419 Finance income (W2) 110,422

W1: Total finance income Rupees Total future lease payments (Rs. 860,000 x 3) 2,580,000 Add: Purchase bargain option 100,000

Gross investment in finance lease 2,680,000 Less: Cost of assets 2,100,000

Total finance income 580,000

Page 211: CAF7-Financial Accounting and Reporting II_Questionbank

Answers

© Emile Woolf International 201 The Institute of Chartered Accountants of Pakistan

W2: Amortization schedule

Date

Principal Opening

Instalment Interest Principal Principal Closing

Rs. Rs. Rs. Rs. Rs. 30.06.2016 2,100,000 860,000 272,941 587,059 1,512,941.20 30.06.2017 1,512,941 860,000 196,640 663,360 849,581.19 30.06.2018 849,581 960,000 110,419* 849,581 nil

580,000 2,099,997.04

Note that there is a rounding adjustment of Rs. 3 in the last interest amount.

Shoaib Leasing Limited

Extracts from the statement of financial position as at June 30, 2016

2016

Rupees

Non-current assets Note 10

Net investment in leases 849,578

Current assets

Current portion of net Investment in leases 663,360

10 Net investment in leases

Minimum lease payments receivables (Note 10.1) 1,720,000

Add: Residual value of leased assets 100,000

Gross Investments in leases 1,820,000

Less: Unearned lease income (Rs. 580,000 - Rs. 272,941 - 3) (307,062)

Net investment in leases (Note 10.2) Note 10.2 1,512,938

Less: Current portion of net investment in leases (663,360)

849,578

10.1 Minimum lease payments

Less than one year 860,000

More than one year and less than 5 years 860,000

1,720,000

10.2 Net investment in leases

Less than one year 663,360

More than one year and less than 5 years 849,578

1,512,938

Page 212: CAF7-Financial Accounting and Reporting II_Questionbank

Financial accounting and reporting II

© Emile Woolf International 202 The Institute of Chartered Accountants of Pakistan

9.8 NEPTUNE LIMITED

(a) Journal entries (i) Finance Lease:

Date Particulars

Debit Credit Rupees

1-Jan-2015 Finance lease debtors 12,000,000

Unearned finance lease income 3,295,690

Sale 8,704,310

(Record sale of vehicles on finance lease)

1-Jan-2015 Bank 2,000,000

Finance lease debtors 2,000,000

(Instalment received under finance lease)

31-Dec-2015 Unearned finance lease income 1,005,647

Finance lease income 1,005,647

(Interest income earned at 15%)

(ii) Operating lease:

1-Jan-2015 Bank 4,000,000

Unearned rental income 4,000,000

(Operating lease instalment received in advance)

31-Dec-2015 Unearned rental income 3,803,333

Rental income (11,410,000÷3)(W2) 3,803,333

(Booking of operating lease income)

31-Dec-2015 Depreciation expenses (15,000,000÷6) 2,500,000

Accumulated depreciation on machine. 2,500,000

(Yearly depreciation on machine)

Reason for choice of leases:

1. Lease A should be accounted for as a finance lease because the lease term covers the entire economic life.

2. Since none of the conditions specified in IAS-17 (Leases) for classification as a finance lease is being met, Lease B shall be considered as an operating lease.

Page 213: CAF7-Financial Accounting and Reporting II_Questionbank

Answers

© Emile Woolf International 203 The Institute of Chartered Accountants of Pakistan

W1 Finance lease:

Year

Opening Balance

Instalment Income at 15%

Recovery of

Principal

Closing balance

Rs. Rs. Rs. Rs. Rs.

2015 8,704,310 2,000,000 1,005,647 994,354 7,709,957

2016 (A) 7,709,957 2,000,000 856,493 1,143,507 6,566,450

2017 6,566,450 2,000,000 684,967 1,315,033 5,251,417

2018 5,251,417 2,000,000 487,713 1,512,287 3,739,130

2019 3,739,130 2,000,000 260,870 1,739,130 2,000,000

2020 2,000,000 2,000,000 0 2,000,000 0

(B) 8,000,000 1,433,550 6,566,450

(A)+(B) 10,000,000 2,290,043 7,709,957

W2 Operating lease:

Rs.

Annual instalment 2015 4,000,000

2016 (4,000,000 × 95%) 3,800,000

2017 (3,800,000 × 95%) 3,610,000

11,410,000

(b) Neptune Limited

Notes to the Financial Statements

For the year ended December 31, 2015

(i) Investment in finance lease

2015 Rs.

Present value of minimum lease payments 7,709,957 Less: current maturity (1,143,507)

6,566,450

Gross investment in finance leases

Net investment in leases

2015 2015

Rs. Rs.

Less than one year 2,000,000 1,143,507

One to five years 8,000,000 6,566,450

10,000,000 7,709,957

Less: unearned finance income (2,290,043)

Net investment in leases 7,709,957

The minimum lease payment has been discounted on interest rate of 15% per annum to arrive at their present value. Rentals are paid in annual instalments.

(ii) Operating lease

Not later than one

year

One to five years

Total

Rs. Rs. Rs.

Future minimum lease payments (W2) 3,800,000 3,610,000 7,410,000

Page 214: CAF7-Financial Accounting and Reporting II_Questionbank

Financial accounting and reporting II

© Emile Woolf International 204 The Institute of Chartered Accountants of Pakistan

9.9 QUARTZ AUTO LIMITED

(a) Entries to record the lease in books of Quartz Auto Limited Description Debit Credit Lease receivable (2,715,224 × 5) + 700,000 14,276,120

Cost of goods sold [(900,000 × 7) - (100,000 ×7 × 0.49718)] 5,951,974

Inventory (900,000 x 7) 6,300,000 Sales (Note – 1) 9,101,974 Unearned finance income 4,826,120

Bank 2,715,224 Lease receivable 2,715,224

Unearned finance income 1,417,500 Finance income 1,417,500

Note – 1 Lower of fair value i.e. 9,450,000 (Rs. 1,350,000 x 7) and PV of minimum lease payment (2,715,227 x 3.35219 = 9,101,974)

(b) Disclosure in the financial statements

1- Net investment in lease 2015 Rs.

Lease receivable (2,715,227 x 4) 10,860,896 Unguaranteed residual amount 700,000

Gross investment in lease 11,560,896

Less: Unearned finance income (4,826,120 – 1,417,500)

(3,408,620)

8,152,276

1.1 Details of investment in finance lease

Gross investment

in lease

Net investment

in lease Not later than one year 2,715,224 1,492,383 Later than one year but not later than five years 8,845,672 6,659,893 Later than five years - -

11,560,896 8,152,276

(W1)

Year

ended Instalment at year end

Interest Principal Net

Investment in Lease

Gross Investment

in Lease

9,450,000 14,276,120

31/06/2015 2,715,224 1,417,500 1,297,724 8,152,276 11,560,896

31/06/2016 2,715,224 1,222,841 1,492,383 6,659,893 8,845,672

31/06/2017 2,715,224 998,984 1,716,240 4,943,653 6,130,448

31/06/2018 2,715,224 741,548 1,973,676 2,969,977 3,415,224

31/06/2019 2,715,224 445,247 2,269,977 700,000 700,000

Page 215: CAF7-Financial Accounting and Reporting II_Questionbank

Answers

© Emile Woolf International 205 The Institute of Chartered Accountants of Pakistan

9.10 LODHI TEXTILE MILLS LIMITED

Particulars Debit Credit

Generator A (i) Cash / Bank 6,000,000 Accumulated depreciation – Generator 2,500,000

Loss on sale/ Impairment loss *1,500,00

0 Property, plant and equipment - Generator 10,000,000

*(This amount comprises of impairment loss amounted to Rs. 1 million and loss on disposal amounted to Rs. 0.5 million.)

(ii) Assets subject to finance lease - Generator 6,000,000 Liabilities against assets subject to finance lease 6,000,000

Generator B (i) Cash / Bank 6,000,000 Accumulated depreciation – Generator 6,000,000 Property, plant and equipment - Generator 12,000,000

(ii) Assets subject to finance lease - Generator 6,000,000 Liabilities against assets subject to finance lease 6,000,000

(iii) Impairment loss 1,000,000 Accumulated impairment (ASFL) - Generator 1,000,000

Generator C (i) Cash / Bank 8,000,000

Accumulated depreciation – Generator 3,000,000

Property, plant and equipment - Generator 10,000,00

0

Deferred income OR Surplus on revaluation of fixed assets 1,000,000

(ii) Assets subject to finance lease - Generator 8,000,000 Liabilities against assets subject to finance lease 8,000,000

Page 216: CAF7-Financial Accounting and Reporting II_Questionbank

Financial accounting and reporting II

© Emile Woolf International 206 The Institute of Chartered Accountants of Pakistan

9.11 NOMAN ENGINEERING LIMITED

Journal entries

Date Description Debit Credit

Rs.000 Rs.000

1-Jul-2014 Bank 20,000

Accumulated depreciation (18,750-15,000) 3,750

Property, plant and equipment 18,750

Deferred gain on disposal (20,000-15,000) 5,000

(Disposal of plant under sale and finance lease back)

1-Jul-2014 Property, plant and equipment

20,000

Long term finance lease liability

20,000

(Plant acquired under sale and lease back)

31-Dec-2014 Long term finance lease liability W.1 1,127

Interest expense W.1 1,373

Bank

2,500

(Payment of 1st. Instalment of lease liability)

30-Jun-2015 Long term finance lease liability 1,204

Interest expense 1,296

Bank 2,500

(Payment of 2nd. Instalment of lease liability)

30-Jun-2015 Deferred gain on disposal (5,000/6) 833

Gain on disposal 833

(Deferred gain on amortised over the life of the plant)

30-Jun-2015 Depreciation expense (20,000/6) 3,333

Accumulated depreciation 3,333

(Depreciation for the year for plant)

Note: If there is no reasonable certainty that the lessee will obtain ownership by the end of the lease term, the asset shall be fully depreciated over the shorter of the lease term and its useful life.

W1: Liability against finance lease

Instalment payments

Interest at

13.731%

Principal balance

Balance 1-Jul-2014 20,000

Payments made on 31-Dec-2014 2,500 1,373 (1,127)

30-Jun-2015 2,500 1,296 (1,204)

5,000 2,669 (2,331)

Balance 30-6-2015

17,669

Page 217: CAF7-Financial Accounting and Reporting II_Questionbank

Answers

© Emile Woolf International 207 The Institute of Chartered Accountants of Pakistan

CHAPTER 10 – IAS 37: PROVISIONS CONTINGENT LIABILITIES AND

CONTINGENT ASSETS AND IAS 10: EVENTS OCCURRING AFTER THE

REPORTING DATE

10.1 BADAR

Decommissioning costs

IAS 37 Provisions, Contingent Liabilities and Contingent Assets only permits a provision to be made if three conditions are met:

(i) The company has a present obligation, either legally or constructively, as a result of a past event;

(ii) Probable outflow of resources is required to settle the obligation; and

(iii) A reliable estimate is available.

Although there is no legal requirement to restore the site, the company has established a constructive obligation by setting a valid expectation in the market, due to its published policies and past practice, from which it cannot realistically withdraw.

It therefore appears probable that Badar will have to pay money to improve the site and so a provision should be created for the expected amount. As the expected payment of Rs.100,000 will not be settled for three years, the provision should be discounted and entered at its net present value of Rs.75,131 (Rs.100,000/(1.1)3). Over the three years, the discounting should be unwound and charged to profit or loss as finance costs, resulting in a provision of Rs.100,000 by the end of the third year.

The cost of the construction work has been correctly capitalised. The cost of the future decommissioning work should be added to this asset so that the total costs of the site can be matched to the revenue from the copper over the period of mining. This will result in an asset of Rs.575,131 which should be depreciated over the three year life in line with anticipated revenues.

10.2 GEORGINA

(1) Litigation for damages

Under IAS37, a provision should only be recognised when:

an entity has a present obligation as a result of a past event

it is probable that an outflow of economic benefits will be required to settle the obligation

a reliable estimate can be made of the amount of the obligation.

Applying this to the facts given:

Georgina’s legal advisors have confirmed that there is a legal obligation. This arose from the past event of the sale, on 1 September 2015 (i.e. before the year end).

Probable is defined as ‘more likely than not’. The legal advisors have confirmed that it is likely that the claim will succeed.

A reliable estimate of Rs.500,000 has been made.

Page 218: CAF7-Financial Accounting and Reporting II_Questionbank

Financial accounting and reporting II

© Emile Woolf International 208 The Institute of Chartered Accountants of Pakistan

Therefore a provision of Rs.500,000 should be made.

Counter-claim

IAS37 requires that such a reimbursement should only be recognised where receipt is ‘virtually certain’. Since the legal advisors are unsure whether this claim will succeed no asset should be recognised in respect of this claim.

(2) Claim for unfair dismissal

In this case, the legal advisers believe that success is unlikely (i.e. possible rather than probable). Therefore this claim meets the IAS37 definition of a contingent liability:

a possible obligation

arising from past events

whose existence will be confirmed only by the occurrence or non-occurrence of one or more uncertain future events.

The liability is a possible one, which will be determined by a future court case or tribunal. It did arise from past events (the dismissal had taken place by the year end).

This contingent liability should be disclosed in the financial statements (unless the legal advisors believe that the possibility of success is in fact remote, and then no disclosure is necessary).

(3) Returns

Applying the IAS37 conditions in (1) to the facts given:

Although there is no legal obligation, a constructive obligation arises from Georgina’s past actions. Georgina has created an expectation in its customers that such refunds will be given.

As at the year end, based on past experience, an outflow of economic benefits is probable.

A reliable estimate can be made. This could be 1% × 400,000 but since the returns are now all in the actual figure of Rs.3,500 can be used.

Therefore a provision of Rs.3,500 should be made.

(4) Closure of division

Applying the above IAS37 conditions in (1) to the facts given:

A present obligation exists because at the year end there is a detailed plan in place and the closure has been announced in the press.

An outflow of economic benefits is probable.

A reliable estimate of Rs.300,000 has been made.

However, IAS37 specifically states in respect of restructuring that any provision should include only direct expenses, not ongoing expenses such as staff relocation or retraining. Therefore a provision of Rs.250,000 (300,000 – 50,000) should be made.

Page 219: CAF7-Financial Accounting and Reporting II_Questionbank

Answers

© Emile Woolf International 209 The Institute of Chartered Accountants of Pakistan

10.3 EARLEY INC

(a) IAS 10 (revised) Events After the Statement of financial position Date states that assets and liabilities should be adjusted for events occurring after the statement of financial position date that provide additional evidence relating to conditions existing at the statement of financial position date. It specifically includes the example of bad debts, where evidence of bankruptcy of a debtor occurs after the year end.

In this case, Nedengy appears to have recovered part of the debt and as such only Rs.200,000 needs to be provided. It may be argued that the receivership has occurred as a result of events occurring after the statement of financial position date, as a result of a change in legislation for example, but this is unlikely.

IAS 18 Revenue states that when uncertainty arises about the collectability of an amount already included in revenue, the amount should be recognised as an expense.

(b) It is likely that the fall in the value of the property will fit the IAS 10 (revised) definition of adjusting events noted in (a) above, unless, again, it can be argued that the decline in the property market occurred after the year-end.

IAS 36 Impairment of assets and IAS 16 Property, Plant and Equipment require that the carrying amount of property, plant and equipment should be reviewed periodically in order to assess whether the recoverable amount has fallen below the carrying amount. Where it has, the property, plant and equipment should be written down to the recoverable amount, either through the statement of profit or loss as an expense, or though other comprehensive income to revaluation reserve in shareholder’s equity, but only to the extent that the balance on the revaluation reserve relates to a previous revaluation surplus on the same asset.

(c) IAS 2 Inventories requires that inventories be stated at the lower on cost and net realisable value. Net realisable value is the estimated selling price in the ordinary course of business less the estimated costs of completion and the estimated costs necessary to make the sale.

Unless Earley was making a significant margin on the tricycles, it is likely that the reduction in selling price of 30% will necessitate a write- down to net realisable value, especially considering the transportation costs to Iraq which must be included. If the Iraqi option is unlikely to proceed, it may be necessary to write the tricycles down to scrap value.

(d) Under IAS 10, the nationalisation is likely to be regarded as a non-adjusting event that merely requires disclosure in the financial statements. IAS 27 Consolidated Financial Statements and Accounting for Investments in Subsidiaries, requires that an investment in a enterprise should be accounted for as an investment (under IAS 39: Financial Instruments: Recognition and Measurement) from the date that it ceases to fall within the definition of a subsidiary and does not become an associate. It seems here that Earley has neither control nor significant influence, nor even an investment as the assets have been in fact, expropriated. The loss of the investment should be accounted for in the year in which it occurred, but disclosed in the current year.

If the loss of the subsidiary results in Earley no longer being a going concern, then the event becomes an adjusting event.

(e) & (f) Both of the events described are non-adjusting event which should be disclosed, but not adjusted for in the current year financial statements.

Page 220: CAF7-Financial Accounting and Reporting II_Questionbank

Financial accounting and reporting II

© Emile Woolf International 210 The Institute of Chartered Accountants of Pakistan

10.4 ACCOUNTING TREATMENTS

(a) IAS 37 Provisions contingent liabilities and contingent assets states that contingent gains should not be recognised as income in the financial statements. The company has a debit balance already in its books which indicates that it must be reasonably certain that at least part of the claim will be paid. This element of the claim then is probably not a contingency at all. The remaining part (the difference between the Rs.15,000 and the Rs.18,600) is, and should be disclosed and not accrued.

(b) IAS 16 Property, Plant and Equipment requires that the carrying amount of property, plant and equipment should be reviewed periodically in order to assess whether the recoverable amount has fallen below the carrying amount. Where it has, the property, plant and equipment should be written down to the recoverable amount through the statement of profit or loss as an expense. In this case this would result in the recognition of an expense of Rs.200,000. (280,000 – 80,000).

It may be the case that the amounts involved are so significant as to warrant separate disclosure in the statement of profit or loss under IAS 8 Net Profit of Loss for the Period, Fundamental Errors and Changes in Accounting Policies.

(c) IAS 37 states that contingent liabilities should not be recognised. Though a provision should be made for amounts where the company has an obligation to pay them.

The question in this case is whether or there is an obligating event within the context of IAS 37. On balance it seems inappropriate to recognise a provision in respect of this amount but the possible liability should be disclosed as a contingent liability.

(i) the nature of the contingency

(ii) the uncertainties surrounding the ultimate outcome

(iii) the likely effect, ie Rs.500,000 loss less likely tax relief.

(d) IAS 2 Inventories requires that inventories be stated at the lower on cost and

net realisable value. Net realisable value is the estimated selling price in the ordinary course of business less the estimated costs of completion and the estimated costs necessary to make the sale.

In this case, cost is Rs.1,800 and net realisable value is Rs.1,600

(e) The company should set up a provision for Rs.100,040, ie should accrue for the 10% probable liability. It should disclose the possible liability under contingent liabilities. The disclosure is as noted in (c) except that the financial

effect is Rs.300,120 (30% Rs.1,000,400). The balance should be ignored as it is a remote contingent liability.

Tutorial note

In (c) above it is not appropriate to provide for 20%receivableRs.500,000, ie Rs.100,000. This would only be appropriate where the event is recurring many times over.

In (e) it is appropriate to use the percentages provided, as warranty work is provided for.

Page 221: CAF7-Financial Accounting and Reporting II_Questionbank

Answers

© Emile Woolf International 211 The Institute of Chartered Accountants of Pakistan

10.5 J-MART LIMITED

(a) Adjusting events:

Adjusting events are events that provide further evidence of conditions that existed at the reporting date.

Examples of adjusting events include:

(i) The subsequent determination of the purchase price or of the proceeds of sale of non-current assets purchased or sold before the year end.

(ii) The renegotiation of amounts owing by customers or the insolvency of a customer

(iii) Amounts received or receivable in respect of insurance or the insolvency of a customer.

(iv) The settlement after the reporting date of a court case that confirms that the entity had a present obligation at the reporting date.

(v) The receipt of the information after the reporting date indicating that an asset was impaired at the reporting date.

(vi) The discovery of fraud or errors that show that the financial statements are incorrect.

Non-adjusting events:

Non-adjusting events are indicative of conditions that arose subsequent to the reporting date.

Examples of non-adjusting events might be:

(i) Losses of non-current assets or inventories as a result of a catastrophe such as fire or flood

(ii) Closing a significant part of the trading activities if this was not begun before the year end

(iii) The value of an investment falls between the reporting date and the accounts are authorised

(iv) Announcement of dividend after year end.

(b) (i) The conditions attached to the sale give rise to a constructive obligation on the reporting date.

A provision for the sales return should be recognised for 5% of June 2015 sales. The related cost should also be reversed.

(ii) Since the law suit was already in progress at year-end and the amount of compensation can also be estimated, it is an adjusting event.

A provision of Rs. 400,000 should be made.

(iii) There is no obligating event at the year end either for the costs of fitting the smoke detectors or for fines under the legislation.

No provision should be recognised in this regard.

(iv) The obligating event is the communication of decision to the customers and employees, which gives rise to a constructive obligation from that date, because it creates a valid expectation that the division will be closed.

Since no communication has yet been made, no provision is required in this regard.

Page 222: CAF7-Financial Accounting and Reporting II_Questionbank

Financial accounting and reporting II

© Emile Woolf International 212 The Institute of Chartered Accountants of Pakistan

(v) The obligating event is the signing of the lease contract, which gives rise to a legal obligation.

A provision is required for the unavoidable rent payments.

(vi) Since the declaration was announced after year-end, there is no past event and no obligation at year-end and is therefore non-adjusting event.

Details of the dividend declaration must, however, be disclosed.

10.6 AKBER CHEMICALS LIMITED

(a) The event is an accident, and since it happened before the year end, it is a past event. However, there is no present obligation since:

(i) there is no law requiring the company to clean the canal.

(ii) there is no constructive obligation to clean the river since:

a public statement has not been made;

there is no established pattern of past practice as this was the first time the company faced such a situation.

Although the company has decided to clean up the river and even has a reliable estimate of the costs thereof, no liability or provision should be recognised in the current year because:

the decision was taken after year end; and

the decision was not yet made public.

(b) It is a non-adjustable event because the event due to which the net realizable value (NRV) of stock has fallen, arose after the reporting date.

However, if this event is material, the company should disclose the decline in NRV in its financial statement for the year ended June 30, 2015.

(c) The company should make the provision because:

(i) the company has a present obligation because of past event

(ii) the claim of the customer is valid and is confirmed by the company's inspection team which shows that an outflow will be required to settle the obligation.

(iii) the amount of outflow is reliably estimated i.e. Rs. 2 million.

Since the company is certain of recovery from the vendor, it should:

(i) disclose it as a separate asset.

(ii) recognise a receivable but the same should not exceed the amount of the related provision i.e. rs. 2.0 million.

Page 223: CAF7-Financial Accounting and Reporting II_Questionbank

Answers

© Emile Woolf International 213 The Institute of Chartered Accountants of Pakistan

10.7 QALLAT INDUSTRIES LIMITED

(i) Provision must be made for estimated future claims by customers for goods already sold.

The expected value i.e. Rs. 10 million ([Rs. 150m x 2%] + [Rs. 70m x 10%]) is the best estimate of the provision.

(ii) Warehouse A: It is an onerous contract. as the warehouse has been sublet at a loss of Rs. 200,000 per month. QIT should therefore create a provision for the onerous contract that arises on vacating the warehouse. This is calculated as the excess of unavoidable costs of the contract over the economic benefits to be received from it. Therefore, QIL should immediately provide for the amount of Rs. 13.2 million. [5.5 years x 12 month x Rs. 200,000] in its financial statements i.e. for the year ended June 30, 2015.

Warehouse B: It is not an onerous contract because the warehouse has been sublet at profit. Hence this would require no adjustment.

(iii) A provision is to be made by QIL against a contingent liability as:

(i) There is a present obligation (legal or constructive) as a result of a past event; i.e. accident occurred on June 15, 2015.

(ii) It is probable that outflow of resources will be required to settle the obligation; and

(iii) A reliable estimate can be made of the amount of the obligation.

The amount of provision shall be Rs. 2.0 million i.e. the most probable amount as determined by the lawyer.

(iv) A provision of Rs. 0.4 million is required in relation to penalty for March 1 to June 30, 2015 because at the reporting date there is a present obligation in respect of a past event.

The reimbursement of penalty amount from the vendor shall be recognised when and only when it is virtually certain that reimbursement will be received if the entity settles the obligation. The reimbursement should be treated as a separate asset in the statement of financial position. However, in profit and loss statement, the expense relating to a provision may be netted off with the amount recognised as recoverable, if any.

10.8 SKYLINE LIMITED

(i) Although the debt owing by the customer existed at the reporting date, the customer’s inability to pay did not exist at that point. This condition only arose in January 2016 after the fire.

Thus, this is a non-adjusting event. However, if it is material for the financial statements, the following disclosure should be made.

Nature of the event

An estimate of its financial effect

(ii) The amount withdrawn before year end i.e. Rs. 1.5 million is an adjusting event as

although it was discovered after year end it existed at the year end. However, since 60% has been recovered subsequently, Rs. 0.6 million would be provided.

Page 224: CAF7-Financial Accounting and Reporting II_Questionbank

Financial accounting and reporting II

© Emile Woolf International 214 The Institute of Chartered Accountants of Pakistan

The further withdrawal of Rs. 6.0 million is a non-adjusting event as it occurred after year end. However, if the events are considered material the following disclosures should be made:

Nature of the event

The gross amount of contingency

The amount recovered subsequently

(iii) SL should not recognise the contingent gain until it is realised. However, if recovery of damages is probable and material

to the financial statements, SL

should disclose the following facts in the financial statements:

Brief description of the nature of the contingent asset

An estimate of the financial effect.

(iv) SL should make a provision of the expected amount i.e. Rs. 1.2 million (Rs. 1.0 million x 60% + Rs. 1.5 million x 40%) because

it is a present obligation as a result of past event;

it is probable that an outflow of resources embodying economic benefits will be required to settle the obligations; and

a reliable estimate can be made of the amount.

In addition, SL should disclose the following in the notes to the financial statements:

Brief nature of the contingent liability

The amount of contingency

An indication of the uncertainties relating to the amount or timing of any outflow.

10.9 WALNUT LIMITED

(i) This is an adjusting post reporting event as it provides evidence of conditions that existed at the end of the reporting period. The reasons for the competitor’s price reduction will not have arisen overnight, but will normally have occurred over a period of time, may be due to superior investment in technology.

An inventory write down of Rs. 2.5 million should be recognised and the amount included as inventory on the Statement of Financial Position reduced to Rs. 12.5 million.

(ii) The provision should be recognised because the obligating event is the communication of event to the public which creates a valid expectation that the division will be closed.

However, the provision should only be recognised to the extent of redundancy costs. IAS prohibits the recognition of future operating losses, staff training and profits on sale of assets.

(iii) This is a non-adjusting event because the burglary and theft of consumable stores occurred after reporting date. However, if the event is material, it should be disclosed in the financial statements unless the loss is recoverable from the insurance company.

Page 225: CAF7-Financial Accounting and Reporting II_Questionbank

Answers

© Emile Woolf International 215 The Institute of Chartered Accountants of Pakistan

(iv) The drop in value of investment in shares is a non-adjusting event. Since the legislation was announced after the reporting date, the event is not a past event. However, if the amount is material, it should be disclosed in the financial statements.

(v) This is an adjusting event as it provides evidence of conditions that existed at the end of the reporting period. The insolvency of a debtor and the inability to pay usually builds up over a period of time and it can therefore be assumed that it was facing financial difficulty at year-end.

A bad debts expense of Rs. 1.5 million should be recognised in SOCI.

(vi) It is a non-adjusting event because the declaration was announced after the year-end and there was no obligation at year end. Details of the bonus shares declaration must, however, be disclosed.

10.10 ATTOCK TECHNOLOGIES LIMITED

(i) Since the event which caused the inventory to be sold at a loss occurred after the year end, it is non-adjusting event. However, the effect of the event should be disclosed in the financial statements for the year ended June 30, 2015.

(ii) It is an adjusting event in accordance with the requirement of IAS-10. The debtor’s balance should be written down by 80% amount.

(iii) It is non-adjusting event as the subsequent reduction in price is due to an event, introduction of competitive product, occurred after the reporting period.

(iv) Since this change was not enacted before the reporting date, it is a non-adjusting event. However, a disclosure should be made for this change.

(v) Since the declaration was announced after the year-end and there was no obligation at year-end it is a non-adjusting event. Details of the dividend declaration must, however, be disclosed.

Page 226: CAF7-Financial Accounting and Reporting II_Questionbank

Financial accounting and reporting II

© Emile Woolf International 216 The Institute of Chartered Accountants of Pakistan

CHAPTER 11 – IAS 8: ACCOUNTING POLICIES, CHANGES IN ACCOUNTING

ESTIMATES AND ERRORS

11.1 WONDER LIMITED

2015

2014

(Restated)

Rs.m Rs.m

Wonder Limited

Extracts of Statement of financial position

For the year ended 30 June 2015

Property, plant and equipment

178.50 111.50

Retained earnings

158.65 95.05

Deferred tax liability

41.85 21.45

PPE: Year 2015: 189 - [20 - (20 × 10% × 1.75)] + [56/4 – 56/7] PPE: Year 2014: 130 - 18.5(Note X)

DTL: Year 2015: [(21.45 + (45 - 27) + {(6+2) × 30%}] DTL: Year 2014: 27 - 5.55 (Note X)

Wonder Limited

Extracts from the Statement of profit or loss for the year ended 30 June 2015

Profit before taxation

98.00 101.50

Taxation

(34.40) (36.45)

Profit after taxation

63.60 65.05

PBT : Year 2015 : 90 + (20 × 10% ) + [(56/4) - (56/7)] PBT : Year 2014 : 120 - 18.5 (Note X)

Tax : Year 2015: 32 + [(6+2) × 30%] Tax : Year 2014 : 42 - 5.55 (Note X)

Wonder Limited

Extracts of statement of changes in equity for the year ended 30 June 2015

Retained earnings

Rs.m

Balance as on 1 July 2013 (108-78) 30.00

Profit for the year ended 30 June 2014 (78 - 12.95 (Note X))- restated 65.05

Balance as at 30 June 2014 - restated 95.05

Profit for the year ended 30 June 2015 63.60

Balance as at 30-June 2015 158.65

Page 227: CAF7-Financial Accounting and Reporting II_Questionbank

Answers

© Emile Woolf International 217 The Institute of Chartered Accountants of Pakistan

Wonder Limited Notes to the financial statements For the year ended 31 December 2015

X Correction of error

During the year ended 30 June 2013, the repair works was erroneously debited to machinery account. The effect of this error is as follows:

2014

Rs.m

Effect on the statement of profit or loss (Increase) / decrease in expenses or losses Repairs and maintenance (20.00)

Depreciation (20 × 10% × 9 ÷ 12) 1.50

Tax expenses (30% × (20-1.5)) 5.55

Decrease in profit for the year (12.95)

Effect on the statement of financial position Increase / (decrease) in assets Property, plant and equipment (20 – 1.5) (18.50)

(Increase) / decrease in liabilities

Deferred tax liability (Rs. 18.5 × 30%) 5.55

(Increase) / decrease in equity Retained earnings (18.50 - 5.55) (12.95)

11.2 DUNCAN

Statement of changes in equity (extract)

Retained earnings

Retained earnings

2015 2014 Rs.000 Rs.000 Opening balance as reported 23,950 22,500 Change in accounting policy (W2) 450 400 ––––––– ––––––– Re-stated balance 24,400 22,900 Profit after tax for the period (W1) 4,442 3,250 Dividends paid (2,500) (1,750) ––––––– ––––––– Closing balance 26,342 24,400 ––––––– –––––––

Workings

(1) Revised profit

2015 2014

Rs.000 Rs.000 Per question 4,712 3,200 Add back: Expenditure for the year 600 500 Minus: Depreciation (870) (450) –––––– –––––– Revised profit 4,442 3,250 –––––– ––––––

Page 228: CAF7-Financial Accounting and Reporting II_Questionbank

Financial accounting and reporting II

© Emile Woolf International 218 The Institute of Chartered Accountants of Pakistan

(2) Prior period adjustment

The prior period adjustment is the reinstatement of the Rs.400,000 asset on 1 January 2014 and the Rs.450,000 asset at 1 January 2015. On 31 December 2015 the closing balance above of Rs.26,342,000 can be reconciled as the original Rs.26,162,000 plus the reinstatement of the remaining asset of Rs.180,000.

11.3 MOHANI MANUFACTURING LIMITED

Mohani Manufacturing (Private) Limited Statement of changes in equity For the year ended December 31, 2015

Retained Earnings

Rs. in million

Balance at December 31, 2013 as previously reported (Rs. 89m – Rs. 21m) 68.00

Effect of change in accounting policy (Rs. 37m - Rs. 35.5m) (1.50)

Balance at December 31, 2013 – restated 66.50 Profit for the year ended December 31, 2014 - restated (W1) 39.70

Balance at December 31, 2014 – restated 106.20 Profit for the year ended December 31, 2015 (W2) 8.80

Balance at December 31, 2015 115.00

W1: Profit for the year ended December 31, 2014 (as restated) Rs. in million

Profit as previously reported 21.00 Incorrect recording of depreciation (Rs. 25 million – Rs. 10 million) 15.00 Reversal of FIFO method

Opening inventory 37.00 Closing inventory (42.30)

(5.30) Application of weighted average method

Opening inventory (35.50) Closing inventory 44.50

9.00

39.70

W2: Adjusted profit for year ended June 30, 2015

Profit as per draft financial statements 15.00 Adjustment in Opening Inventory

FIFO 42.30 Weighted average (44.50)

(2.20) Adjustment in Closing Inventory

FIFO (58.40) Weighted average 54.40

(4.00)

Adjusted profit 8.80

Page 229: CAF7-Financial Accounting and Reporting II_Questionbank

Answers

© Emile Woolf International 219 The Institute of Chartered Accountants of Pakistan

CHAPTER 12 – IAS 12: INCOME TAXES

12.1 FRANCESCA

Rs. Rs.

Opening liability 1,340,600

Capital allowances during the year 50,000,000

Depreciation charged during the year (45,000,000)

–––––––––––

5,000,000 30% 1,500,000

–––––––––––

Interest receivable in statement of profit or loss 50,000

Interest received in tax computation (45,000)

–––––––––––

Receivable in statement of financial position

5,000 30% 1,500

–––––––––––

Interest payable in statement of profit or loss 32,000

Interest paid in tax computation (28,000)

–––––––––––

Payable in balance sheet 4,000 30% (1,200)

–––––––––––

Development costs as allowable expense 500,600 30% 150,180

Revaluation 6,000,000

Carrying value (4,900,500)

–––––––––––

Revaluation surplus 1,099,500 x 30% 329,850

––––––––––– ––––––––––

Closing liability 3,320,930

––––––––––

Rs.

Charged to the revaluation reserve 329,850 Charged in the statement of profit or loss (balancing figure) 1,650,480 –––––––––– Total movement on the provision of (3,320,930 – 1,340,600) 1,980,330 ––––––––––

Page 230: CAF7-Financial Accounting and Reporting II_Questionbank

Financial accounting and reporting II

© Emile Woolf International 220 The Institute of Chartered Accountants of Pakistan

12.2 SHEP (I)

(a) Corporate income tax liability - year ended 31st December 2015

Rs.

Profit per accounts 121,000 Add Depreciation 11,000 ———— 133,000 Less tax depreciation (15,000) ———— Taxable profits 117,000 ———— Tax payable @ 30% 35,100 ————

(b) Deferred tax liability

Rs.

Carrying amount (48,000 + 12,000 = 60,000 – 11,000) 49,000 Tax base (48,000 + 12,000 = 60,000 – 15,000) 45,000 ——— Temporary difference (4,000) ———

Deferred tax liability required @ 30% (1,200) ———

(c) Movement on the deferred tax liability

Rs.

Balance b/f Statement of profit or loss (balancing figure) 1,200 ——— Balance c/f 1,200 ———

(d) Statement of profit or loss note

Rs. Current tax expense 35,100 Deferred tax expense 1,200 ———— Tax expense 36,300 ————

Page 231: CAF7-Financial Accounting and Reporting II_Questionbank

Answers

© Emile Woolf International 221 The Institute of Chartered Accountants of Pakistan

12.3 SHEP (II)

(a) Corporate income tax liability - year ended 31st December 2016

Rs.

Profit per accounts 125,000 Add Depreciation 14,000 Interest payable 500 Provision 1,200 Fine 6,000 ———— 146,700 Less tax allowance (given) (16,000) Interest receivable (150) ———— Taxable profits 130,550 ————

Tax payable @ 30% 39,165 ————

(b) Deferred tax liability

Carrying Tax Temporary amount base difference Rs. Rs. Rs.

Tangible assets Carrying amount (49bf – 14) 35,000 Tax base (45bf – 16) 29,000 6,000

Interest payable (25,000 x 8% x 3/12) (500) (500)

Interest receivable (4,000 x 15% x 3/12) 150 150

Provision (1,200) (1,200) ——— ——— ——— 33,450 29,000 4,450 ——— ——— ——— Deferred tax @30% 1,335 ———

(c) Movement on the deferred tax liability

Rs.

Balance b/f 1,200 Statement of profit or loss (balancing figure) 135 ——— Balance c/f 1,335 ———

(d) Statement of profit or loss note

Rs. Current tax expense 39,165 Deferred tax expense 135 ———— Tax expense 39,300 ————

Page 232: CAF7-Financial Accounting and Reporting II_Questionbank

Financial accounting and reporting II

© Emile Woolf International 222 The Institute of Chartered Accountants of Pakistan

(e) Tax reconciliation

Rs. Accounting profit 125,000 ———— Accounting profit @ 30% 37,500 Tax effect of the fine (6,000 @ 30%) 1,800 ———— Tax expense 39,300 ————

12.4 SHEP (III)

(a) Corporate income tax liability - year ended 31st December 2017

Rs.

Profit per accounts 175,000 Add Depreciation 18,500

Interest payable (note) Provision 2,000 Entertainment 20,000 ———— 215,500 Less tax allowance (given) (24,700)

Interest receivable (note) Development costs (17,800) Provision (500) ———— Taxable profits 172,500 ———— Tax payable @ 30% 51,750 ————

Note

There is no adjustment to profit for the interest paid and the interest receivable.

Consider the interest payable. The tax authority will disallow the closing accrual but will allow last year’s accrual (that has been paid in this year) as a deduction. These amounts are equal so there is no net effect.

Similar comments can be made about the interest receivable.

Page 233: CAF7-Financial Accounting and Reporting II_Questionbank

Answers

© Emile Woolf International 223 The Institute of Chartered Accountants of Pakistan

(b) Deferred tax liability

Carrying Tax Temporary amount base difference Rs. Rs. Rs. Tangible assets Carrying amount (35bf – 18.5) 16,500 Tax base (29bf – 24.7) 4,300 12,200

Interest payable (500) (500)

Interest receivable 150 150

Provision (2,700) (2,700)

Development expenditure 17,800 17,800 ——— —— ——— 31,250 4,300 26,950 ——— —— ——— Deferred tax @ 30% 8,085 ———

(c) Movement on the deferred tax liability

Rs.

Balance b/f 1,335 Statement of profit or loss (balancing figure) 6,750 ——— Balance c/f 8,085 ———

(d) Statement of profit or loss note

Rs. Current tax expense 51,750 Deferred tax expense 6,750 ———— Tax expense 58,500 ————

(e) Tax reconciliation

Rs. Accounting profit 175,000 ———— Accounting profit @ 30% 52,500 Tax effect of the fine (20,000 @ 30%) 6,000 ———— Tax expense 58,500 ————

Page 234: CAF7-Financial Accounting and Reporting II_Questionbank

Financial accounting and reporting II

© Emile Woolf International 224 The Institute of Chartered Accountants of Pakistan

12.5 SHEP (IV)

(a) Corporate income tax liability - year ended 31st December 2017

Rs.

Taxable profits (as before) 172,500 ————

Tax payable @ 34% 58,650 ————

(b) Deferred tax liability

Rs.

Temporary difference (as before) 26,950 ———

Deferred tax @ 34% 9,163 ———

(c) Movement on the deferred tax liability

Rs.

Balance b/f 1,335 Adjustment due to change in rate 178 ——— Opening balance restated to 34% (1,335 x 34/30) 1,513 Statement of profit or loss (balancing figure) 7,650 ——— Balance c/f 9,163 ———

(d) Statement of profit or loss note

Rs. Current tax expense 58,650 Deferred tax expense relating to origination and reversal

of temporary differences 7,650 Deferred tax expense resulting from increase in tax rate 178 ———— Tax expense 66,478 ————

(e) Tax reconciliation

Rs. Accounting profit 175,000 ———— Accounting profit @ 34% 59,500 Tax effect of the fine (20,000 @ 34%) 6,800 Increase in opening deferred tax balances due to

change in rate 178 ———— Tax expense 66,478 ————

Page 235: CAF7-Financial Accounting and Reporting II_Questionbank

Answers

© Emile Woolf International 225 The Institute of Chartered Accountants of Pakistan

12.6 WAQAR LIMITED

a) Computation of current period income tax liability 2015 2014 Rs.m Rs.m

Accounting profit before tax 40.00 30.00 Less: Admissible deductions Capital Gain (10.00) (8.00) Tax depreciation on furniture and fittings Rs. 40.5 x 10% (4.05) Rs. 40.5 (1-10%) x 10% (3.65) Tax depreciation on Machinery Rs. 90 x 10% (9.00) Rs. 90 (1-10%) x 10% (8.10) Add: Inadmissible deductions Accounting depreciation on machinery 25.00 25.00 Accounting depreciation on furniture and fittings 5.00 5.00

Taxable profit 48.25 38.95 Tax rate 30% 35%

Tax payable (current tax) 14.48 13.63

b) Deferred taxation computation

NBV (W1)

Tax base (W1)

Temporary difference

Deferred tax liability

Working 2 Rs.m Rs.m Rs.m Rs.m

At December 31,2013

Machinery 175.00 90.00 85.00 29.75 Furniture and fittings 40.00 40.50 (0.50) (0.18)

Deferred tax liability at December 31,2013 (35%)

29.57

At December 31,

2014 Machinery 150.00 81.00 69.00 24.15

Furniture and fittings 35.00 36.45 (1.45) (0.51)

Deferred tax liability at December 31,2014 (35%)

23.64

WDV as at

December 31, 2015 Machinery 125.00 72.90 52.10 15.63

Furniture and fittings 30.00 32.80 (2.80) (0.84)

Deferred tax liability at December 31,2015 (35%)

14.79

Page 236: CAF7-Financial Accounting and Reporting II_Questionbank

Financial accounting and reporting II

© Emile Woolf International 226 The Institute of Chartered Accountants of Pakistan

Working 1 Carrying amount and tax base of machinery NBV Tax base Cost b/f 200.0 200.0 Accumulated depreciation b/f (25.0)

At 31 December 2013 175.0 90.0 Accounting depreciation (200/8 years) (25.0) Tax depreciation (10% of WDV) (9.0)

At 31 December 2014 150.0 81.0 Accounting depreciation (200/8 years) (25.0) Tax depreciation (10% of WDV) (8.1)

At 31 December 2015 125.0 72.9

Carrying amount and tax base of furniture and fittings NBV Tax base Cost b/f 50.0 50.0 Accumulated depreciation b/f (10.0)

At 31 December 2013 40.0 40.5 Accounting depreciation (10% 50) (5.0) Tax depreciation (10% of WDV) (4.05)

At 31 December 2014 35.0 36.45 Accounting depreciation (10% 50) (5.0) Tax depreciation (10% of WDV) (3.65)

At 31 December 2015 30.0 32.8

c) Movement on deferred taxation account (W2) 2015 2014 At January 1 23.64 29.57 Change due to change in rate (23.64 5/35) (3.38) -

20.26 Change due to origination and reversal of temporary

differences in the period (balancing figure) (5.47) (5.93)

At December 31 14.79 23.64

d) Tax expense 2015 2014 Current tax 14.48 13.63 Deferred tax: - Due to origination and reversal of temporary

differences in the period (3.38) Due to change in rate (5.47) (5.93)

Tax expense 5.63 7.7

e) Tax reconciliation 2015 2014 Accounting profit 40.0 30.0 Tax rate 30% 35%

12.0 10.5 Tax effect of untaxed gain: 30% 10.0 (3.0) 35% 8.0 (2.8) Decrease in opening deferred tax balances due to

change in rate (with rounding adjustment) (3.37)

Tax expense 5.63 7.7

Page 237: CAF7-Financial Accounting and Reporting II_Questionbank

Answers

© Emile Woolf International 227 The Institute of Chartered Accountants of Pakistan

12.7 SHAKIR INDUSTRIES

COMPUTATION OF TAX EXPENSE

FOR THE YEAR ENDED DECEMBER 31, 2015 2015

Rs. in million

Profit before tax 15.80 Add: Inadmissible expenses

Accounting depreciation (Rs. 1.1 million + Rs. 0.7 million) 1.80 Financial charges on finance lease 0.15 Penalty paid to SECP 0.70 Provision for gratuity 2.40

5.05 Less: Admissible expenses Rs.m

Tax depreciation 1.65 Lease payments 0.65 Payment of gratuity 1.60 Borrowing cost capitalised 2.30

6.20

Taxable profit for the year 14.65

Current tax expense @ 35% 5.13

COMPUTATION OF DEFERRED TAX EXPENSE FOR THE YEAR ENDED DECEMBER 31, 2015

Carrying amount

Tax base

Temp difference

Rs.m Rs.m Rs.m

Fixed assets – Owned 16.70 13.85 2.85 Fixed assets – Leased 1.80 - 1.80 Capital work in progress 2.30 - 2.30 Provision for gratuity (0.7 + 2.4 – 1.6) (1.50) - (1.50)

Obligation against assets subject to finance lease (1.20) - (1.20)

Total 4.25

Deferred tax expense @ 35% 1.49

Rs. in million

Deferred tax liability (Opening) 0.55

Deferred tax expense for the year (balancing figure) 0.94

Deferred tax liability as at December 31, 2015 (Rs. 4.25 million x 35%) 1.49

Page 238: CAF7-Financial Accounting and Reporting II_Questionbank

Financial accounting and reporting II

© Emile Woolf International 228 The Institute of Chartered Accountants of Pakistan

12.8 MARS LIMITED

(a) Date Particulars Debit Credit Rupees

01.07.2014 Motor Vehicle - Cost 1,600,000 Obligations under finance lease 1,600,000

Capitalisation of the lease

01.07.2014 Obligations under finance lease 480,000 Bank 480,000

First lease payment made in advance

30.06.2015 Finance charges 153,451 Accrued finance charges 153,451

Finance charge accrual for the year ended June 30, 2015 Working: (Rs. 1,600,000 480,000) 13.701% = Rs. 153,451)

30.06.2015 Depreciation 400,000 Accumulated depreciation - Motor

Vehicle 400,000

Depreciation charge for the year ended June 30, 2015 Working: Rs. 1,600,000 ÷ 4 = Rs. 400,000. Assuming that there is no reasonable certainty about transfer of

ownership at the end of lease term.

30.06.2015 Tax expense (W1) 1,492,035 Tax payable 1,492,035

Recognition of tax expense for the year ended June 30, 2015)

30.06.2015 Tax expense 22,035 Deferred tax (W2) 22,035

Recognition of deferred tax asset.

W1 Tax computation Rs.

Accounting profit before tax 4,900,000 Add: Depreciation on leased assets 400,000 Add: Finance charges 153,451 Less: Lease payment (480,000)

Taxable profit 4,973,451

Tax @ 30% 1,492,035

Page 239: CAF7-Financial Accounting and Reporting II_Questionbank

Answers

© Emile Woolf International 229 The Institute of Chartered Accountants of Pakistan

W2 Deferred tax computation

Carrying amount

Tax base

Difference

Taxable temporary difference Leased assets 1,200,000 - 1,200,000 Deductible temporary difference Obligations under finance

lease

(1,120,000) -

(1,120,000) Accrued finance charges (153,451) (153,451)

Net taxable temporary difference

(73,451)

Deferred tax @ 30% (Asset) 22,035

(b) Liabilities against assets subject to finance lease (W3) 2015 Rs.

Present value of minimum lease payments 1,120,000 Less: Current maturity shown under current liabilities (326,549)

793,451

Minimum lease payments (W3) Not later than 1 year 480,000 Later than 1 year and not later than 5 years (480,000 × 2) 960,000

1,440,000 Less: future finance charges on finance lease (320,000)

1,120,000

Present value of finance lease liabilities (W3) Not later than 1 year 326,549 Later than 1 year and not later than 5 years

(371,289 + 422,162) 793,451

1,120,000

The minimum lease payment has been discounted at an interest rate of

13.701% to arrive at their present value. Rentals are paid in annual instalments.

W3: Repayment Schedule

Years Opening Balance

Principal repayment

Interest 13.701%

Annual payment

Closing Balance

Rs.m Rs.m Rs.m Rs.m Rs.m

2015 1,600,000 480,000 480,000 1,120,000 2016 1,120,000 326,549 153,451 480,000 793,451 2017 793,451 371,289 108,711 480,000 422,162 2018 422,162 422,162 57,838 480,000 -

320,000

Page 240: CAF7-Financial Accounting and Reporting II_Questionbank

Financial accounting and reporting II

© Emile Woolf International 230 The Institute of Chartered Accountants of Pakistan

12.9 BILAL ENGINEERING LIMITED

(a) Computation of current taxation

Rs.m Rs.m

Profit before tax 50.000 Add: Accounting depreciation 10.000

Financial charges on lease liability (1.00 – 0.3) × 13.701% 0.096

Amortization of research and development cost for the year 1.000

Less: Tax depreciation (7.000)

Annual instalment of lease payment (0.300)

Amortization of research and development cost (15 × 0.9/10) (1.350)

Current year taxable income 52.446

Tax liability for the year (52.446 × 35%) 18.356 Tax liability for prior periods (0.100 × 35%) 0.035

18.391

Deferred taxation Accounting depreciation 10.000 Tax depreciation (7.000) 3.000

Financial charges on finance lease liability(1.00 – 0.3) × 13.701% 0.096

Annual instalment of lease payment allowed under tax (0.300) (0.204)

Amortization charged in accounts 1.000 Amortization cost claimed in tax (1.350) (0.350)

Excess of taxable income over accounting profit due to time differences 2.446

Deferred tax credit at 35% (0.856)

Total tax expenses (current and deferred) 17.535

(b) Bilal Engineering Limited: Notes to the financial statements for the year ended December 31, 2015

1.1 Relationship between tax expense and accounting

profit 2015 Rs.m

Accounting profit before tax 50.000

Tax on accounting profit at 35% 17.500 Tax on expenses disallowed (Permanent Difference) 0.035

Effective tax rate/tax charge 17.535

(c) Journal entries Debit Credit Rs.m Rs.m

1 Income tax expenses 18.391 Provision for taxation 18.391 (Tax provision for 2015)

2 Deferred tax asset 0.856 Tax expenses – deferred 0.856 (Deferred tax credit for 2015)

Page 241: CAF7-Financial Accounting and Reporting II_Questionbank

Answers

© Emile Woolf International 231 The Institute of Chartered Accountants of Pakistan

12.10 GALAXY INTERNATIONAL

28 : TAXATION 2015 2014 Rs.m Rs.m

Current - for the year (W – 1) 0.84 - Deferred (W – 2) 6.95 (0.96)

7.79 (0.96)

28.1 : Relationship between tax expense and accounting profit Profit/(Loss) before taxation 23.50 (1.75)

Tax at the applicable rate of 35% 8.23 (0.61) Tax effect of exempt income (1.25 x 35%) (0.44) (0.35)

7.79 (0.96)

W1 : Computation of Current Tax (Loss) / profit before tax as per books 23.50 (1.75) Add: Allowable income / Disallowed expenses Accounting depreciation 15.00 15.00 Provision for gratuity 2.20 1.70 Accrued expenses - 2.00 Less: Disallowed income / Allowable expenses Tax depreciation (6.00) (45.00) Interest income from SIBs (Exempt) (1.25) (1.00) Accrued expenses (2.00)

Taxable income / (loss) 31.45 (29.05)

Tax liability (@ 35% 11.01 - Tax loss to be brought forward (29.05 x 35%) (10.17) -

Tax payable 0.84 -

W -2: Computation of Deferred Tax Timing differences (cumulative) on account of: Depreciation (2015: 30-51, 2014: 15-45) 21.00 30.00 Accrued expenses - (2.00) Provision for gratuity (3.90) (1.70 ) Tax losses - (29.05)

17.10 (2.75)

Deferred tax @ 35% 5.99 (0.96) Add: Opening deferred tax (dr.) 0.96 -

Charge/(Reversal) for the year 6.95 (0.96)

Page 242: CAF7-Financial Accounting and Reporting II_Questionbank

Financial accounting and reporting II

© Emile Woolf International 232 The Institute of Chartered Accountants of Pakistan

12.11 APRICOT LIMITED

Taxation 2015 2014 Rs.m Rs.m

Current (W1) 20.48 10.76 Deferred (W2) (1.58) (21.35)

18.90 (10.59)

Relationship between tax expense and accounting profit 2015 Profit before taxation 60.00

Tax at the applicable rate of 35% 21.00 Less: Tax effect of exempt income (2.10)

18.90

W1: Computation of Current Tax Profit before tax as per books 60.00 45.00 Add: Allowable income / Disallowed expenses Accounting depreciation 10.00 9.00 Tax profit on sale of fixed assets 1.00 - Bad debt expense 5.00 7.00

Less: Disallowed income / Allowable expenses Tax depreciation (8.00) (7.00) Accounting profit on sale of fixed assets (0.50) - Capital gain (6.00) - Bad debts written off (3.00) (4.00)

Taxable income 58.50 50.00

Tax liability (@ 35%) 20.48 17.50

2015 2014 Rs.m Rs.m

W2: Computation of Deferred Tax Fixed assets (2014: 95-90, 2015: 82.5-80) (W2.1) 0.87 1.75 Provision for bad debts (2014: 12×35%, 2015: 14×35%)

[W2.2] (4.90) (4.20)

Closing balance of deferred tax (4.03) (2.45) Less: Opening balance (2.45) (18.90)

Charge for the year (1.58) (21.35)

W2.1 Movement of Fixed Assets Accounting Tax Opening balance 95.00 90.00 Disposal during the year (2.50) (2.00) Depreciation for the year - 2015 (10.00) (8.00)

Closing balance 82.50 80.00

W2.2 Movement of provision for bad debts 2015 2014 Opening balance 12.00 9.00 Provision for the year 5.00 7.00 Write off during the year (3.00) (4.00)

Closing balance 14.00 12.00

Page 243: CAF7-Financial Accounting and Reporting II_Questionbank

Answers

© Emile Woolf International 233 The Institute of Chartered Accountants of Pakistan

CHAPTER 13 – RATIO ANALYSIS

13.1 WASIM

Ratios

Year 7 Year 6 Gross profit % =

100 x Sales

prof it Gross

405

2,160 x 100 =19%

362

1,806 x 100 = 20%

Net profit % =

Net prof it

Salesx 100

9

2,160 x 100 = 0.4%

53

1,806 x 100 = 2.9%

Return on capital employed =

Prof it bef ore interest and tax

Share capital and reserv es+Long - term debt capital

15

246 x 100 = 6%

56

190 x 100 = 29%

Asset turnover =

Sales

Share capital and reserv es+Long - term debt capital x 100

2,160

246= 8.8 times

1,806

190= 9.5 times

Current ratio =

Current assets

Current liabilities

422

254=1.7 times

265

147=1.8 times

Quick ratio =

Current assets excluding inv entory

Current liabilities

422 -106

2541.2 times

265 - 61

1471.4 times

Average time to collect =

Trade receiv ables

Sales x 365

316 x 365

2,160 53 day s

198 x 365

1,806 40 day s

Average time to pay =

Trade pay ables

Cost of purchases x 365

198 x 365

1,755= 41 day s

142 x 365

1,444= 36 day s

Inventory turnover =

Inv entory

Cost of sales x 365

106 x 365

1,755 22 day s

61 x 365

1,444=15 day s

13.2 AMIR AND MO

Amir Mo

Gross profit % =

100 x Sales

prof it Gross

90,000

150,000 x 100 = 60%

490,000

700,000 x 100 = 70%

Net profit % =

Net prof it

Salesx 100

44,895

150,000 x 100 = 30%

270,830

700,000 x 100 = 39%

Page 244: CAF7-Financial Accounting and Reporting II_Questionbank

Financial accounting and reporting II

© Emile Woolf International 234 The Institute of Chartered Accountants of Pakistan

Return on capital employed =

Prof it bef ore interest and tax

Share capital and reserv es+Long - term debt capital

Amir

61,500 +500

207,395 +10,000 x 100 = 28.5%

Mo

371,000 +12,000

565,580 +250,000 x 100 = 47%

Asset turnover =

Sales

Share capital and reserv es+Long - term debt capital x 100

Amir

150,000

207,395 +10,000= 0.7 times

Mo

700,000

565,580 +250,000= 0.85 times

Amir Mo

Current ratio =

Current assets

Current liabilities

50,000

22,605= 2.2 times

153,250

117,670=1.3 times

Quick ratio =

Current assets excluding inv entory

Current liabilities

50,000 -12,000

22,605=1.7 times

153,250 - 26,250

117,670=1.1 times

Average time to collect =

Trade receiv ables

Sales x 365

37,500

150,000 x 365 = 91 day s

105,000

700,000 x 365 = 55 day s

Average time to pay =

Trade pay ables

Cost of purchases x 365

22,605

60,000 x 365 =137 day s

117,670

210,000 x 365 = 204 day s

Inventory turnover =

Inv entory

Cost of sales x 365

12,000

60,000 x 365 = 73 day s

26,250

210,000 x 365 = 46 day s

Page 245: CAF7-Financial Accounting and Reporting II_Questionbank

Answers

© Emile Woolf International 235 The Institute of Chartered Accountants of Pakistan

CHAPTER 14 – ETHICAL ISSUES IN FINANCIAL REPORTING

14.1 ETHICAL ISSUES

The range of comments made by Arif raises questions over his ethical behaviour and professional standards.

A chartered accountant should be unbiased when involved in preparing and reviewing financial information. A chartered accountant should prepare financial statements fairly, honestly, and in accordance with relevant professional standards and must not be influenced by considerations of the impact of reported results.

Arif’s failings

Arif appears to be influenced by the need to achieve a specified level of profit. This is not appropriate and calls his integrity into question.

In addition Arif’s professional competence seems to be suspect. His comment on not being up to date on all of the little technicalities in IFRS s suggests that he has not maintained a level of professional competence appropriate to his professional role.

ICAP members have a responsibility to engage in continuing professional development in order to ensure that their technical knowledge and professional skills are kept up to date. Arif should seek continuing professional development activities and improve his knowledge on ethical standards. Furthermore, it might be expected that as Waheed’s superior he should set an example to Waheed and guide him in his responsibilities. Clearly this is not happening.

As a member of ICAP Arif should be aware of the ICAP code of ethics. Arif should know of the danger of self-interest threats and intimidation threats to himself and to others. His attempt to influence the outcome of a fellow professional by applying such a threat to that individual is very unprofessional.

Waheed’s ethical issues

Waheed faces a self-interest threat, in that there is the possibility of a bonus provided the earnings per share figure remains the same as last year. Arif has also suggested that she can influence the Board’s decision over employing him as a replacement finance director – another self-interest threat to Waheed. Both of these threats must be ignored.

Arif’s comments imply that his application of professional responsibility is lacking. This may extend into the way in which the current financial statements have been prepared. Waheed must be very careful (as always) to carry out the review with all due care.

Waheed should first discuss his recommendations with Arif and remind his of his professional responsibilities to ensure that the accounting standards are correctly followed. If the financial statements are found to contain errors or incorrect accounting treatment then they must be amended. If Arif refuses to amend the draft financial statements if necessary Waheed should discuss the matter with other board members (including non- executives and the audit committee, if possible). Further action might include consulting with ICAP.

Page 246: CAF7-Financial Accounting and Reporting II_Questionbank

Financial accounting and reporting II

© Emile Woolf International 236 The Institute of Chartered Accountants of Pakistan

14.2 SINDH INDUSTRIES LTD

(a) Financial reporting issues

Revenue

IAS 18 Revenue sets out the rules to be followed in recognising revenue.

The fact that the customer cannot cancel the contract is not relevant to the recognition of revenue. Revenue from providing a service is recognised according to the stage of completion subject to satisfying criteria set out in IAS 18. In the absence of other information the revenue in this contract should be recognised over the life of the contract as time progresses. As the contract was only signed just before the year end, none of the revenue can be recognised in 2015.

The credit for the amount received should be recognised as a liability. This represents the obligation that the company has to provide the service over the next two years.

The fact that the customer cannot cancel the contract is not relevant to the recognition of revenue. If Sindh Industries failed to provide the service they would be sued for restitution. Therefore the revenue can only be recognised as the service is provided.

New factory

Borrowing costs directly attributable to construction of an asset which necessarily takes a substantial period to get ready for its intended use should be capitalised as part of the cost of that asset under IAS 23 Borrowing Costs. IAS 23 states that the capitalisation of borrowing costs should commence when three conditions are all met for the first time: borrowing costs are being incurred, expenditure is being incurred and activities to prepare the asset are being undertaken. Although borrowing costs were incurred throughout the year and expenditure was incurred from 1 February 2015 (the date the land was purchased), construction only started on 1 June 2015. Therefore this is the date on which capitalisation commences.

Capitalisation ceases when substantially all of the activities required to make the asset ready for use/sale have been completed, that is on 30 September 2015. (The actual date on which the factory was brought into use is irrelevant.) Therefore the period of capitalisation should be four months.

Where construction is financed from general borrowings, the calculation of the amount to be capitalised should be based on the weighted average cost of borrowings. This is:

(Rs.1,000,000 × 9.75%) + (Rs.1,750,000 × 10%) + (Rs.2,500,000 × 8%)/ (Rs.1,000,000 + Rs.1,750,000 + Rs.2,500,000) = 9%

Therefore the amount capitalised should be 9% × Rs.4.5 million (land Rs.1.8 million plus construction costs Rs.2.7 million) × 4/12 = Rs.135,000. The total cost of the factory should be measured at Rs.4,635,000 (Rs.1.8 million plus Rs.2.7 million, plus Rs.135,000). The amount that has been recognised in the statement of financial position should be reduced by Rs.315,000 (Rs.450,000 – Rs.135,000). Finance costs recognised in profit or loss should be increased by Rs.315,000.

Land should not be depreciated because it has an indefinite life. Under IAS 16 Property, Plant and Equipment depreciation charges should start when the asset becomes available for use, from 1 October 2015 in this case.

Page 247: CAF7-Financial Accounting and Reporting II_Questionbank

Answers

© Emile Woolf International 237 The Institute of Chartered Accountants of Pakistan

Depreciation of Rs.35,000 ((Rs.2.7 million, plus (Rs.135,000 × 2.7/4.5) ÷ 20) × 3/12) should be recognised in profit or loss for the year ended 31 December 2015 and the carrying amount of the asset reduced by the same amount to Rs.4.6 million.

Useful life of the blast furnace

Depreciation of the blast furnace has been based on an estimated useful life of 20 years. This is at variance with a report by a qualified expert. The asset valuation specialist treats the furnace as being made up of two components, the main structure and the lining, which must be replaced at regular five yearly intervals over the life of the asset. This is the approach required by IAS 16. The uncertainties inherent in business mean that many items in financial statements cannot be measured with certainty, but estimates should always be made using the most up to date and reliable information. Where estimates have been prepared by professionals with relevant qualifications, then it is nearly always most appropriate to use those estimates. Therefore in accordance with the valuer’s report the main structure of the furnace should be depreciated over 15 years from 1 January 2015 and the lining should be depreciated over five years from that date.

The reassessment of the estimated lives of assets is a change in accounting estimate, rather than a change in accounting policy (IAS 8 Accounting Policies, Changes in Accounting Estimates and Errors). Changes in accounting estimate should be dealt with on a prospective basis. This is achieved by including the effect of the change in profit or loss in current and future periods. The additional depreciation should be calculated as:

Rs.000

Revised depreciation: main structure

((Rs.3.5m – Rs.1.4m)/15 years) 140

lining (Rs.1.4m/5 years) 280

420

Current depreciation (Rs.3.5m/20 years) (175)

Additional depreciation 245

IAS 8 requires the disclosure of the nature and amount of the effect of the change in the estimate of useful lives on the profit for the year.

(b) Revised financial statements

Statement of profit or loss extract for the year ended 31 December 2015

Borrowing Blast

Draft Revenue costs furnace Revised

Rs.000 Rs.000 Rs.000 Rs.000 Rs.000

Profit before tax 2,500 (1,000) (315)+ (35) (245) 905

Page 248: CAF7-Financial Accounting and Reporting II_Questionbank

Financial accounting and reporting II

© Emile Woolf International 238 The Institute of Chartered Accountants of Pakistan

Statement of financial position at 31 December 2015

Borrowing Blast

Draft Revenue costs furnace Revised

Rs.000 Rs.000 Rs.000 Rs.000 Rs.000

Non-current assets Property, plant and equipment 12,000 (315) + (35) (245) 11,405 Current assets 3,500 3,500

Total assets 15,500 14,905

Share capital 2,000 2,000 Retained earnings 6,000 (1,000) (315) + (35) (245) 4,405

Equity 8,000 6,405 Non-current liabilities 5,000 500 5,500 Current liabilities 2,500 500 3,000

Total equity and liabilities 15,500 15,905

(c) Ethical issues

It is noticeable that all the adjustments required reduce profit. This and the background to the previous finance director’s resignation suggest serious problems.

It is not clear who actually prepared the draft financial statements. If they were prepared by more junior staff in the absence of a finance director, some of the adjustments (for example, the calculation of borrowing costs to be capitalised) could be the result of genuine errors or lack of accounting knowledge. However, it seems reasonably clear that the managing director has attempted to influence the treatment of the revenue and the estimated useful life of at least one significant non-current asset. (Note: the directors have reviewed the useful lives of several items of plant and machinery and it is possible that other assets besides the furnace are being depreciated over unrealistically long periods.)

It seems almost certain that the previous finance director resigned as a result of pressure from the managing director (and possibly from other members of the Board) to present the financial statements in a favourable light. The directors intend to seek a stock market listing in the near future. Therefore they have clear motives for manipulating the profit figure and also (perhaps) for making controversial decisions before the financial statements come under much greater scrutiny as a result of the listing. The job title of financial controller is also significant. It suggests that the role has been downgraded and that the person holding it has less authority than the rest of the Board.

Possible courses of action:

Discuss with the managing director the financial reporting standards that apply to the transactions and explain the implications of non-compliance. If the managing director is himself a member of a professional body then it might be worth pointing out to him that he himself is bound by an ethical code.

Advise him that as a Chartered Accountant you are bound by the ICAP code of ethics, and that you would not be prepared to compromise your views of the figures he has prepared for career advancement.

Consider speaking to the other directors (or audit committee if there is one) and seeking their support.

If all of these actions produce a negative response then it would be appropriate to consult the ICAP ethical handbook and/or the Institute.

If all else fails then consider seeking alternative employment.

Page 249: CAF7-Financial Accounting and Reporting II_Questionbank
Page 250: CAF7-Financial Accounting and Reporting II_Questionbank

2015

FINANCIAL ACCOUNTINGAND REPORTING IIQUESTION BANK

Head Office-Karachi: Chartered Accountants Avenue, Clifton, Karachi-75600 Phone: (92-21) 99251636-39, UAN: 111-000-422, Fax: (92-21) 99251626, e-mail: [email protected]

Regional Office-Lahore: 155-156, West Wood Colony, Thokar Niaz Baig, Raiwind Road, Lahore Phone: (92-42) 37515910-12, UAN: 111-000-422, e-mail: [email protected]

Islamabad Office: Sector G-10/4, Mauve Area, Islamabad UAN: 111-000-422, Fax: (92-51) 9106095, e-mail: [email protected]

Faisalabad Office: 36-Z, Commerical Center, Near Mujahid, Hospital Madina Town, Faisalabad Phone: (92-41) 8531028, Fax: (92-41) 8503227, e-mail: [email protected]

Multan Office: 3rd Floor, Parklane Tower, Officers’ Colony, Near Eid Gaah Chowk, Khanewal Road, Multan. Phone: (92-61) 6510511-6510611, Fax: (92-61) 6510411, e-mail: [email protected]

Peshawar Office: House No. 30, Old Jamrud Road, University Town, Peshawar Phone: (92-91) 5851648, Fax: (92-91) 5851649, e-mail: [email protected]

Gujranwala Office: 2nd Floor, Gujranwala Business Center, Opp. Chamber of Commerce, Main G.T. Road, Gujranwala. Phone: (92-55) 3252710, e-mail: [email protected]

Sukkur Office: Admin Block Sukkur IBA, Airport Road, Sukkur Phone: (92-71) 5806109, e-mail: [email protected]

Quetta Office: Civic Business Center, Hali Road, Quetta Cantt Phone: (92-81) 2865533, e-mail: [email protected]

Mirpur AJK Office: Basic Health Unit (BHU) Building Sector D, New City Mirpur, Azad Jammu and Kashmir e-mail: [email protected]